1707 QUESTIONS FOR A CHANCE TO LIVE IN LONDON!

1.    A 65yo man presents with painless hematuria, IVU is normal, prostate is mildly enlarged with mild frequency. What is the most appropriate next step?

  1. US Abdomen
  2. Flexible cystoscopy
  3. MRI
  4. Nuclear imaging
  5. PSA

Ans. The key is B. Flexible cystoscopy. [Painless hematuria in an elderly (here 65 years old man) indicates carcinoma bladder for which flexible cystoscopy is done.

Here BEP is not advanced to cause hemorrhage. There is mild enlargement of prostate and mild symptoms of prostration and hem- orrhage is unlikely at this initial stage of BEP which makes Bladder cancer as the likely cause of painless hematuria.

It is also less likely to be prostate cancer as symptoms of prostration are mild (indicates disease is not advanced). Moreover bleeding in cancer prostate is less common].

 

2.      A 74yo smoker presented to his GP with cough and SOB. Exam revealed pigmentation of the oral mucosa and also over the palms and soles. Tests show that he is diabetic and hypokalemic. What is the most probable dx?

  1. Pseudocushing syndrome
  2. Conns disease
  3. Ectopic ACTH
  4. Cushings disease
  5. Hypothyroidism

Ans. The key is C. Ectopic ACTH. [The patient is smoker and probably developed squamous cell lung cancer which is working as a tumour producing ectopic ACTH causing pigmentation. Resulting raised cortisole is leading to diabetes and hypokalemia (though small cell carcinoma is usual cause but squamous cell carcinoma can produce ectopic ACTH as paraneoplastic syndrome also)].

3.      A 44yo woman has lost weight over 12 months. She has also noticed episodes where her heart beats rapidly and strongly. She has a regular pulse rate of 90bpm. Her ECG shows sinus rhythm. What is the most appropriate inv to be done?

  1. Thyroid antibodies
  2. TFT
  3. ECG
  4. Echocardiogram
  5. Plasma glucose

Ans. The key is B. TFT. [The patient has paroxysmal atrial fibrillation That is why there is no arrhythmia in between attacks. From the given option TFT is the appropriate test as thyrotoxycosis is a leading cause of paroxysmal atrial fibrillation and this ladies weight loss also makes thyrotoxycosis as the probable cause here].

 

4.       A 79yo anorexic male complains of thirst and fatigue. He has symptoms of frequency, urgency and terminal dribbling. His urea and creatinine levels are high. His serum calcium is 1.9 and he is anemic. His BP is 165/95 mmHg. What is the most probable dx?

  1. BPH
  2. Prostate carcinoma
  3. Chronic pyelonephritis
  4. Benign nephrosclerosis

Ans. The key is B. Prostate Carcinoma.

Explanation for Ǫuestion no. 4:

First to say in this case (almost all features goes in favour of prostatic carcinoma like- frequency, urgency and terminal dribbling are features of prostatism; Age, anorexia and anaemia are constitutional features of carcinoma prostate and it would be accurate presentation if it was hypercalcaemia. But given calcium level is of hypocalcaemic level and it is the main cause of discrepancy to this question). Renal failure from ureteral obstruction in carcinoma prostate is a common presenta- tion. Thirst is a feature of hypercalcaemia (here may be erroneously calcium level is given in hypocalcaemic level ; probably a bad recall).

Prostate biopsy is the confirmatory diagnosis and others like PSA is suggestive.

***There are some suggestion that Renal Failure may be the cause of hypocalcemia.

5.      A 64yo man has recently suffered from an MI and is on aspirin, atorvastatin and ramipril. He has been having trouble sleeping and has been losing weight for the past 4 months. He doesn’t feel like doing anything he used to enjoy and has stopped socializing. He says he gets tired easily and can’t concentrate on anything. What is the most appropriate tx?

  1. Lofepramine
  2. Dosulepin
  3. Citalopram
  4. Fluoxetine
  5. Phenelzine

Ans. The key is C. Citalopram. [Among SSRIs Sertraline is the drug of choice for ischemic heart disease. Next choice is citalopram (as it is of- ten related to torsades de pointes it is not 1st choice). If SSRI cannot be used Mirtazapine is recommended as next antidepressant].

 

6.      A 67yo man after a stroke, presents with left sided ptosis and constricted pupil. He also has loss of pain and temp on the right side of his body and left side of his face. Which part of the brain is most likely affected?

  1. Frontal cortex
  2. Cerebellum
  3. Pons
  4. Medulla
  5. Parietal cortex

Ans. The key is D. Medulla. [The name of the condition is “Lateral medullary syndrome” [ipsilateral Horner syndrome and contralateral loss of pain and temperature sense].

 

7.     A 60yo man presents with dysphagia and pain on swallowing both solids and liquids. A barium meal shows gross dilatation of the esophagus with a smooth narrowing at the lower end of the esopha- gus. What is the SINGLE most likely cause of dysphagia?

  1. Achalasia
  2. Myasthenia gravis
  3. Esophageal carcinoma
  4. Esophageal web
  5. Systemic sclerosis

Ans. The key is A. Achalasia. [Dysphagia for both solid and liquid or prominently liquid suggest achalasia where dysphagia to solid

suggest stricture. Also gross dilatation of oesophagus with smooth narrowing at lower end is seen in achalasia. In achalasia dysphagia is usually described as progressive].

 

8.       A man undergoes a pneumonectomy. After surgery, invs show hyponatremia. What could be the cause of the biochemical change?

  1. Removal of hormonally active tumor
  2. Excess dextrose
  3. Excess colloid
  4. Excessive K+
  5. Hemodilution

Ans. The key is A. Removal of harmonically active tumour. [Ectopic ACTH secreting tumour causes hypernatremia and body’s homeo- static mechanism try to lower the level of high sodium and do a lesser degree though sodium remains in hypernatremic level or even it may be normal (this question does not mention any preoperative hyperna- tremia). Removal of that tumour results in negative sodium balance for time being which results hyponatremia while gradually it tends to rise again to normal level].

 

9.      A pregnant lady came with pain in her calf muscle with local rise in temp to the antenatal clinic. What tx should be started?

  1. Aspirin
  2. LMWH
  3. Paracetamol
  4. Cocodamol
  5. Aspirin and heparin

Ans. The key is B. LMWH. [Injections with low molecular weight heparin (LMWH) are usually used to treat pregnant women with DVT. LMWH is an anticoagulant, which means it prevents the blood clot getting bigger. It does not affect the developing baby ( Ref: nhs. uk)].

 

10.         A 53yo female presents with an acute painful hot knee joint. She is a known case of RA. On examination, the knee is red, tender and swollen. The hamstring muscles are in spasm. Her temp is 38.5C and BP is 120/80mmHg. What is the SINGLE best next inv?

  1. Joint aspiration for cytology and culture and sensitivity
  2. Joint aspiration for positive birefrengent crystals
  3. Joint aspiration for negative birefrengent crystals
  4. Blood culture
  5. Serum uric acid

Ans. The likely key is A. Joint aspiration for cytology and culture and sensitivity. [Case of septic arthritis. Any chronically arthritic joint

is predisposed to infection. Moreover chronic use of steroid in Rh. arthritis is one of the important predisposing factor. In this age group likely organism is Staphylococcus. In younger age group Neisseria gonorrhea is more common].

11.      An 80yo man presented with pain in his lower back and hip. He also complains of waking up in the night to go to the washroom and has urgency as well as dribbling. What is the most likely dx?

  1. BPH
  2. Prostatitis
  3. UTI
  4. Prostate carcinoma
  5. Bladder carcinoma

Ans. The key is D. Prostate carcinoma. [Age, nocturia, urgency and dribbling points towards prostate pathology. Pain of lower back and hip points towards bony metastases from prostate cancer. Blood test for PSA; Prostate biopsy; MRI (if initial biopsy is negative, to decide repeat biopsy).

Treatment options: 1. Active treatment [i) radical prostatectomy ii) radical radiotherapy iii) hormone therapy iv) brachytherapy v) pelvic radiotherapy vi) orchidectomy

  1. Active surveillance
  2. Watchful waiting

t4. Palliative care (Source: NICE)].

 

 

 

 

12.     An 18yo female has peri-orbital blisters. Some of them are crust- ed, others secreting pinkish fluid. What is the most likely dx?

  1. Shingles
  2. Chicken pox
  3. Varicella
  4. Rubella
  5. Measles

Ans. The key is A. Shingles. [Here ophthalmic division of trigeminal nerve is involved. Typically shingles are unilateral].

 

13.      A 29yo lady who is a bank manager is referred by the GP to the medical OPC due to a long hx of tiredness and pain in the joints. An autoimmune screen result showed smooth muscle antibodies positive. What is the most appropriate next inv?

  1. ECG
  2. TFT
  3. LFT
  4. Serum glucose
  5. Jejunal biopsy

Ans. The key is C. LFT. [A case of autoimmune hepatitis. Autoimmune hepatitis is an uncommon cause of chronic hepatitis which if un-

treated can lead to cirrhosis. However with treatment outlook is very good. Smooth muscle antibody is positive in autoimmune hepatitis. Definitive investigation is liver biopsy. Treated with steroid [start with high dose prednisolone]. Azathioprine is commonly added with steroid to reduce its dose as steroid has more side effects than azathioprine].

 

14.      A 5yo with recurrent chest pain, finger clubbing with offensive stool. Choose the single most likely inv?

  1. Endomyseal/Alpha glidin antibody
  2. Sweat test
  3. Barium meal
  4. ECG
  5. Glucose tolerance test

Ans. The key is B. Sweat test. [Recurrent chest pain from frequent lung infections including pneumonia or bronchitis.”Clubbing” of the fingers is a classic features of Cystic Fibrosis, although not present in many patients. The digestive enzymes are not being produced, food is not adequately digested (malabsorption) and excess fat and protein is lost in the stools, making them bulky, oily, smelly and difficult to flush away].

 

15.     A clinical picture of breast cancer originated from the mammary duct. Biopsy was done and there were neoplastic cells found. Choose the histological picture of the cancer.

 

  1. Neoplastic cells are arranged in small clusters occupying a space between collagen bundles (Seirrhous carcinoma)
  2. Spindle cell neoplasms with margins, which infiltrate adjacent structure, fat invaded (Breast sarcoma)
  3. Small cells with round nucleus and scant indistinct cytoplasm (Lob- ular carcinoma)

Ans. The key is C. Small cells with round nucleus and scant indistinct cytoplasm (Lobular carcinoma)

 

16.      A 22yo man has a reduced conscious level and a fixed dilated pupil after being involved in a MVC. Choose the single most appropriate option?

  1. Facial nerve
  2. Oculomotor nerve
  3. Olfactory nerve
  4. Optic nerve
  5. Trigeminal nerve

Ans. The key is B. Oculomotor nerve. [3rd nerve damage can cause fixed dilated pupil].

 

17.    A man with suspected active TB wants to be treated at home. What should be done to prevent the spread of disease?

  1. Immediate start of the tx with Anti-TB drugs
  2. All family members should be immediately vaccinated with BCG vaccine
  3. Patient should be isolated in a negative pressure chamber in his house
  4. Universal prevention application protocol

Ans. The key is D. Universal prevention application protocol.

 

18.      A 7yo child is brought to the ED with a 1 day hx of being listless. On examination, the child is drowsy with an extensive non-blanching rash. What advice would you give the parents?

  1. All family members need antibiotic therapy
  2. Only the mother should be given rifampicin prophylaxis
  3. All family members need isolation
  4. All family members should be given rifampicin prophylaxis

Ans. The key is D. All family members should be given rifampicin pro- phylaxis. [Meningococcal disease. Diagnosis is done with blood or CSF PCR. Initial prehospital management: Benzyl penicillin or cefotaxime].

 

19.     A 47yo man has a temp of 39C and is delirious. He has developed blisters mainly on his trunk, which appeared a few hours ago. He is well and not on any medications. He last travelled 5 months ago to Italy. Which of the following is the most likely dx?

  1. Shingles
  2. Chicken pox
  3. Pemphigoid
  4. Bullous pemphigus

Ans. The key is B. Chicken pox. [Centripetal distribution of blisters favours chickenpox. Adults more commonly develop a more general- ized brain inflammation (“encephalitis”) whose symptoms may include delirium and seizures. Incubation period of chicken-pox is 10-21 days. So this travel history is not significant].

 

20.        A 64yo pt has been having freq episodes of secretory diarrhea, which is extremely watery, with large amounts of mucus. A dx of villous adenoma was made after endoscopy. What electrolyte abnor- mality is most likely in this pt?

  1. Hyperkalemia
  2. Hypernatremia
  3. Hyponatremia
  4. Hypokalemia

Ans. Key not given. Correct key is both C and D! [Villous adenoma can cause both hyponatremia and hypokalemia].

 

21.     A pt with an acute gout attack came to the ED. What drug should be given to relieve symptoms?

  1. NSAIDs
  2. Allopurinol
  3. Ibuprofen

Ans. The key is A. NSAIDs. [Oral NSAIDs commenced immediately and continue for 1 – 2 weeks; Colchicine can be effective alternative but is slower to work than NSAIDs. Intra articular corticosteroids are highly effective in acute gouty monoarthritis.

 

22.       A pt was lying down on the operating table in a position with his arms hanging down for 3 hours. Soon after he woke up, he com- plains of numbness and weakness in that hand and has limited wrist movement/wrist drop and sensory loss over dorsum of that hand, weakness of extension of the fingers and loss of sensation at the web of the thumb. What structure is likely to be damaged?

  1. Radial nerve
  2. Median nerve
  3. Ulnar nerve
  4. Axillary nerve
  5. Suprascapular nerve

Ans. The key is A. Radial nerve. [Here arm hanging down compressing the radial nerve at the spiral groove is the cause of given scenario].

 

23.       A pt who was previously on 120mg slow release oral morphine has had his dose increased to 200mg. He is still in significant pain. He complains of drowsiness and constipation. What is the next step in the management?

  1. Increase slow release morphine dose
  2. Fentanyl patch
  3. Replace morphine with oral hydromorphone
  4. Replace morphine with oxycodone
  5. Subcutaneous morphine

Ans. The key is D. Replace morphine with oxycodone.

24.        A 40yo woman notices increasing lower abdominal distention with little/no pain. On examination, a lobulated cystic mass is felt and it seems to be arising from the pelvis. What is the most appro- priate inv?

  1. CA 125
  2. CA 153
  3. CA 199
  4. CEA
  5. AFP

Ans. The key is A. CA 125. [Ovarian ca is the likely diagnosis for which tumour marker is CA 125].

 

25.       A resident of a nursing home presented with rashes in his finger webs and also on his abdomen, with complaints of itching which

is severe at night. He was dx with scabies. What the best tx for his condition?

  1. 5% permethrin
  2. Doxycycline
  3. 5% permethrin
  4. Reassure
  5. Acyclovir

Ans. The key is C. %5 permethrin. [Scabies outbreaks in nursing homes and cases of crusted scabies may require combination therapy consisting of topical application of permethrin and 2 oral doses of ivermectin at 200 mcg/kg (administered 1 wk apart)].

 

26.       A 34yo alcoholic is found passed out in front of a local pub. The ambulance crew informs you that he was sweating when they found him and there were cans of cider lying empty around him. What is the initial stage of inv?

  1. Capillary blood sugar
  2. CT head
  3. MRI head
  4. ABG
  5. MCV

Ans. The key is A. Capillary blood sugar. [Alcohol induced hypoglycemia can present as this case].

 

27.       A young boy fell on his outstretched hand and has presented with pain around the elbow. He has absent radial pulse on the affected hand. What is the most likely dx?

  1. Dislocated elbow
  2. Angulated supracondylar fx
  3. Undisplaced fx of radial head
  4. Posterior dislocation of shoulder

Ans. The key is B. Angulated supracondyllar fx. [Damage or occlusion of the bracheal artery is the cause of absent radial pulse. Often closed reduction results in restoration of normal anatomy and correction of occlusion of bracheal artery and establishes circulation again but in few instances open reduction is required to fix the occluded artery].

 

28.        A 65yo woman presented with transient arm and leg weakness as well as a sudden loss of vision in the left eye. Her symptoms resolved within the next couple of hours. What is the most appropriate next inv?

  1. CT brain
  2. Echo
  3. Doppler USG
  4. Arteriography
  5. 24h ECG

Ans. The key is c. [A case of TIA. Probable cause carotid artery atherosclerosis].

 

29.       A man complains of loss of sensation in his little and ring finger. Which nerve is most likely to be involved?

  1. Median nerve
  2. Ulnar nerve
  3. Radial nerve
  4. Long thoracic nerve
  5. Axillary nerve

Ans. The key is B. Ulner nerve. [Compression of ulner nerve at the elbow, 30. known as cubital tunnel syndrome, causes numbness in the 5th (pinky) finger, along the half (lengthwise) of the 4th (ring) finger closest to the 5th finger, and the back half of the hand over the 5th finger]. 30. A young man complains of double vision on seeing to the right. Which nerve is most likely to be involved?

  1. Left abducens
  2. Right abducens
  3. Left trochlear
  4. Right trochlear
  5. Right oculomotor

Ans. The key is B. Right abducens. [Diplopia on seeing to right

indicates right lateral rectus palsy which is supplied by right abducent nerve].

 

31.   A 45yo man keeps having intrusive thoughts about having dirt under the bed. He can’t keep himself from thinking about these thoughts. If he tries to resist, he starts having palpitations. What is the most likely dx?

  1. OC personality
  2. OCD
  3. Schizophrenia
  4. Panic disorder
  5. Phobia

Ans. The key is B. OCD. [Here patients thoughts are obsession and though no compulsive act is described (like repeated cleansing of dirt) but his nature of thought like inability to resist the thinking or getting palpitation on trying to avoid thinking can be regarded as compulsion of thought. CBT 1st line. SSRIs].

 

32.    A 33yo man presents with an itchy scaly annular rash on his thigh after a walk in the park. Which of the following drugs will treat his condition?

  1. Erythromycin
  2. Doxycycline
  3. Penicillin
  4. Amoxicillin

Ans. The key is B. Doxycycline. [Itchy scaly annular rash after a walk in the park indicates erythema migrans caused by the spirochete Borrelia Burgdorferi transmitted by bite of pinhead-sized ixodes ticks leading to lyme disease].

 

33.    A pt with cerebral mets has polyuria and polydipsia. What part of the brain would be affected?

  1. Cerebral cortex
  2. Cerebellum
  3. Diencephalon
  4. Pons
  5. Medulla

Ans. The key is C. Diencephalon. [Diencephalon is the caudal (posterior) part of the forebrain, containing the epithalamus, thalamus, hypothalamus, and ventral thalamus and the third ventricle. Hypothalamus produce ADH and hens lesion of diencephalon (hypo- thalsamus) may produce cranial diabetes insipidus.

34.    A 32yo man presented with painless hematuria. He is hypertensive but the rest of the exam is unremarkable. What is the most likely dx?

  1. Polycystic kidneys
  2. Ca bladder
  3. Ca prostate
  4. TTP
  5. HUS

Ans. The key is A. Polycystic kidneys. [Painless haematuria at an younger age with hypertension is suggestive of polycystic kidney disease. Renal ultrasound is used to diagnose the condition].

 

35.    A 45yo female complains of pain in the inner side of her right thigh. She was dx with benign ovarian mass on the right. Which nerve is responsible for this pain?

  1. Femoral nerve
  2. Obturator nerve
  3. Iliohypogastric nerve
  4. Ovarian branch of splanchic nerve
  5. Pudendal nerve

Ans. The key is B. Obturator nerve. [The Obturator nerve is respon- sible for the sensory innervation of the skin of the medial aspect of the thigh].

 

36.    A 37yo lady strongly believes that a famous politician has been sending her flowers every day and is in love with her. However, this is not the case. What is the most likely dx?

  1. Erotomania
  2. Pyromania
  3. Kleptomania
  4. Trichotillomania
  5. Grandiosity

Ans. 1. The key is A. Erotomania. [Erotomania is a type of delusion in which the affected person believes that another person, usually a stranger, high-status or famous person, is in love with them].

Pyromania is an impulse control disorder in which individuals repeat- edly fail to resist impulses to deliberately start fires, in order to relieve tension or for instant gratification.

Kleptomania is the inability to refrain from the urge to steal items.

Trichotillomania is an impulse disorder characterized by the com- pulsive urge to pull out one’s hair, leading to noticeable hair loss and balding.

Grandiosity refers to an unrealistic sense of superiority.

 

37.   A 3yo child has been brought with facial lacerations. On exam- ination he has some cuts over his right cheek and under the eye. The GCS on initial evaluation is 15. What is the appropriate next inv?

  1. Skull XR
  2. Facial XR
  3. CT scan
  4. MRI
  5. Observation

Ans. The key is B. Facial X-ray. [Normal GCS makes intracranial lesion less likely. As there is facial injury to exclude any facial bone fracture we can do facial X-ray].

 

38.    A 73yo woman has lymphadenopathy and splenomegaly. She feels well but has had recurrent chest infections recently. Choose the single most likely blood film findings?

  1. Atypical lymphocytes
  2. Excess of mature lymphocytes
  3. Plasma cells
  4. Multiple immature granulocytes with blast cells
  5. Numerous blast cells

Ans. The key is B. Excess of mature lymphocytes. [Dx is CLL. Age of patient (usually above 50 yrs), lymhadenopathy and splenomegaly, appearance of lymphocytes (mature lymphocytes – but functionally not normal). Repeated chest infection points towards abnormal func- tion of lymphocytes against infection].

 

 

 

39.    A lady presents with itching around the breast and greenish foul smelling discharge from the nipple. She had a similar episode before. What is the most likely dx?

  1. Duct papilloma
  2. Duct ectasia
  3. Breast abscess
  4. Periductal mastitis
  5. Mammary duct fistula

Ans. The key is B. Duct ectasia. [Duct ectasia of the breast or mam- mary duct ectasia or plasma cell mastitis is a condition in which the lactiferous duct becomes blocked or clogged. This is the most com- mon cause of greenish discharge. Mammary duct ectasia can mimic breast cancer. It is a disorder of peri- or post-menopausal age].

40.     A young male whose sclera was noted to be yellow by his col- leagues has a hx of taking OTC drugs for some pain. Tests showed raised bilirubin, ALT and AST normal. The provocation test with IV nicotinic acid is positive and produces further rise in the serum bilirubin levels. What is the most likely dx?

  1. Acute hepatitis
  2. Drug hypersensitivity
  3. Gilberts syndrome
  4. Acute pancreatitis

Ans. The key is C. Gilbert’s syndrome. [Only bilirubin is increased but not the liver enzymes. Also positive nicotinic acid provocation test is in its favour].

 

41.A 24yo biker has been rescued after being trapped under rocks for almost 12h. He complains of reddish brown urine. His creatinine is 350umol/L and his urea is 15mmol/L. What is the most imp step in the management of this patient?

  1. Dialysis
  2. IV NS
  3. IV dextrose
  4. IV KCl
  5. Pain relief

Ans. Key is B. IV NS. [It is a case of rhabdomyolysis which is initially treated with IV NS].

 

42.  A 74yo man who has been a smoker since he was 20 has recently been dx with SCLC. What serum electrolyte picture will confirm the presence of SIADH?

  1. High serum Na, low serum osmolarity, high urine osmolarity
  2. Low serum Na, low serum osmolarity, high urine osmolarity
  3. Low serum Na, high serum osmolarity, high urine osmolarity
  4. High serum Na, low serum osmolarity, low urine osmolarity
  5. High serum Na, high serum osmolarity, low urine osmolarity

Ans. The key is B. Low serum Na, low serum osmolarity, high urine osmolarity.

43.  A man brought into the ED after being stabbed in the chest. Chest is bilaterally clear with muffled heart sounds. BP is 60/nil. Pulse is 120bpm. JVP raised. What is the most likely dx?

  1. Pulmonary embolism
  2. Cardiac tamponade
  3. Pericardial effusion
  4. Hemothorax
  5. Pneumothorax

Ans. The key is B. Cardiac tamponade. [Chest is clear, so there is no pneumothorax or pleural effusion. Muffled heart sound is due to fluid in pericardial space, low BP from reduced chamber expansion due to pericardial fluid’s pressure and restricted right heart expansion causes raised JVP].

 

44.  A 50yo pt is admitted for elective herniorraphy. Which of the following options will lead to a postponement of the operation?

  1. SBP 110mmHg
  2. MI 2 months ago
  3. Hgb 12g/dl
  4. Pain around hernia
  5. Abdominal distention

Ans. The key is B. MI 2 months ago. [After MI elective surgery should not be done before 6 months post MI, as operation in earlier than this time has significant increase in mortality].

 

 

 

45.  A 32yo woman of 39wks gestation attends the antenatal day unit feeling very unwell with sudden onset of epigastric pain associated with nausea and vomiting. Her temp is 36.7C. Exam: she is found

to have RUQ tenderness. Her blood results show mild anemia, low platelets, elevated liver enzymes and hemolysis. What is the most likely dx?

  1. Acute fatty liver of pregnancy
  2. Acute pyelonephritis
  3. Cholecystitis
  4. HELLP syndrome
  5. Acute hepatitis

Ans. The key is D. HELLP syndrome. [HELLP syndrome is a

life-threatening liver disorder thought to be a type of severe pre- eclampsia. It is characterized by Hemolysis (destruction of red blood cells), Elevated Liver enzymes (which indicate liver damage), and Low Platelet count. HELLP is usually related to preeclampsia. The main treatment is to deliver the baby as soon as possible [as early as after 34 weeks if multisystem disease is present].

 

46.  A woman comes with an ulcerated lesion 3 cm in the labia majo- rum. What is the lymphatic drainage of this area?

  1. External iliac
  2. Superficial inguinal LN
  3. Para-aortic
  4. Iliac
  5. Aortic

Ans. Key is B. Superficial inguinal LN.

 

47. A man post-cholecystectomy presented with jaundice, fever and dark urine. What is the most diagnostic inv?

  1. ERCP
  2. USG Abdomen
  3. CT Scan
  4. MRCP
  5. MRI

Ans. The key is A. ERCP [Post operative US will not give good results. We shall not go for ercp first as it has complications like injury and pancreatitis. Acceptable options are CT, MRI and MRCP among which most appropriate is MRCP! But as the Question wants most diagnos- tic it is ERCP (though not practical)!!! The diagnosis here is choledo- colithiasis with cholangitis].

 

48.  A 79yo stumbled and sustained a minor head injury 2 weeks ago. He has become increasingly confused, drowsy and unsteady. He has a GCS of 13. He takes warfarin for Afib. What is the most likely dx?

  1. Extradural hemorrhage
  2. Cerebellar hemorrhage
  3. Epidural hemorrhage
  4. Subdural hemorrhage
  5. Subarachnoid hemorrhage

Ans. 1. The key is D. Subdural hematoma. [In elderly head injury usu- ally leads to subdural hematoma even if head injury is minor or trivial and extradural hematoma in elderly is extremely uncommon even in more severe head injury. Management: 1st line: Evacuation by barr hole craniostomy. 2nd line: Craniotomy if the clot is organized. Up to 3 weeks burrhole may be possible but longer than this clot is mostly organized and Flap craniotomy is usually required].

49.  A 25yo female complains of intermittent pain in her fingers. She describes episodes of numbness and burning of the fingers. She wears gloves whenever she leaves the house. What is the most probable dx?

  1. Kawasaki disease
  2. Takayasu arteritis
  3. Buerger’s disease
  4. Embolism
  5. Raynaud’s phenomenon

Ans. The key is E. Raynaud’s phenomenon. [Intermittent nature points towards some triggers and wearing of gloves during going out indicates cold weather. Also female sex makes the diagnosis of Raynaud’s phenomenon more likely].

 

50.  A 22yo lady has been unwell for some time. She came to the hospital with complaints of fever and painful vesicles in her left ear. What is the most probable dx?

  1. Acne
  2. Herpes zoster
  3. Chicken pox
  4. Insect bite
  5. Cellulitis

Ans. The key is B. Herpes Zoster. [This is a case of Herpes zoster oticus and if facial nerve is also involved then it is called Ramsay Hunt syndrome].

 

51. A 5yo girl had earache and some yellowish foul smelling discharge, perforation at the attic and conductive hearing loss. She has no past hx of any ear infections. What is the most appropriate dx?

  1. Acute OM
  2. OM with effusion
  3. Acquired cholesteatoma
  4. Congenital cholesteatoma
  5. Otitis externa

Ans. The key is c. Acquired cholesteatoma. [Acquired cholesteatomas develop as a result of chronic middle ear infection and are usually associated with perforation of the tympanic membrane at the attic (mass is seen in attick with perforation at pars flaccida- in contrast to medial to tympanic membrane which is in congenital). Clinical presen- tation usually consists of conductive hearing loss, often with purulent discharge from the ear].

In congenital:

  • mass medial to the tympanic membrane
  • normal tympanic membrane
  • no previous history of ear discharge, perforation or ear

[discrepancy like no previous infection is the only point in favour of congenital here and may be due to a bad recall!!!]

 

52. A female with T1DM would like to know about a deficiency of vita- mins in pregnancy that can be harmful. A deficiency of which vitamin can lead to teratogenic effects in the child?

  1. Folic acid
  2. Vit B12
  3. Thiamine
  4. Riboflavin
  5. Pyridoxine

Ans. The key is A. Folic acid. [Frequently associated with neural tube defect].

 

 

53. A 23yo woman has been having pain at the base of her thumb, the pain is reproduced when lifting her 3 month old baby or changing diapers and also with forceful abduction of the thumb against resis- tance. What is the likely cause?

  1. Avascular necrosis of scaphoid
  2. Trigger
  3. De Quervain’s tenosynovitis

Ans. The key is c. De Quervain’s tenosiovitis. [Can be diagnosed by Finkelstein’s test:

The physician grasps the thumb and the hand is ulnar deviated sharp- ly. If sharp pain occurs along the distal radius (top of forearm, about an inch below the wrist), de Quervain’s syndrome is likely].

 

54.  A 6m child presents with fever and cough. His mother has rushed him to the ED asking for help. Exam: temp=39C and the child is feeding poorly. Dx?

  1. Bronchiolitis
  2. Asthma
  3. Bronchitis

Ans. The key is A. Bronchiolitis. [Management: 1. Oxygen inhalation

  1. Nasogastric feeding. DON’T USE: i) bronchodilator ii) steroid iii) antibiotics routinely. [OHCS, 9th edition, page-160]

 

55.    A 75yo man collapsed while walking in his garden. He recovered fully within 30 mins with BP 110/80 mmHg and regular pulse of 70bpm. He has a systolic murmur on examination. His activities have been reduced lately which he attributes to old age. What is the definitive diagnostic inv that will assist you with his condition?

  1. ECG
  2. Echo
  3. 24h ECG monitoring
  4. 24h BP monitoring
  5. Prv CIN

Ans. B. Echo. [2 mainD/D. Aortic stenosis and hypertrophic cardio- myopathy. Aortic stenosis – more likely in elderly. And hypertrophic cardiomyopathy – less likely in this age].

 

56.     A 35yo man with a hx of schizophrenia is brought to the ER by his friends due to drowsiness. On examination he is generally rigid. A dx of neuroleptic malignant syndrome except:

  1. Renal failure
  2. Pyrexia
  3. Elevated creatinine kinase
  4. Usually occurs after prolonged tx
  5. Tachycardia

Ans. The key is D. Usually occurs after prolonged tx. [Malignant neuro- leptic syndrome usually occurs SOON after start or in increasing dose of antipsychotic drugs! All others mentioned are seen in this syn- drome like renal failure, pyrexia, elevated creatinine kinase, tachycardia etc].

57.   A 33yo drug addict wants to quit. She says she is ready to stop the drug abuse. She is supported by her friends and family. What drug tx would you give her?

  1. Benzodiazepines
  2. Diazipoxide
  3. Lithium
  4. Methadone
  5. Disulfiram
    • What is the key.
    • What drugs should you use in i) tobacco abuse and in ii) alcohol abuse?

Ans. 1. Key is d. Methadone. (used in opiate abuse)

Ans. 2. i) tobacco abuse: a) bupropion ii) alcohol: a) acamprosate b) disulfirum

 

58.    A 16m child presents with drooling, sore throat and loss of voice. He has fever with a temp of 38.2C. What is your next step towards management?

  1. Direct pharyngoscopy
  2. Call ENT surgeon
  3. Call anesthesiologist
  4. IV fuilds
  5. Start antibiotics
  6. 1. What is the key?
  7. 2. What is the diagnosis?
  8. 3. What is the urgent management?

Ans. 1.The key is C. Call anesthesiologist. [As an expert to intubate].

Ans. 2. Diagnosis is Acute epiglottitis.

Ans. 3. In given case urgent intubation is needed to secure airway to prevent closure of airway.

 

59.    A 62yo woman complains of unsteadiness when walking. On examination she has pyramidal weakness of her left lower limb and reduced pain and temp sensation on right leg and right side of trunk up to the umbilicus. Joint position sense is impaired at her left great toe but is normal elsewhere. She has a definite left extensor plantar response and the right plantar response is equivocal. Where is the lesion?

  1. Left cervical cord
  2. Midline mid-thoracic cord
  3. Right mid-thoracic cord
  4. Left mid-thoracic cord
  5. Left lumbo-sacral plexus
  6. 1. What is the key?
  7. 2. What is the name of this condition?

Ans. 1. The key is d. Left mid-thoracic cord.

Ans. 2. Brown-sequard syndrome. [In brown-sequard syndrome paralysis and loss of proprioception on the same (or ipsilateral) side as the injury or lesion, and loss of pain and temperature sensation on the opposite (or contralateral) side as the lesion].

 

60.     A 26yo man presents to ED with increasing SOB on left side and chest pain. He has been a heavy smoker for the past 4 years. He doesn’t have any past med hx. What is the likely dx?

  1. Pulmonary embolism
  2. MI
  3. Asthma
  4. Pleural effusion
  5. Pneumothorax

 

Ans. The key is e. Pneumothorax. [Increased shortness of breath and chest pain with no past medical history favours the dx of pneumo- thorax. Heavy smoking or tobacco is a risk factor for spontaneous pneumothorax].

 

61.   A pt with hepatocellular ca has raised levels of ferritin. What is the most probable cause?

  1. Hemochromatosis
  2. A1 antitrypsin def
  3. Cystic fibrosis

Ans. The key is A. Haemochromatosis. [Hemochromatosis itself is a cause of hepatocellular carcinoma and associated with raised level of ferritin. Serum ferritin levels elevated higher than 200 mcg/L in

premenopausal women and 300 mcg/L in men and postmenopausal women indicate primary iron overload due to hemochromatosis, espe- cially when associated with high transferrin saturation and evidence of liver disease. Ferritin concentration higher than 1000 mcg/L suggests liver damage with fibrosis or cirrhosis].

 

62.    A woman has electric pains in her face that start with the jaw and move upwards. Her corneal reflexes are normal. What is the most likely dx?

  1. Atypical face pain
  2. Trigeminal neuralgia
  3. Tempero-mandibular joint dysfunction
  4. GCA
  5. Herpes zoster

Ans. 1. Key is b. Trigeminal neuralgia. [Electric pains in her face that starts with the jaw and moves upwards [this description indicates neurological pain of trigeminal nerve. In trigeminal neuralgia tran- sient loss of corneal reflexs are seen just after attacks but in between attacks corneal reflexes are quite normal].

 

63.    A 32yo man presented with slow progressive dysphagia. There is past hx of retro-sternal discomfort and he has been treated with prokinetics and H2 blockers. What is the probably dx?

  1. Foreign body
  2. Plummer vinson syndrome
  3. Pharyngeal puch
  4. Peptic stricture
  5. Esophageal Ca

Ans. 1. The key is D. Peptic stricture. [Progressive dysphagia to mostly solid is suggestive of peptic stricture which is supported here by the use of prokinetic drugs and H2 blockers which are used for reflux oesophagitis].

55.    A 56yo man comes with hx of right sided weakness & left sided visual loss. Where is the occlusion?

  1. Ant meningeal artery
  2. Mid meningeal artery
  3. Mid cerebral artery
  4. Carotid artery
  5. Ant cerebral artery
  6. Ant communicating artery
  7. 1. What is the key?
  8. 2. How will you differentiate between middle cerebral artery occlu- sion from anterior cerebral artery occlusion?

Ans. 1. The key is d. Carotid artery. [Carotid artery divides to internal and external carotid of which internal continues as middle cerebral ultimately. But just before it becomes middle cerebral internal carotid gives rise to ophthalmic branch. So middle cerebral occlusion may give partial visual loss but not complete mono-ocular blindness. For complete mono-ocular blindness occlusion should be proximal to ophthalmic artery i.e. either in internal carotid or more proximally to carotid artery].

Ans. 2. Middle cerebral artery occlusion: paralysis or weakness of contralateral face and arm (faciobracheal). Sensory loss of the contra- latera face and arm.

Anterior cerebral artery occlusion: paralysis or weakness of the contra- lateral foot and leg. Sensory loss at the contralateral foot and leg.

 

56.    A young college student is found in his dorm unconscious. He has tachyarrhythmia and high fever. He also seems to be bleeding from his nose, which on examination shows a perforated nasal septum. What is the most likely dx?

  1. Marijuana OD
  2. Cocaine OD
  3. Heroin OD
  4. Alcohol OD
  5. CO poisoning
  6. 1. What is the key?
  7. 2. What are the points that favours the diagnosis in given ques- tion?
  8. 3. What are other important findings?

Ans. 1. Key is B. Cocaine overdose.

Ans. 2. Points in favour: i) Tachyrhythmia ii) High fever iii) perforated nasal septum iv) unconsciousness

Ans. 3. Other findings of cocaine toxicity: i) Psychiatric: anxiety, para- noia ii) Tachypnoea iii) Increased energy and talking rapidly iv) Dilated pupils. Also: [rhabdomyolysis, metabolic acidosis, convulsion].

57.    A 56yo pt whose pain was relieved by oral Morphine, now pres- ents with progressively worsening pain relieved by increasing the dose of oral morphine. However, the pt complains that the increased morphine makes him drowsy and his is unable to carry out his daily activities. What is the next step in his management?

  1. Oral oxycodone
  2. Oral tramadol
  3. PCA
  4. IV Fentanyl
  5. Diamorphine

Ans. The key is oral oxycodon.

 

58.   A 30yo man presents with a 5cm neck mass anterior to the ster- nocleido-mastoid muscle on the left side in its upper third. He states that the swelling has been treated with antibiotics for infection in the past. What’s the most likely cause?

  1. Branchial cyst
  2. Parotitis
  3. Pharyngeal pouch
  4. Thyroglossal cyst
  5. Thyroid swelling
  6. 1. What is the key?
  7. 2. Justify your answer.

Ans. 1. The key is A. Branchial cyst.

Ans. 2. i) Branchial cyst is anterior triangular lump. [parotid is also anterior triangular lump but parotitis regresses with appropriate treat- ment i.e. becomes normal in size]. ii) pharyngeal pouch is posterior triangular lump. iii) Thyroglossal is midline lump. iv) thyroid swelling moves with swallowing].

 

59.    An 18yo man is rushed into the ER by his friends who left him immediately before they could be interviewed by staff. He is semi- conscious, RR=8/min, BP=120/70mmHg, pulse=60bpm. He is noted to have needle track marks on his arms and his pupils are small. What is the single best initial tx?

  1. Insulin
  2. Naloxone
  3. Methadone
  4. Gastric lavage
  5. 1. What is the key?
  6. 2. What is the diagnosis?
  7. 3. What are the points in favour of the diagnosis?

Ans. 1. The key is B. Naloxone.

Ans. 2. The diagnosis is opiate overdose.

Ans. 3. Points in favour are: i) reduced consciousness ii) RR 8/min (12<) iii) hypotension (here lower normal) iv) miosis v) needle track marks on his arms.

 

60.    A 30yo man and wife present to the reproductive endocrine clinic because of infertility. The man is tall, has bilateral gynecomastia. Examination of the testes reveals bilateral small, firm testes. Which of the following inv is most helpful in dx?

  1. CT of pituitary
  2. Chromosomal analysis
  3. Measure of serum gonadotropins
  4. Measure of serum testosterone
  5. 1. What is the key?
  6. 2. What is the diagnosis?
  7. 3. What are the points in favour of your diagnosis?

Ans. 1. The key is B. Chromosomal analysis.

Ans. 2. The diagnosis is Klinefelter’s syndrome. (xxy)

Ans. 3. The points in favour are: i) Infertility ii) Tall stature iii) Bilateral gynaecomastia iv) Bilateral small firm testes.

 

61.    An 18yo female just received her A-Level results and she didn’t get into the university of her choice. She was brought into the ED after ingestion of 24 paracetamol tablets. Exam: confused and tired. Initial management has been done. Inv after 24h: normal CBC, ABG = pH7.1, PT=17s, Bilirubin=4umol/L, creatinine=83umol/L. What is the next step in management?

  1. Observation for another 24h
  2. Refer to psychologist
  3. Give N-Acetylcysteine
  4. Discharge with psychiatry referral
  5. Liver transplantation
  6. 1. What is the key?
  7. 2. What are the indications of this management?

Ans. 1. The key is E. Liver transplantation.

Ans. 2. King’s College Hospital criteria for liver transplantation in paracetamol-induced acute liver failure.

arterial pH <7.3 or arterial lactate >3.0 mmol/L after adequate fluid resuscitation, OR if all three of the following occur in a 24-hour period: Creatinine >300 mol/L. PT >100 seconds (INR >6.5). Grade III/IV encephalopathy.

 

62.   A 75yo alcoholic presents with a mass up to umbilicus, urinary dribbling, incontinence, and clothes smelling of ammonia. What is the next step in management?

  1. Urethral catheter
  2. Suprapubic catheter
  3. Antibiotics
  4. Condom catheter
  5. Nephrostomy
  6. 1. What is the key?
  7. 2. What is the cause of this retention?

Ans. 1. The key is A. Urethral catheter.

Ans. 2. Alcohol consumption [Alcoholism can cause urinary retention but it is a less common cause of retention].

63.    In CRF, main cause of Vit D deficiency is the failure of:

  1. Vit D absorption in intestines
  2. 25 alpha hydroxylation of Vit D
  3. Excess Vit D loss in urine
  4. 1 alpha hydroxylation of Vit D
  5. Availability of Vit D precursors

Ans. The key is D. 1 alpha hydroxylation of Vit D. [There are 3 steps in the synthesis of vitamin D. i) Cholecalciferol in the skin from 7-dehy- drocholesterol under the action of ultraviolet light. ii) Hydroxylation in the liver of cholecalciferol to 25-hydroxycholecalciferol. iii) Hydroxyl- ation in the kidneys (1 alpha hydroxylation by enzyme 1-alphahydrox- ylase) of 25-hydroxycholecalciferol to calcitriol (1,25-dihydroxyvitamin D3)].

 

64.    Pt with puffiness of face and rash showing cotton wool spots on fundoscopy. What’s the dx?

  1. Macular degeneration
  2. Hypertensive retinopathy
  3. Diabetic background
  4. Proliferative diabetic retinopathy
  5. SLE
  6. 1. What is the key?
  7. 2. Why there is puffyness of face?
  8. 3. Why there is cotton wool spots on fundoscopy? What is the most common ocular manifestation of SLE?

Ans. 1. The key is SLE.

Ans. 2. Puffiness is due to lupus nephritis.

Ans. 3. SLE, can involve the retina. The classic lesion of SLE is a white fluffy appearing lesion within the retina known as a cotton wool spot. The most common ocular manifestation in SLE is Keratoconjunctivits sicca.

65.    A 35yo man presents with progressive breathlessness. He gave a hx of polyarthralgia with painful lesions on the shin. CXR: bilateral hilar lymphadenopathy. What’s the most likely dx?

  1. Bronchial asthma
  2. Cystic fibrosis
  3. Sarcoidosis
  4. Bronchiectasis
  5. Pneumonia
  6. 1. What is the key?
  7. 2. What is the specific name of this condition? What is the triad?

Ans. 1. The key is C. Sarcoidosis.

Ans. 2. Lofgren syndrome. The triad is i) Erythema nodosum ii) Bilat- eral hilar lymphadenopathy iii) Arthralgia.

 

 

 

66.    A child presents with clean wound, but he has never been immu- nized as his parents were worried about it. There is no contraindica- tion to immunization, what is the best management?

  1. Full course of DTP
  2. 1 single injection DT
  3. 1 single injection DTP
  4. Only Ig
  5. Antibiotic

Ans. The key is A. Full course of DTP.

 

67.    A 65yo HTN man presents with lower abdominal pain and back pain. An expansive abdominal mass is palpated lateral and superior to the umbilicus. What is the single most discriminating inv?

  1. Laparascopy
  2. KUB XR
  3. Pelvic US
  4. Rectal exam
  5. Abdominal US
  6. 1. What is the key?
  7. 2. What is the diagnosis?
  8. 3. What are the points given here in favour of your diagnosis?

Ans. 1. The key is E. Abdominal US.

Ans. 2. The diagnosis is Abdominal aortic aneurism.

Ans. 3. Points in favour of AAA are i) hypertension ii) abdominal pain

  • back pain iv) expansile abdominal mass lateral and superior to the

68.   A 55yo man has had severe pain in the right hypochondrium for 24h. The pain comes in waves and is accompanied by nausea. Nothing seems to relieve the pain. He feels hot and sweaty but has normal temp. What is the most appropriate next inv?

  1. US Abdomen
  2. ERCP
  3. MRCP
  4. Serum amylase
  5. UGI endoscopy
  6. 1. What is the key?
  7. 2. What is the diagnosis?
  8. 3. What are the points in favour of your diagnosis?

Ans. 1. The key is A. US abdomen.

Ans. 2. The diagnosis is biliary colic.

Ans. 3. Points in favour- i) severe right hypochondrial pain. ii) colicky nature of the pain (comes in waves) iii) nausea iv) absence of fever iv) absence of jaundice. [OHCM, 9th edition, page-637].

 

 

 

 

 

69.    A 67yo man has deteriorating vision in his left eye. He has longstanding COPD and is on multiple drug therapy. What single medication is likely to cause this visual deterioration?

  1. B2 agonist
  2. Corticosteroid
  3. Diuretic
  4. Theophylline
  5. 1. What is the key?
  6. 2. What is the cause of deteriorating vision?

Ans. 1. The key is B. Corticosteroid.

Ans. 2. Prolonged corticostiroids [also topical i.e. eye drop] can cause cataract.

 

70.    A woman who returned from abroad after 3 weeks of holiday complains of severe diarrhea of 3 weeks. She also developed IDA and folic acid def. What condition best describes her situation?

  1. Jejunal villous atrophy
  2. Chronic diarrhea secretions
  3. Malabsorption
  4. Increased catabolism
  5. Increased secretions of acid

 

  1. What is the key?

Ans. The key is C. Malabsorption.

 

71.     A 35yo male is bitterly annoyed with people around him. He thinks that people are putting ideas into his head. What is the single most likely dx?

  1. Thought block
  2. Thought insertion
  3. Thought broadcasting
  4. Thought withdrawal
  5. Reference
  6. 1. What is the key?
  7. 2. In which disease you will find this feature?

Ans. 1. The key is B. Thought insertion.

Ans. 2. It is seen in schizophrenia.

 

72.   A 10yo girl presents with hoarseness of the voice. She is a known case of bronchial asthma and has been on oral steroids for a while. What is the most likely cause of hoarseness?

  1. Laryngeal candidiasis
  2. Infective tonsillitis
  3. Laryngeal edema
  4. Allergic drug reaction
  5. Ludwigs angina
  6. 1. What is the key?
  7. 2. What is the reason for this condition?

Ans. 1. The key is A. Laryngeal candidiasis.

Ans. 2. Steroids predisposes to fungal infection.

73.    A lady with breast cancer has undergone axillary LN clearance. She develops arm swelling after being stung by a bee. What is the most likely mechanism responsible for the swelling?

  1. Lymphedema
  2. Cellulitis
  3. Hypersensitivity reaction
  4. DVT
  5. Fluid retention
  6. 1. What is the key?
  7. 2. What is the reason for this condition?

Ans. 1. The key is A. Lymphoedema.

Ans. 2. Reason is compromised lymphatic drainage of arm due to ax- illary LN clearance. So if there is any inflammation or swelling, lymph drainage is compromised further giving rise to swelling of limb of that

operated side.

 

74.    A 34yo pt presents with 50% partial thickness burns. What should be the most appropriate management?

  1. IV fluids calculated from the time of hospital arrival
  2. IV fluids calculated from the time of burn
  3. No IVF
  4. IV dextrose stat
  5. Burns ointment
  6. 1. What is the key?
  7. 2. How the calculation of fluid is made?

Ans. 1. The key is B. IV fluids calculated from the time of burn. Ans. 2. Resuscitation fluids required in the first 24 hours from the time of injury. For adults: 3 ml (in partial thickness burn) of Hart- mann’s solution/kg body weight/% total body surface area. Half of

this calculated volume is given in the first 8 hours and the other half is given over the following 16 hours.

 

75.     A 54yo man has recently been dx with moderate depression. He has hx of MI and is suffering from insomnia. What is the drug of choice for him?

  1. Citalopram
  2. Lofepramine
  3. ECT
  4. Haloperidol
  5. Diazepam

Ans. Key is A. Citalopram. [Sertraline is the drug of choice in post MI as citalopram may be a risk factor for precipitating torsades-de- pointes. But if sertraline is not in option second choice is citalopram and where SSRIs are not suitable next choice is mirtazapine].

 

76.    A man presented with cellulitis and swelling. He was started on flucloxacillin. What other medication do you want to add?

  1. Vancomycin
  2. Penicillin
  3. Metronidazole
  4. Ceftriaxone
  5. Amoxicillin
  6. 1. What is the key?
  7. 2. Please mention why?

Ans. 1. The key is B. Penicillin.

Ans. 2. cellulitis is usually caused by staphylococcus and streptococcus. To cover both Flucloxacillin (for staphylococcus) and

Penicillin (to cover streptococci) should be prescribed.

 

77.    A 24yo college student presents with nausea, vomiting, headache, neck stiffness and a fever of 38.4C. What is the most appropriate empirical antibiotic to be started?

  1. Ceftriaxone
  2. Penicillin
  3. Gentamicin
  4. Tazobactam
  5. Meropenem

Ans. The key is A. Ceftriaxone. [Ceftriaxone is the drug of choice in hospital management. Probable dx is meningitis. [In OHCM- Cefotaxime <55yrs and Cefotaxime + Ampicillin if age >55yrs (pre- hospital)].

 

78.    A man with prosthetic heart valve underwent hemicolectomy and after some days complains of left hypochondriac pain, fever and has a systolic murmur. What is the next inv to ascertain the cause of HF?

  1. CT
  2. Blood culture
  3. ECG
  4. MRI
  5. Radioactive thyroid scan
  6. 1. What is the key?
  7. 2. What is the diagnosis?
  8. 3. Why have you made this diagnosis?
  9. 4. What are the important risk factors for this condition?

Ans. 1. The key is B. Blood culture.

Ans. 2. The diagnosis is infective endocarditis.

Ans. 3. Fever + new murmur = endocarditis until proven otherwise. Ans. 4. Important risk factors: dermatitis, IV injections, renal failure, organ transplantation, DM, post operative wond. Risk factors for ab- normal valves: aortic or mitral valve disease, tricuspid valve in IV drug users, prosthetic valves.

 

79.     A 45yo man with posterior gastric ulcer presented with severe excruciating pain which subsided after conservative treatment. 10 days later he developed swinging pyrexia. US shows a collection in the peritoneum. What will be the most likely location of the collec- tion?

  1. Hepatorenal puch
  2. Left paracolic gutter
  3. Subphrenic
  4. Pelvic cavity
  5. Lesser sac

 

Ans. The key is E. Lesser sac.

 

80.    A 23yo lady was prescribed with azithromycin 1gm for her chla- mydial pelvic infection. She has got a new boyfriend for the last 2 months. She has recently started contraception to avoid conception. Which of the following contraception method will be affected by azithromycin?

  1. Barrier
  2. IUCD
  3. POP
  4. COCP

Ans. None of them! Before it was thought that hepatic enzyme inhibitor drugs may affect COCP but later it was established that actually there is no such significant effect. Only drugs like refumpicin, rifabutin etc. can cause this. [For exam purpose if you have to choose one please choose D. COCP].

 

81.     An 11yo boy is being checked by the diabetic specialist nurse. His HbA1c was high and he has been skipping meals recently. He has been unhappy at school. Which single member of the clinical team would you refer him to next?

  1. GP
  2. Pediatrician
  3. Dietician
  4. Clinical psychologist

Ans. The key is D. Clinical psychologist. [Skipping meals, unhappy at school these are psychological issues. So he should be referred to clinical psychologist].

 

82.   A 35yo man who has served in the army presents with lack of interest in enjoyable activities and feeling low. He doesn’t feel like reading the news or watching movies as he believes there is violence everywhere. What is the most appropriate first line therapy?

  1. Citalopram
  2. Lofepramine
  3. CBT
  4. Chlordiazepoxide
  5. Desensitization
  6. 1. What is the key?
  7. 2. What is the diagnosis?
  8. 3. What is the first line treatment?

Ans. 1. The key is C. CBT

Ans. 2. The diagnosis is mild depressive illness.

Ans. 3. In mild depressive illness CBT is preferred option.

83.    A man has reducible bulge below the pubic tubercle, and on occlusion of the deep inguinal ring, cough impulse is present. What is the most likely dx?

  1. Direct inguinal
  2. Indirect inguinal
  3. Femoral
  4. Spigelian
  5. Lumbar
  6. 1. What is the key?
  7. 2. What are the points in favour of your answer?

Ans. 1. The key is C. Femoral hernia.

Ans. 2. It is just below the pubic tubercle that is just below the inguinal ligament. On occlusion of deep ring cough impulse is present (means visible) as femoral hernia doesn’t come through deep ring but indirect inguinal hernia.

 

84.    A 48yo woman is admitted to ED with a productive cough and moderate fever. She has often central chest pain and regurgitation of undigested food most times but doesn’t suffer from acid reflux. These symptoms have been present for the last 3.5 months which affects both food and drink. A CXR shows an air-fluid level behind a normal sized heart. What is the most likely dx?

  1. Pharyngeal pouch
  2. Hiatus hernia
  3. Bulbar palsy
  4. Achalasia
  5. TB
  6. 1. What is the key?
  7. 2. What are the points in favour?

Ans. 1. The key is D. Achalasia.

Ans. 2. Points in favour: Aspiration pneumonia due to retained food and fluid in oesophagus. In achalasia usually there is no acid reflux. Dysphagia for both food and drink. Air-fluid level behind heart.

Why it is not hiatus hernia?

Ans. Differentiating point:-i) In hiatus hernia usually you will get associated GORD ii) Also in hiatus hernia there may be nausea or vomiting.

Why it is not pharyngeal pouch?

Ans. In pharyngeal pouch there will be halitosis.

85.    A 64yo man has been waking up in the middle of the night to go to the bathroom. He also had difficulty in initiating micturition

and complains of tthe pt was prepared for a TURP. What electrolyte abnormality is highly likely due to this surgery?

a.   Hypokalemia

  1. Hypocalcemia

c.  Hyperkalemia

  1. Hyponatremia

e.   Hypernatremia

  1. 1. What is the key?
  2. 2. Why this happens?

Ans. 1. The key is D. Hyponatremia.

Ans. 2. Absorption of fluid used for bladder irrigation to flush out blood clots and IV fluids all may lead to hypervolaemia and dilutional hyponatremia.

 

86.    A 56yo lady has developed severe right sided headache which worsens whenever she comes to bright light since the last 4 days. She feels nauseated, but doesn’t vomit. What is the most likely dx?

  1. SAH
  2. Brain tumor
  3. Migraine
  4. Cluster headache
  5. Subdural headache
  6. 1. What is the key?
  7. 2. What is the type of the given case?
  8. 3. What are the points in favour of mentioned type?

Ans. 1. The key is C. Migraine.

Ans. 2. It is migraine without aura.

Ans. 3. Criteria of migraine without aura: 5 headaches lasting 4-72 hours + nausea/vomiting (or photo/phono-phobia) + any 2 of: i) uni- lateral ii) pulsating iii) worsen by routine activity [OHCM, 9th edition, page-462].

87.    A 35yo man presented with hematuria, abdominal swelling and has a BP of 190/140. What is the most diagnostic inv?

  1. Cystoscopy
  2. USG
  3. CT
  4. Renal biopsy
  5. Urine analysis
  6. 1. What is the key?
  7. 2. What is the diagnosis?
  8. 3. What will be the USG findings to establish diagnosis in given case?

 

Ans. 1. The key is B. USG.

Ans. 2. The diagnosis is ADPKD.

Ans. 3. In given case patients age is 35. So the USG diagnostic criteria is: Age 18 – 39 yrs>3 unilateral or, bilateral cysts, 40 – 59 yrs >2 cysts in each kidney, >60 yrs >4 cysts in each kidney. [OHCM, 9th edition, page- 312].

 

 

88.    A young man is brought to the ED after an RTA. His GCS on initial evaluation is 6. What is the most appropriate next step?

  1. CT
  2. MRI
  3. IV fluids
  4. Skull XR
  5. Secure airway

Ans. The key is E. Secure airway.

89.    A 65yo man presented with frank hematuria. He has no other urinary symptoms. What is the most appropriate next step that will lead to the dx?

  1. IVU
  2. US Abdomen
  3. Cystoscopy
  4. Mid-stream urine for culture
  5. Transrectal US
  6. 1. What is the key?
  7. 2. What is the diagnosis?
  8. 3. What are the reasons for this diagnosis?
  9. 4. If there is painless haematuria in young (say 25-30yrs) what diagnosis will come first?

Ans. 1. Key is C. Cystoscopy.

Ans. 2. Bladder cancer.

Ans. 3. Age 65, asymptomatic haematuria.

Ans. 4. ADPKD [at the beginning there is very few or no symptoms].

90.    A 30yo woman had a gradual decrease of visual acuity since the last 3 years. Now she has a disability due to very low vision. What’s the dx?

  1. Glaucoma
  2. Cataract
  3. Macular degeneration
  4. Retinitis pigmentosa
  5. Keratitis
  6. 1. What is the key?
  7. 2. Why it is not the other given D/D s?

Ans. 1. The key is D. Retinitis pigmentosa.

Ans. 2. i) It is not angle closure glaucoma as angle closure glaucoma occurs usually after the age of 50; In open angle glaucoma visual loss is not evenly gradual rather occurs a bit suddenly at its later part. It is not cataract as cataract occurs usually in elderly. In macular degener- ation near blindness does not occur rather causes inability to identify face or cannot read small prints; otherwise peripheral vision is not that depressed. In keratitis will be pain, redness, photophobia and vision is ok.

 

 

91.       A 27yo lady has had an uncomplicated pregnancy so far. She came to the hospital 2h ago after her water broke. The midwife is looking at her now. She has regular contractions. P.V exam revealed 2cm dilated cervix. Vital signs are normal. What stage of labour is she in?

  1. Second stage
  2. First stage
  3. Latent stage
  4. Third stage
  5. Active phase

Ans. The key is B. First stage. First stage starts with softening of cer- vix with start of opening of cervix and ends when cevix is fully dilated (i.e. 10 cm dilated).

92.      A 2yo boy fell off his tricycle and hurt his arm. He got up to start crying, but before there was any sound, he went pale, unconscious and rigid. He recovered after 1-2 mins but remained pale. After an hour he was back to normal. His mother says she was afraid he was going to die, and that he had a similar episode 3 months prior after falling down some steps. What single inv is indicated?

  1. CT head
  2. EEG
  3. CBC
  4. None
  5. Skeletal survey
  6. 1. What is the key?
  7. 2. What is the diagnosis?

Ans. 1. The key is D. None.

Ans. 2. Diagnosis is breath holding spell.

93.       A 29yo woman had just delivered a still born vaginally, following a major placental abruption. Choose the single most likely predispos- ing factor for developing PPH in this lady?

  1. Retained product
  2. DIC
  3. Fibroid uterus
  4. Uterine infection
  5. Large placental site
  6. 1. What is the key?
  7. 2. What are the causes of this condition here?

Ans. 1. The key is B. DIC.

Ans. 2. Pregnancy itself is a risk factor for DIC. Placental abruption is a more common cause of DIC. Other causes of pregnancy related DIC are: eclampsia, retention of a dead fetus, amniotic fluid embolism, retained placenta or bacterial sepsis.

 

94.       A 28yo woman has delivered with rotational forceps after an 8h labor and 3h second stage. Choose the single most likely predispos- ing factor for PPH for this pt?

  1. Atonic uterus
  2. Cervical/vaginal trauma
  3. Retained product
  4. Preterm labor
  5. Uterine infection

Ans. The key is B. Cervical/vaginal trauma. [complication of forceps delivery].

 

95.       A 50yo man has had anterior resection of the rectum for carcinoma. He expressed concerns about control of post-op pain in discussions with the anaesthetist before surgery. What is the best management strategy?

  1. Oral diclofenac
  2. Oral codeine
  3. IM morphine
  4. IM dihydrocodeine
  5. Ondansetron oral

Ans. The key is C. IM morphine. [Some post operative pain is severe pain which needs strong opioid analgesics].

 

96.       A 73yo male presents with enlarged cervical nodes. He has had recurrent infections over the last year. His conjunctiva is pale. Choose the single cell type you will find on the blood film.

  1. Granulocyte without blast cells
  2. Myelofibroblasts
  3. Plasma cells
  4. Mature lymphocytes
  5. 1. What is the key?
  6. 2. What is the diagnosis?
  7. 3. What are the points in favour of your diagnosis?

Ans. 1. The key is D. Mature lymphocytes.

Ans. 2. The diagnosis is CLL.

Ans. 3. It is CLL because of his age (73 yrs), cervical lymphadenpathy, recurrent infections (mature but functionally defective lymphocytes), and pale conjunctiva (anaemia).

97.       A 45yo lady has 10m hx of SOB. She is found to have irregularly irregular pulse and loud P2 with fixed splitting and ejection systolic murmur in left 2nd ICS. What is the probable dx?

  1. TOF
  2. ASD
  3. VSD
  4. PDA
  5. CoA
  6. 1. What is the key?
  7. 2. What is the diagnosis?

Ans. 1. The key is B. Atrial septal defect.

Ans. 2. Diagnosis is ASD with atrial fibrillation. [i) atrial fibrillation

= irregularly irregular pulse. ii) ASD = SOB, fixed splitting with loud P2, ESM in pulmonary area]. This picture is typical. One should not misdiagnose SOB, ESM in pulmonary area and loud P2 as pulmonary hypertension (though in elderly this can develop with ASD).

98.      A 5m baby present with recurrent vomiting. Mother noticed some of the vomitus is blood stained. Choose the single most likely inv?

  1. Upper GI endoscopy
  2. Barium meal
  3. US
  4. Colonoscopy
  5. CT abdomen

Ans. The key is A. upper GI endoscopy.

99.       A 76yo is treated with HTN. He suffers from pain and redness at the MTP joint of his right big toe. Which of the following anti-HTN cause this symptoms?

  1. Losartan
  2. Bendroflumethiazide
  3. Ramipril
  4. Bisoprolol
  5. Verapamil
  6. 1. What is the key?
  7. 2. What is the diagnosis?
  8. 3. What is the cause of the disease?

Ans. 1. The key is B. Bendroflumethiazide

Ans. 2. Diagnosis is acute gout.

Ans. 3. Thiazide diuretics may cause hyperuricemia and thus cause or precipitate gout.

100.     A 33yo male involved in a street fight presents with bruises and deformity in the upper part of his leg. XR shows fx of the neck of fibula. What is the single most associated nerve injury?

  1. Sciatic nerve
  2. Gluteal nerve
  3. Musculocutaneous nerve
  4. Lateral peroneal nerve
  5. Tibial nerve
  6. Femoral nerve

Ans. The key is D. Lateral peroneal nerve. [Lateral peroneal nerve is other name of superficial peroneal nerve].

101.    A 35yo man presents with hx of dyspepsia. H.Pylori antibodies are negative. No improvement is seen after 1m of tx. What is the next step?

  1. Urea breath test
  2. Gastroscopy
  3. CT
  4. MRI
  5. 1. What is the key?
  6. 2. What may be the D/D here?
  7. 3. At this age what are the indications of this procedure?

Ans. 1. Gastroscopy.

Ans. 2. Not responding to treatment D/D is: i) Jollinger Elison syn- drome ii) Ca stomach

Ans. 3. Indications of gastroscopy in a 35 yo man (for man of age

<50): i) Acute symptoms with H/O previous episode (PUD) ii) Alarm features [weight loss, anaemia, vomiting, hematemesis and melaena, dysphagia, palpable abdominal mass], fear of cancer, evidence of organic disease.

102.   A 15yo male has bilateral ankle edema. His BP=110/70mmHg and urinalysis shows protein++++. What is the most likely dx?

  1. HUS
  2. IgA nephropathy
  3. Membranous GN
  4. Minimal change GN
  5. Nephrotic syndrome
  6. 1. What is the key?
  7. 2. What are the points in favour of your diagnosis?
  8. What is the treatment?

Ans. 1. The key is D. Minimal change disease.

Ans. 2. Points in favour: i) Age 15 ii) Ankle oedema iii) Normotension

  1. Heavy

Ans. 3. Treatment of choice is steroid (prednisolon). Failure of steroid or frequent relapse (>3) cyclophosphamide.

103.   A 28yo man has developed a red, raised rash on trunk after playing football. His PMH shows he had childhood asthma. The rash is becoming increasingly itchy. What is the most appropriate tx?

  1. Oral chlorpheneraime
  2. Oral amoxicillin
  3. IM adrenaline
  4. Nebulized salbutamol
  5. Histamine
  6. What is the key?
  7. 2. What is the diagnosis?

Ans. 1. The key is A. Oral chlorpheneramine.

Ans. 2. Diagnosis is Atopy (allergy).

 

 

104.   A 72yo man has been advised to have antibiotic prophylaxis for some years now before dental tx. He has never experienced chest pain. Three weeks ago, he noticed breathlessness on exertion and for one week he had orthopnea. His pulse is normal. What is the most probable dx?

  1. Aortic regurgitation
  2. Ischemic mitral regurgitation
  3. Mitral valve prolapse
  4. Pulmonary stenosis
  5. Mitral valve stenosis

Ans. The kay is E. Mitral valve stenosis. [Mitral stenosis is the most common valvular complication of rheumatic fever].

 

105.    A 37yo woman presents with fatigue. Exam: angular stomatitis, no koilonychea. Choose the single cell type you will find on the blood film.

  1. Macrocytes
  2. Microcytes
  3. Granulocytes wthout blast cells
  4. Blast cells
  5. 1. What is the key?
  6. 2. What is the cause here?
  7. 3. What are the points in favour of mentioned cause?

Ans. 1. The given key is A. Macrocytes.

Ans. 2. The cause here is VIT. B12 or folate deficiency.

Ans. 3. Points in favour of Vit. B12 or folate deficiency: i) fatigue (anae- mia) ii) angular stomatitis (can be seen in Vit. B12 or folate deficiency)

iii) absence of koilonychea is against IDA (Iron Deficiency Anaemia).

106.   A 4yo boy with a febrile convulsion lasting eight minutes has been given IV lorazepam to control them. What is the single most likely serious side effect?

  1. Amnesia
  2. Anaphylactic shock
  3. Apnea
  4. Bronchospasm
  5. Cardiac arrhythmia

Ans. The key is C. Apnoea. [Respiratory depression may occur follow- ing lorazepam administration].

107.   A 4wk girl has been dx of having breast milk jaundice. She is otherwise well. What is the single most appropriate management?

  1. Continue breastfeeding
  2. Exchange transfusion
  3. Increase fluid intake
  4. Phototherapy
  5. Stop breastfeeding
  6. 1. What is the key?
  7. 2. What is breast milk jaundice?
  8. 3. What type of hyperbilirubinemia occurs in breast milk jaundice?
  9. 4. What is the cause of this jaundice?

Ans. 1. The key is A. Continue breast feeding.

Ans. 2. If jaundice lasts past the first week of life in a breastfed baby who is otherwise healthy, the condition may be called “breast milk jaundice.”

Ans. 3. Unconjugated hyperbilirubinaemia.

Ans. 4. Cause of breast milk jaundice: factors in a mother’s milk that help a baby absorb bilirubin from the intestine.

108.   A 12yo girl when playing in the garden accidentally stepped on a hive and was bitten several times. She has numerous wheals on her body and complains of severe itching. What is the single most appropriate management?

  1. Oral antihistamine
  2. IV antihistamine
  3. IM adrenaline
  4. Oral ciprofloxacin
  5. Reassurance

Ans. The given key is C. IM adrenaline which is a wrong key. The correct answer is A. Oral antihistamine. [Intravenous antihistamines are used as an adjunct to epinephrine in the emergency treatment of anaphylaxis and angioedema. Parenteral antihistamine is not recom- mended in less severe allergic reaction (other than anaphylaxis)].

Followings are the indications of adrenaline in anaphylaxis: Horseness of voice

Wheeze

Shortness of breath Shock

Stridor

Swelling of the tongue and cheek Facial swelling

109.    A term baby born to a 30yo woman of blood group A-ve develops severe jaundice within the first 24h of birth. What is the most likely dx?

  1. Hereditary spherocytosis
  2. G6PD
  3. ABO incompatibility
  4. Rh incompatibility
  5. Physiological jaundice

Ans. The key is D. Rh incompatibility.

  1. A 4yo girl is found to have bounding pulse and continuous ma- chinery What is the most probable dx?

 

  1. TOF
  2. ASD
  3. VSD
  4. PDA
  5. CoA

Ans. The key is D. PDA. [Continuous mechinary murmur is well known feature of PDA].

111.     A 12yo child with episodes of sudden bluish discoloration and brief loss of consciousness. Exam: clubbing, central cyanosis, systolic thrill with systolic ejection murmur in 2nd left ICS. What is the most probable dx?

  1. TOF
  2. ASD
  3. VSD
  4. PDA
  5. CoA

Ans. The key A. TOF. [TOF usually does not become symptomatic at birth or early infancy and given features (central cyanosis and club- bing with murmur of right ventricular outflow obstruction i.e. ejection systolic murmur in 2nd left ICS) are well known features of TOF]. [Te- tralogy of fallot = 1. VSD + 2. Overriding of the aorta + Right ventricular outflow tract obstruction + Right ventricular hypertrophy].

112.   An 8yo child who is tall for his age and has a refractory error for which he wears glasses has presented with severe crushing chest pain. What is the most likely dx?

  1. Fragile X syndrome
  2. Prader-willi syndrome
  3. DiGeorge syndrome
  4. Marfans syndrome
  5. 1. What is the key?
  6. 2. What is the cause of this severe crushing chest pain?
  7. 3. What are the most common cardiac abnormalitis found in this disease?

Ans. 1. The key is D. Marfans syndrome.

Ans. 2. Cause of severe crushing chest pain may be aortic dissection. Ans. 3. Most common cardiac abnormalities in Marfans syndrome are: dilatation of the aorta and mitral regurgitation.

113.    A 4yo child presents with pain of spontaneous onset in his knee of 2 days duration. He has developed mild fever in the 2nd day. He can walk but has a limp. Exam: painful restriction in the right hip. What is the most probable dx?

  1. Osteosarcoma
  2. Septic arthritis
  3. TB arthritis
  4. Exostosis
  5. Osteomyelitis
  6. 1. What is the key?
  7. 2. What are the points in favour of your diagnosis?

Ans. Given key is E. Osteomyelitis which is a wrong key. The correct answer is B. Septic arthritis.

 

Ans. Points in favour of diagnosis: i) Pain in joints (knee and hip). In osteeomyelitis there is no joint pain but pain in other parts of bone like shaft. ii) Fever iii) Painful restricted movement of joint.

NB This controversial question was debated and ultimately settled as septic arthritis by old plabbers.

 

 

114.    A man with anterior resection and end to end anastomosis done complains of severe pain in the chest and abdominal distension. What is the most appropriate inv likely to review the cause this deterioration?

  1. XR abdomen
  2. Exploratory laparoscopy
  3. CT
  4. US
  5. Laparotomy

Ans. The key is E. Laparotomy. This is wrong key! Correct key is C. CT. [As many anastomotic leak is self limiting so Laparotomy can not be the answer! Definitive diagnosis is made by CT scan. Here is one link attached. http://teachmesurgery.com/perioperative/gastrointestinal/ anastomotic-leak/

 

115.     Pt with hx of alcoholism, ataxic gait, hallucinations and loss of memory. He is given acamprosate. What other drug can you give with this?

  1. Chlordiazepoxide
  2. Thiamine
  3. Diazepam
  4. Disulfiram
  5. Haloperido

Ans. The key is B. Thiamine. [These are features of wernicke’s enceph- alopathy].

116.    A 35yo male builder presented with sudden onset of severe abdominal pain. He was previously fit and well other than taking ibuprofen for a long term knee injury. On examination he is in severe pain, pulse=110bpm, BP=110/70mmHg and has a rigid abdomen. What is the most likely dx?

  1. Biliary peritonitis
  2. Ischemic colon
  3. Pancreatic necrosis
  4. Perforated diverticulum
  5. Perforated peptic ulcer

Ans. The key is E. Perforated peptic ulcer. [NSAIDs induced perfora- tion. Points in favour- 1. Prolonged use of NSAIDs, 2. Sudden onset of severe abdominal pain, 3. Rigid abdomen].

 

 

 

 

117.    A woman 5 days post-op for bilateral salphingo-oopherectomy and abdominal hysterectomy has developed abdominal pain and vomiting a/w abdominal distension and can’t pass gas. No bowel sounds heard, although well hydrated. What is the most appropriate next step?

  1. XR abdomen
  2. Exploratory laparoscopy
  3. CT
  4. USG
  5. Barium enema
  6. 1. What is the key?
  7. 2. What is the diagnosis?
  8. 3. What are the causes of it?
  9. 4. What is the management?

Ans. 1. The key is A. X-ray abdomen.

Ans. 2. The diagnosis is paralytic ileas.

Ans. 3. Causes of paralytic ileus: i) electrolyte imbalance ii) gastro- enteritis iii) appendicitis iv) pancreatitis v) surgical complications and

  1. vi) certain drugs.

Ans. 4. Management of paralytic ileus: i) nil by mouth ii) nasogastric suction to alleviate the distension and remove the obstruction.

118.    A 30yo man complains of hoarseness of voice. Exam: unilateral immobile vocal cord. What is the most probable dx?

  1. Graves disease
  2. Hematoma
  3. Unilateral recurrent laryngeal nerve injury
  4. External laryngeal nerve injury
  5. Tracheomalacia

Ans. The key is C. unilateral recurrent laryngeal nerve injury.

119.    A 38yo woman has delivered after an induced labor which lasted 26h. choose the single most likely predisposing factor for postpar- tum hemorrhage?

  1. Atonic uterus
  2. Cervical/vaginal trauma
  3. Rupture uterus
  4. Fibroid uterus
  5. Age of mother

Ans. The key is A. Atonic uterus. [Prolonged labour is a risk factor for PPH secondary to atonic uterus].

120.    A 32yo woman in tears describing constant irritability with her 2 small children and inability to relax. She describes herself as easily startled with poor sleep and disturbed nightmares following a house fire a year ago, while the family slept. What is the single best tx?

  1. Rassurance
  2. Relaxation therapy
  3. Quetiapine
  4. Lofepramine
  5. Fluoxetine
  6. 1. What is the key
  7. 2. What is the diagnosis?
  8. 3. What are the points in favour of your diagnosis?

Ans. 1 The key is E. Fluoxetine. [NICE guidelines suggest that trauma-focussed psychological therapies (CBT or EMDR) should be offered before medication, wherever possible. As these are not in option the best answer here is Fluoxetine (SSRI). At present, there is evidence that EMDR, psychotherapy, behaviour therapy and antide- pressants are all effective. There is not enough information for us to say that one of these treatments is better than another].

Ans. 2. The diagnosis is post traumatic stress disorder.

Ans. 3. Points in favour of PTSD: i) H/O stressor (house fire a year ago) ii) Nightmares of the stressor iii) Hyper arousal (very anxious and inability to relax (leading to irritability) iv) associated depression (poor sleep, tearful).

Note: Fluoxetin and peroxetin are the drugs of choice in PTSD. CBT is the non-pharmacological treatment.

 

121.     A 22yo woman with longstanding constipation has severe ano-rectal pain on defecation. Rectal exam: impossible due to pain and spasm. What is the most probable dx?

  1. Anal hematoma
  2. Anal fissure
  3. Anal abscess
  4. Protalgia fugax
  5. Hemorrhoids

Ans. The key is B. Anal fissure.

122.   A 20yo student attends the OPD with complaint of breathless- ness on and off, cough and sputum. His sleep is disturbed and skin is very dry in flexural areas of the body. Exam: tachypnea, hyperres- onant percussion and wheezing on auscultation. What is the most likely dx?

  1. Extrinsic allergic alveolitis
  2. Asthma
  3. Wegener’s granulomatosis
  4. COPD
  5. Cystic fibrosis
  6. What is the key?
  7. What are the diagnostic criteria?

Ans. The key is B. Asthma.

Ans. 2. Diagnostic criteria of asthma: i) Airway hyper-responsiveness to certain stimuli ii) Recurrent variable airflow limitation usually reversible iii) presents as wheezing, breathlessness, chest tightness and cough.

123.    A pt with thought disorder washes hands 6x each time he uses the toilet. What is the best management?

  1. Psychodynamic therapy
  2. CBT
  3. Antipsychotics
  4. Refer to dermatology
  5. Reassure
  6. 1. What is the key?
  7. 2. What is the diagnosis?

Ans. 1. The key is B. CBT. [For OCD CBT is the 1st treatment of choice and if fails comes drugs].

Ans. 2. The diagnosis is obsessive compulsive disorder.

124.    A 25yo woman presented to her GP on a routine check up. Upon vaginal exam, she was fine except for finding of cervical ectropion which was painless but mild contact bleeding on touch. What is the next management?

  1. Endometrial ablation
  2. Cervical smear (2nd line)
  3. Colposcopy
  4. Antibiotics (1st line)
  5. Vaginal US
  6. Pack with gauze and leave to dry
  7. 1. What is the key?
  8. 2. Points in favour correct of key.

Ans. The key is D. Antibiotics. This is a wrong key. The correct key is B. Cervical smear. In UK 25 yr is the age when 1st smear is offered. [Be- fore antibiotics we have to take swab. As there is no option of swab and age is 25 yrs we shall go for smear].

Ans. 2. Cervical smear is a screaning service which follows its own schedule and can not be done unless it is due or overdue. As the Lady is of 25 years and has ectropion and contact bleeding smear is the best option here as in UK 1st smear is offered at 25 yrs.

125.    A 32yo had a normal vaginal delivery 10 days ago. Her uterus has involuted normally. Choose the single most likely predisposing factor for PPH?

  1. Retained product
  2. DIC
  3. Uterine infection
  4. Von Willebrand disease
  5. Primary PPH
  6. 1. What is the key?
  7. 2. What type of PPH it would be?

Ans. 1. The key is C. uterine infection.

Ans. 2. Secondary PPH

126.    A 37yo man slipped while he was walking home and fell on his out stretched hand. He complains of pain in the right arm. XR

showed fx of the head of radius. What is the single most associated nerve injury?

  1. Radial nerve
  2. Musculocutaneous nerve
  3. Median nerve
  4. Ulnar nerve
  5. 1. What is the key?
  6. 2. What is the root value?

Ans. 1. The key is A. Radial nerve. [At wrist, radial nerve injury cause

finger drop].

Ans. 2. Root value of radial nerve: C5,6,7,8 and T1.

 

 

127.    A butcher stabbed accidently his groin. He bled so much that the towel was soaked in blood and BP=80/50mmHg, pulse=130bpm. What % of circulatory blood did he lose?

  1. <15%
  2. 15-30%
  3. 30-40%
  4. 40-50%
  5. >50%
  6. 1. What is the key?
  7. 2. What is the classification of blood loss according to vita sign?

Ans. 1. The key is C. 30-40%

Ans. 2. Classification:

Class 1 up to 15% of blood volume lost: pulse <100; systolic BP normal; pulse pressure normal; Respiratory rate 14-20; urine output greater than 30 ml/hour.

Class 2 15%-30% blood volume lost: pulse 100-120; systolic blood pressure normal; pulse pressure decreased; respiratory rate 20-30; urine output 20-30 ml/hour.

Class 3 30%-40% blood volume lost: pulse 120-140; systolic BP decreased; pulse pressure decreased, respiratory rate 30-40; urine output 5-15 ml/hr

Class 4, blood loss of greater than 40%: pulse rate >140; systolic BP decreased; pulse pressure decreased’ respiratory rate >35; urine output negligible.

128.    A 67yo man presents with palpitations. ECG shows an irregular rhythm and HR=140bpm. He is otherwise stable, BP=124/80mmHg. What is the most appropriate management?

  1. Bisoprolol
  2. ACEi
  3. Ramipril
  4. Digoxin

Ans. The key is A. Bisoprolol. [In acute AF (<48h) if stable rate control by verapamil 40-120mg/8hourly po or bisoprolol 2.5-5mg/d po. In chronic AF (>48h) rate control with beta-blocker or rate limiting CCB; OHCM, 9th edition, page-124].

 

129.     A 78yo man is depressed after his wife’s death. He has been neglecting himself. His son found him in a miserable state when he went to visit. The son can’t deal with his father. What is the appropri- ate management?

  1. Voluntary admission to psychiatry ward
  2. Hand over to social worker
  3. Request son to move in with father
  4. Send pt to care home

Ans. The key is A. Voluntary admission to psychiatry ward.

 

 

 

130.    An old alcoholic presents with cough, fever, bilateral cavitating consolidation. What is the most probable cause?

  1. Gram +ve diplococcic
  2. Coagulase +ve cocci
  3. Gram –ve cocci
  4. AFB
  5. Coagulase –ve cocci
  6. 1. What is the key?
  7. 2. What is the organism?

Ans. 1. The key is B. Coagulase +ve cocci. [The picture is of pneumonia and bilateral cavitating consolidation favours staphylococcus as the causative agent].

Ans. 2. Name of organism is Staphylococcus aureus. [Staphylococci can be both coagulase positive and coagulase negative. Gram positive diplococcic is pneumococcus and gram negative nisseria, moraxella catarrhalis and hemophilus influenza. For AFB there should be low grade evening fever with night sweats, weight loss, anorexia etc].

131.     A 67yo man had successful thrombolysis for an inf MI 1 month ago and was discharged after 5days. He is now readmitted with pulmonary edema. What is the most probable dx?

  1. Aortic regurgitation
  2. Ischemic mitral regurgitation
  3. Mitral valve prolapse
  4. Pulmonary stenosis
  5. Rheumatic mitral valve stenosis

Ans. The key is B. Ischaemic mitral regurgitation. [ischaemic mitral regurgitation > raised pulmonary capillary pressure > pulmonary oe- dema]. [ Inferior myocardial infarction causes left ventricular remod- eling, which displaces posterior papillary muscle away from its normal position, leading to ischemic mitral regurgitation].

132.   A 60yo lady who had stroke 3 years ago now reports having in- creased dyspnea on exertion and atrial fibrillation. CXR: straight left border on the cardiac silhouette. What is the most probable dx?

  1. Aortic regurgitation
  2. Ischemic mitral regurgitation
  3. Mitral valve prolapse
  4. Pulmonary stenosis
  5. Rheumatic mitral valve stenosis
  6. 1. What is the key?
  7. 2. What are the points in favour of your answer?

Ans. 1. The key is E. Rheumatic mitral valve stenosis.

Ans. 2. Points in favour: i) Dyspnoea on exertion ii) Straight left border of the cardiac silhouette. To straighten the left heart border it requires underfilling of the left ventricle and aorta which occurs in mitral stenosis. Iii) Atrial fibrillation is a common association.

 

 

 

 

 

133.    A 60yo diabetic complains of pain in thigh and gluteal region on walking up the stairs for the last 6 months. She is a heavy smoker and has ischemic heart disease. What is the most appropriate dx?

  1. Thromboangitis Obliterans
  2. Sciatica
  3. DVT
  4. Atherosclerosis
  5. Embolus
  6. 1. What is the key?
  7. 2. What are the points in favour?

Ans. 1. The key is D. Atherosclerosis.

Ans. 2. i) It is not sciatica as sciatica pain is worse when sitting. There may be weakness, numbness, difficulty moving the leg or foot. A constant pain on one side of the rear. A shooting pain that makes it difficult to stand up. ii) It is not DVT as no swelling, warmth or redness of skin are there iii) It is not thromboangitis obliterans as pulses are ok, no colour change or reduced hair growth, no ulceration or gangrene

  1. iv) not embolism as no pain (rest pain), no numbness, no redness or itching or rash, no ulceration of

134.    A 3yo child who looks wasted on examination has a hx of diar- rhea on and off. The mother describes the stool as bulky, frothy and difficult to flush. What is the single inv most likely to lead to dx?

  1. Sweat chloride test
  2. Anti-endomysial antibodies
  3. LFT
  4. US abdomen
  5. TFT
  6. 1. What is the key?
  7. 2. What is the diagnosis?

Ans. 1. The key is B. Anti-endomysial antibody

Ans. 2. The diagnosis is celiac disease. [It is not cystic fibrosis as lung problem is most commonly seen in cystic fibrosis along with GI problem like indigestion].

135.     A 45yo woman has had severe epigastric and right hypochondri- al pain for a few hours. She has a normal CBC, serum ALP is raised, normal transaminase. 3 months ago she had a cholecystectomy done. What is the most appropriate inv?

  1. US abdomen
  2. ERCP
  3. MRCP
  4. CT abdomen
  5. Upper GI endoscopy
  6. 1. What is the key?
  7. 2. What is the diagnosis?

Ans. 1. The key is B. ERCP. It is probably a wrong key and correct key should be C. MRCP. [Post operative US of abdomen does not give good result for hepatobiliary system. ERCP is invasive procedure and it has its considerable complications like cholangitis, injury, pancreati- tis etc. Among given options MRCP is most appropriate. We shall go for ERCP after making the dx confirm. For this MRCP is preferred. If the question asks which is the “DEFINITIVE” or the “MOST DIAG- NOSTIC” then the option will be ERCP].

Ans. 2. Diagnosis is choledocolithiasis.

136.    A 53yo woman presented with pain in the eye, blurry vision and clumsiness for 3 months. She has a hx of difficulty in swallowing and weakness in her right upper limb 2y ago. What is the inv of choice?

  1. CSF analysis
  2. EEG
  3. EMG
  4. MRI brain
  5. Visual evoked response test
  6. What is the key?

 

  1. What is the diagnosis?

Ans. 1. The key is D. MRI brain.

Ans. 2. Diagnosis is multiple sclerosis.

137.     A 55yo male presents with malaise and tiredness. Exam: spleen approaching RIF, no lymphadenopathy. Choose the single cell type?

  1. Helmet shaped cell
  2. Sickle cell
  3. Granulocyte without blast cells
  4. Blast cells
  5. 1. What is the key?
  6. 2. What is the diagnosis?
  7. 3. What are the diagnostic features?

Ans. 1. The key is C. Granulocyte without blast cells.

Ans. 2. The diagnosis is CML.

Ans. 3. Diagnostic features are i) increased number of mature granu- locytes ii) huge splenomegaly.

[* Helmet shaped cells (or shistocytes) = hemolytic or Microangiopath- ic hemolytic anemia

  • Sickle cell = sickle cell anemia
  • Blast cells (immature cells) = in acute leukemia].

138.    A 6yo pt comes with easy bruising in different places when she falls. CBC: WBC=25, Hgb=10.9, Plt=45. Her paul bunnel test +ve. What is the most likely dx?

  1. Glandular fever
  2. ITP
  3. Trauma
  4. NAI
  5. Septicemia
  6. 1. What is the key?
  7. 2. What are the lab. Values that suggests the diagnosis here?

Ans. 1. The key is A. Glandular fever.

Ans. 2. Suggestive lab. Values: WBC=25 (leucocytosis), Hgb=10.9 (usually patient is not anaemic), Plt=45 (thrombocytopenia-leading to easy bruising), Positive paul bunnel test.

139.     A 41yo woman who has completed her family, has suffered from extremely heavy periods for many years. No medical tx has worked. She admits that she would rather avoid open surgery. After discus- sion, you collectively decide on a procedure that wouldn’t require open surgery or GA. Select the most appropriate management for this case.

  1. Endometrial ablation
  2. Hysterectomy
  3. Fibroid resection
  4. Myomectomy
  5. Uterine artery embolization

Ans. The key is E. Uterine artery embolization. [Done by interven- tional radiologist expert in arterial embolization technique. Particles are placed in uterine artery to block circulation to uterine body. No operation or GA is required].

140.    A girl with hx of allergies visited a friend’s farm. She got stridor, wheeze and erythematous rash. What is the most appropriate tx?

  1. 25ml IM adrenaline
  2. 25ml PO adrenaline
  3. 25ml IV adrenaline
  4. IV chlorphearamine

Ans. The key is A. 0.25 ml IM adrenaline [Presence of stridor and wheeze are suggestive of anaphilaxis and treatment option is IM adrenaline].

141.     A 5yo boy is referred to the hospital and seen with his father who is worried that he has been listless. He is not sure why his GP suggested he should come to the ED and is keen to get some tablets and go home. Exam: tired and irritable, swelling around eyes. Renal biopsy: remarkable for podocyte fusion on EM. What is the most probable dx?

  1. NAI
  2. Myelodysplastic disease
  3. HSP
  4. Membranous GN
  5. Minimal change GN

Ans. The key is E. Minimal change glomerulonephritis. [Podocyte fusion on electron microscopy].

142.   A 6yo boy is brought to the hospital for a 3rd episode of sore throat in 1 month. He is found bleeding from gums and nose and has pale conjunctiva. What’s the single cell type?

  1. Clumped platelets
  2. Microcytes
  3. Granulocyte without blast cells
  4. Blast cells
  5. Mature lymphocytes
  6. 1. What is the key?
  7. 2. What is the diagnosis?
  8. 3. What are the points that favour diagnosis?

Ans. 1. The key is D. Blast cells.

Ans. 2. The diagnosis is ALL

Ans. 3. Points in favour: i) Age-6yrs ii) recurrent infection (sorethroat) due to neutrpenia and abnormal lymphoblasts which cannot protect from infection iii) thrombocytopenia causing gum and nose bleeding. Iii) anaemia (pale conjunctiva) due to reduced red cell production from marrow occupation by blast cells. [Here debate came why it is not aplastic anaemia? There is no risk factor mentioned for this patient for aplastic anaemia. There may be congenital aplastic anaemia but again it would present earlier in life. So it goes more with leukaemia but it cannot be confirmed unless we do bone marrow aspiration.]

143.    A 23yo man has been stabbed in the back and has SOB. The tra- chea is not deviated, he has engorged neck veins and absent breath sounds on the right. What is the most appropriate dx?

  1. Tension pneumothorax
  2. Cardiac tamponade
  3. Simple pneumothorax
  4. Hemothorax
  5. Pleural effusion
  6. 1. What is the key?
  7. 2. What are the point in favour of your answer?

Ans. 1. The key is A. Tension pneumothorax.

Ans. 2. Points in favour: i) Stab wound in the back ii) SOB iii) En- gorged neck vein iv) Absent breath sound.

These features are common for both hemothorax and tension pneu- mothorax and tracheal deviation is common to both! This is probabil- ity of bad recall!!

 

144.    A 44yo pt comes with right hemiparesis. Exam: left sided ptosis and left dilated pupil. Where is the lesion?

  1. Cerebral infarct
  2. Cerebellar infarct
  3. Medulla oblongata
  4. Pons
  5. Midbrain
  6. 1. What is the key?
  7. 2. What is the name of this condition?

Ans. 1. The key is E. Midbrain.

Ans. 2. Weber syndrome [presence of ipsilateral oculomotor nerve palsy and contralateral hemiparesis or hemiplagia].

145.    A 50yo man has a stab wound to his left anterior chest at the level of the 4th ICS. He has a BP 80mmHg, pulse=130bpm. His neck veins are dilated and his heart sounds are faint. His trachea is central. What is the most appropriate dx?

  1. Cardiac tamponade
  2. Diaphragmatic rupture
  3. Fractured ribs
  4. Tension pneumothorax
  5. Traumatic rupture of aorta
  6. 1. What is the Key?
  7. What are the points in favour of your answer?

Ans. 1. Theakey is Cardiac tempoade.

Ans. 2. Points in favour: i) Systolic BP 80 mmHg ii) Pulse 130 bpm iii) Engorged neck vein iv) Faint heart sounds v) Trachea is central.

146.     A 15yo boy has a soft painless swelling in the left scrotum, blue in color and can be compressed. What is the most appropriate next step?

  1. Analgesia
  2. Antibiotic
  3. Biopsy
  4. Immediate surgery
  5. Reassurance
  6. 1. What is the key?

Q.2. What is the name of this condition?

Ans. 1. The key is E. Reassurance.

Ans. 2. Name of the condition is Varicocele.

147.    A 12yo pt presents with copious diarrhea. Exam: urine out- put=low, mucous membrane=dry, skin turgor=low. What is the most appropriate initial management?

  1. Antibiotic
  2. Antimotility
  3. Anti-emetic
  4. Fluid replacement
  5. Reassurance
  6. 1. What is the key?
  7. 2. What is the diagnosis and why?

Ans. 1. The key is D. Fluid replacement.

Ans. 2. Diagnosis is severe dehydration. Points in favour: i) low urine output ii) dry mucous membrane and iii) low skin turgor.

148.    A 60yo smoker presents with cramp-like pain in the calves relived by rest and non-healing ulcers. Exam: cold extremities with lack of hair around the ankles, absent distal pulses. What is the most probable dx?

  1. Intermittent claudication
  2. Chronic ischemia of the limbs
  3. Buerger’s disease
  4. DVT
  5. DM
  6. 1. What is the key?
  7. 2. Points that support your diagnosis.

Ans. 1. The key is B. Chronic ischaemia of the limb.

Ans. 2. Intermittent claudication is a symptom not diagnosis. It is not buerger’s disease as buerger occur in more younger heavy smoker (before the age of 50yrs), It is not DVT as dvt pain or tenderness is not of an intermittent claudication pattern. Again in DM there is no intermittent claudication.

149.    An otherwise healthy 13yo boy presents with recurrent episodes of facial and tongue swelling and abdominal pain. His father has had similar episodes. What is the most likely dx?

  1. C1 esterase deficiency
  2. HIV
  3. Mumps
  4. Sarcoidosis
  5. Sjogren’s syndrome
  6. 1. What is the key?
  7. 2. What is the name of this condition?
  8. 3. Why it is not acquired?

Ans. 1. The key is A. C1 esterase inhibitor deficiency.

Ans. 2. Hereditary angioedema.

Ans. 3. Acquired angeoedema usually manifest after the age of 40 yrs.

150.    A 25yo had an LSCS 24h ago for fetal distress. She now com- plains of intermittent vaginal bleeding. Observations: O2 sat=98% in air, BP=124/82mmHg, pulse=84bpm, temp=37.8C. The midwife tells you that she had a retained placenta, which required manual removal in the OT. Choose the most appropriate C-Section complication in this case?

  1. Retained POC
  2. Aspiration pneumonitis
  3. Endometritis
  4. Uterine rupture
  5. DIC
  6. 1. What is the key?
  7. 2. What are the points in favour?

Ans. 1. The key is C. Endometritis.

Ans. 2. More handling of tissue like manual removal of placenta, intermittent vaginal bleeding and raised temperature points toward infective process like endometritis.

151.     A 30yo woman has brief episodes of severe shooting pain in the rectum. Rectal examination and flexible sigmoidoscopy are normal. What is the most probable dx?

  1. Anal hematoma
  2. Anal fissure
  3. Rectal carcinoma
  4. Proctalgia fugax
  5. Piles

Ans. is D. Proctagia fugax [normal rectal examination and flexible sigmoidoscopy excludes other options].

152.   A 78yo male, DM and HTN, had a fall and since then is unable to walk. He presents with deformity and tenderness over the right hip area. XR=fx of femur neck. What is the single most associated nerve injury?

  1. Sciatic nerve
  2. Gluteal nerve
  3. Lateral peroneal nerve
  4. Tibial nerve
  5. Femoral nerve

Ans. The key is A. Sciatic nerve. [Sciatic nerve injury though may occur but may not be very common!]

 

153.    A 20yo man has a head on collision in a car. On presentation he is breathless, has chest pain and fx of 5-7th rib. CXR confirms this. What is the most appropriate initial action in this pt?

  1. Antibiotics
  2. Analgesia
  3. O2 by mask
  4. Physiotherapy
  5. Refer to surgeon

Ans. The key is C. O2 by mask. [There was debate in this forum that pain relief should be given first which will automatically relieve breath- ing problem. But others told O2 first]. O2 first is the correct answer!

 

 

[http://www.urmc.rochester.edu/encyclopedia/content.aspx?content- typeid=22&contentid=flailchest]

154.    A 28yo man with complains of headache and nose bleeds also has pain in the lower limbs on exertion. Exam: radio-femoral delay, cold legs with weak pulse and mild systolic murmur with normal S1S2. What is the most probable dx?

  1. TOF
  2. ASD
  3. VSD
  4. PDA
  5. CoA

Ans. The key is coarctation of aorta. [headache and nosebleeds – >hy- pertension, pain in lower limb on exertion -> as reduced blood supply to leg due to coarctation, radio-femoral delay, cold legs with week pulse, mid-systolic murmur are all features of coarctation of aorta].

155.     A 23yo male has a tonic clonic seizure whilst at college. His GCS is 12, BP=120/77mmHg, HR=99bpm. What is the most appropriate inv for his condition?

  1. CT
  2. MRI
  3. Serum blood glucose
  4. Serum drug levels

Ans. The key is C. Serum blood glucose [it is also possible that he may have taken drug, even though first we have to do serum glucose as low blood glucose can be very easily managed and it needs urgent management to save life. If it is excluded then we can look for other

 

causes which may be not fatal in short time as hypoglycaemia].

156.    A 20yo man complains of recent onset of itching which followed a viral infection. There are numerous wheals of all sizes on his skin particularly after he has scratched it. These can last up to an hour. What is the most probable dx?

  1. Uremia
  2. Urticaria
  3. Psychogenic itching
  4. Atopic eczema
  5. Primary biliary cirrhosis

Ans. The key is B. Urticaria.

157.    A 75yo lady who had mitral valve replacement 13 yrs ago has developed recurrent breathlessness. Her husband has noticed prom- inent pulsation in her neck. She complains of abdominal pain and ankle swelling. What is the most probable dx?

  1. Aortic regurgitation
  2. Mitral regurgitation
  3. Mitral stenosis
  4. Tricuspid regurgitation
  5. Pulmonary stenosis

Ans. The key is D. Tricuspid regurgitation. [Points in favour: i) recur- rent breathlessness – if the cause is LV dysfunction, ii) prominent pulsation in the neck – giant v waves, iii) abdominal pain – pain in liver on exertion, ankle swelling; These are features of tricuspid regurgita- tion. Reference:- OHCM, 9th edition, page- 142].

158.   A 45yo T1DM had an annual check up. Ophthalmoscopy showed dot and blot hemorrhage + hard exudate and multiple cotton wool spots. What is the next step in management?

  1. Reassurance and annual screening only
  2. Urgent referral to ophthalmologist
  3. Laser therapy
  4. Non-urgent referral to ophthalmologist
  5. Nothing can be done

Ans. The key is D. Non-urgent referral to ophthalmologist. [It is pre-proliferative retinopathy so non-urgent referral; If proliferative (with neovascularization) urgent referral].

159.    A 2m baby who has ambiguous genitalia presents to the ED with vomiting. Labs: Na+=125mmol/L, K+=6mmol/L. What is the most likely dx?

  1. Fragile X syndrome
  2. Turners syndrome
  3. Noonan syndrome
  4. Congenital adrenal hyperplasia

 

  1. 1. What is the key?
  2. 2. What are the points in favour?

Ans. 1. The key is D. Congenital adrenal hyperplasia

Ans. 2. Points in favour: i) ambiguous genitalia ii) salt wasting mani- fested as hyponatremia and hyperkalemia (In mild forms of salt-wast- ing adrenal hyperplasia, salt wasting may not become apparent until an illness stresses the child). [here hyperkalaemia inspite of vomiting is indicating the disease].

160.    A 40yo man collapsed at home and died. The GPs report says he suffered from T2DM and BMI=35. What is the most likely cause of death?

  1. MI
  2. DM
  3. HF
  4. PE
  5. Renal failure
  6. 1. What is the key?
  7. 2. Why the patient’s death was unnoticed?

Ans. 1. The key is A. MI.

Ans. 2. In diabetics MI become painless when the patient develop autonomic neuropathy (till there is no autonomic neuropathy diabetic patients will feel MI pain). In this case the disease was unnoticed as it was a painless attack.

161.     A 38yo pt presented with tingling, numbness, paraesthesia, resp stridor and involuntary spasm of the upper extremities. She has undergone surgery for thyroid carcinoma a week ago. What is the most likely dx?

  1. Thyroid storm
  2. Hyperparathyroidism
  3. Unilateral recurrent laryngeal nerve injury
  4. External laryngeal nerve injury
  5. Hypocalcemia
  6. 1. What is the key?
  7. 2. What is the cause of this condition?
  8. 3. Why there is respiratory stridor?

Ans. 1. The key is E. Hypocalcaemia.

Ans. 2. Hypocalcaemia may be due to accidental parathyroid gland removal during thyroidectomy.

Ans. 3. Laryngospasm is a feature in hypocalcaemia which may cause stridor.

55.      A 20yo man has a head on collision in a car. On presentation he is breathless, has chest pain and fx of 5-7th rib. CXR confirms this. What is the most appropriate initial action in this pt?

  1. Antibiotics
  2. Analgesia
  3. O2 by mask
  4. Physiotherapy
  5. Refer to surgeon

Ans. The key is C. O2 by mask. [There was debate in this forum that pain relief should be given first which will automatically relieve breath- ing problem. But others told O2 first]. O2 first is the correct answer!

 

 

[http://www.urmc.rochester.edu/encyclopedia/content.aspx?content- typeid=22&contentid=flailchest]

56.      A 28yo man with complains of headache and nose bleeds also has pain in the lower limbs on exertion. Exam: radio-femoral delay, cold legs with weak pulse and mild systolic murmur with normal S1S2. What is the most probable dx?

  1. TOF
  2. ASD
  3. VSD
  4. PDA
  5. CoA

Ans. The key is coarctation of aorta. [headache and nosebleeds – >hy- pertension, pain in lower limb on exertion -> as reduced blood supply to leg due to coarctation, radio-femoral delay, cold legs with week pulse, mid-systolic murmur are all features of coarctation of aorta].

57.       A 23yo male has a tonic clonic seizure whilst at college. His GCS is 12, BP=120/77mmHg, HR=99bpm. What is the most appropriate inv for his condition?

  1. CT
  2. MRI
  3. Serum blood glucose
  4. Serum drug levels

Ans. The key is C. Serum blood glucose [it is also possible that he may have taken drug, even though first we have to do serum glucose as low blood glucose can be very easily managed and it needs urgent management to save life. If it is excluded then we can look for other

causes which may be not fatal in short time as hypoglycaemia].

58.      A 20yo man complains of recent onset of itching which followed a viral infection. There are numerous wheals of all sizes on his skin particularly after he has scratched it. These can last up to an hour. What is the most probable dx?

  1. Uremia
  2. Urticaria
  3. Psychogenic itching
  4. Atopic eczema
  5. Primary biliary cirrhosis

Ans. The key is B. Urticaria.

59.      A 75yo lady who had mitral valve replacement 13 yrs ago has developed recurrent breathlessness. Her husband has noticed prom- inent pulsation in her neck. She complains of abdominal pain and ankle swelling. What is the most probable dx?

  1. Aortic regurgitation
  2. Mitral regurgitation
  3. Mitral stenosis
  4. Tricuspid regurgitation
  5. Pulmonary stenosis

Ans. The key is D. Tricuspid regurgitation. [Points in favour: i) recur- rent breathlessness – if the cause is LV dysfunction, ii) prominent pulsation in the neck – giant v waves, iii) abdominal pain – pain in liver on exertion, ankle swelling; These are features of tricuspid regurgita- tion. Reference:- OHCM, 9th edition, page- 142].

60.     A 45yo T1DM had an annual check up. Ophthalmoscopy showed dot and blot hemorrhage + hard exudate and multiple cotton wool spots. What is the next step in management?

  1. Reassurance and annual screening only
  2. Urgent referral to ophthalmologist
  3. Laser therapy
  4. Non-urgent referral to ophthalmologist
  5. Nothing can be done

Ans. The key is D. Non-urgent referral to ophthalmologist. [It is pre-proliferative retinopathy so non-urgent referral; If proliferative (with neovascularization) urgent referral].

61.      A 2m baby who has ambiguous genitalia presents to the ED with vomiting. Labs: Na+=125mmol/L, K+=6mmol/L. What is the most likely dx?

  1. Fragile X syndrome
  2. Turners syndrome
  3. Noonan syndrome
  4. Congenital adrenal hyperplasia
  1. 1. What is the key?
  2. 2. What are the points in favour?

Ans. 1. The key is D. Congenital adrenal hyperplasia

Ans. 2. Points in favour: i) ambiguous genitalia ii) salt wasting mani- fested as hyponatremia and hyperkalemia (In mild forms of salt-wast- ing adrenal hyperplasia, salt wasting may not become apparent until an illness stresses the child). [here hyperkalaemia inspite of vomiting is indicating the disease].

62.      A 40yo man collapsed at home and died. The GPs report says he suffered from T2DM and BMI=35. What is the most likely cause of death?

  1. MI
  2. DM
  3. HF
  4. PE
  5. Renal failure
  6. 1. What is the key?
  7. 2. Why the patient’s death was unnoticed?

Ans. 1. The key is A. MI.

Ans. 2. In diabetics MI become painless when the patient develop autonomic neuropathy (till there is no autonomic neuropathy diabetic patients will feel MI pain). In this case the disease was unnoticed as it was a painless attack.

63.       A 38yo pt presented with tingling, numbness, paraesthesia, resp stridor and involuntary spasm of the upper extremities. She has undergone surgery for thyroid carcinoma a week ago. What is the most likely dx?

  1. Thyroid storm
  2. Hyperparathyroidism
  3. Unilateral recurrent laryngeal nerve injury
  4. External laryngeal nerve injury
  5. Hypocalcemia
  6. 1. What is the key?
  7. 2. What is the cause of this condition?
  8. 3. Why there is respiratory stridor?

Ans. 1. The key is E. Hypocalcaemia.

Ans. 2. Hypocalcaemia may be due to accidental parathyroid gland removal during thyroidectomy.

Ans. 3. Laryngospasm is a feature in hypocalcaemia which may cause stridor.

 

64.     A 50yo chronic smoker came to OPD with complaint of chronic productive cough, SOB and wheeze. Labs: CBC=increase in PCV. CXR

>6ribs seen above the diaphragm in midclavicular line. ABG=pO2 decreased. What is the most likely dx?

  1. Interstitial lung disease
  2. Wegener’s granulomatosis
  3. Ca bronchi
  4. COPD
  5. Amyloidosis
  6. 1. What is the key?
  7. 2. What are the points in favour?

Ans. 1. The key is D. COPD.

Ans. 2. Points in favour: i) Age 50 yrs ii) Chronic smoker iii) Chronic productive cough, SOB and Wheeze iv) Raised PCV secondary to chronic hypoxaemia v) Low set diaphragm and widened horizontal ribs vi) Hypoxaemia on ABG.

 

 

65.      A 44yo pt has sudden onset of breathlessness and stridor few minutes after extubation for thyroidectomy. The pat had longstand- ing goiter for which he had the surgery. What is the most likely dx?

  1. Thyroid storm
  2. Hematoma
  3. Unilateral recurrent laryngeal nerve injury
  4. External laryngeal nerve injury
  5. Tracheomalacia

Ans. The key is tracheomalacia. [Prolonged pressure over trachea by goiter is a cause of tracheomalacia following thryroidectomy].

66.      A 15yo boy presents with generalized edema. His urinalysis reveals protein +++, eGFR =110. What is the most likely dx?

  1. IgA nephropathy
  2. Membranous nephropathy
  3. Minimal change disease
  4. PSGN
  5. Lupus nephritis

Ans. The key is C. Minimal change disease. [Points in favour: i) Age 15 yrs ii) Generalized oedema iii) Protein in urine +++ vi) Normal eGFR of 110 (Normal range- 90 to 120 mL/min)].

67.      A 72yo man is receiving chemotherapy for SCLC. He has his 4th tx 8 days ago. He has a cough with some green sputum but feels well. Temp=37.6C. Chest exam = few coarse crepitations in the right base. HR=92bpm. CBC: Hgb=12.5g/dL, WBC=1.1, Neutrophils=0.6, Plt=89. Sputum, urine and blood culture sent to microbiology. What is the most appropriate management?

  1. Broad spectrum antibiotics IV
  2. Broad spectrum antibiotics PO
  3. GCSF
  4. Postpone tx until bacteriology results available
  5. Reassure and send home
  6. 1. What is the key?
  7. 2. What is the Diagnosis?
  8. 3. What is the treatment of low WBC count?

Ans. 1. The key is A. Broad spectrum antibiotics IV

Ans. 2. The diagnosis is lower respiratory tract infection.

Ans. 3. GCSF subcutaneously. [it is the treatment of chemotherapy induced leucopenia].

68.      A 25yo woman with T1DM has delivered a baby weighing 4.5kg. Her uterus is well contracted. Choose the single most likely predis- posing factor for PPH from the options.

  1. Atonic uterus
  2. Cervical/vaginal trauma
  3. Retained POC
  4. Large placental site
  5. Rupture uterus
  6. 1. What is the key?
  7. 2. Reason for your answer.

Ans. 1. The key is B. Cervical/vaginal trauma

Ans. 2. The baby is a big baby. If patient’s uterus was not well con- tracted we would fear of atonic uterus! But as uterus is well contract- ed it is not atonic uterus. Rather most likely cause is trauma dring delivery of this big baby.

69.      A 23yo lady presents with headache. Exam: photophobia and generalized rash that doesn’t blanch on pressure. What must be done immediately?

  1. IV benzylpenicillin
  2. Isolate pt
  3. Gown and mask
  4. Blood culture

Ans. The key is A. IV benzylpenicillin.

70.     A 4yo baby has generalized tonic-clonic seizure and fever of 39C. his mother informs you that this has happened 3-4x before. What is the most likely dx?

  1. Febrile convulsion
  2. Absence seizures
  3. Epilepsy
  4. Partial complex seizure

Ans: The key is C. Epilepsy. It is wrong key. Correct key should be A. Febrile convulsion. [Points in favour: seizers in presence of high fever. Same previous illness indicate seizers during fever which goes against epilepsy. Though usually febrile convulsion dosen’t tend to recur but

it can recur as well. Epilepsy has no relation to fever but is due to pathology in brain].

71.      A middle aged Asian presents with episodes of fever with rigors and chills for last 1y. Blood film: ring form plasmodium with schuff- ner’s dots in RBCs. What is the drug to eradicate this infection?

  1. Doxycycline
  2. Mefloquine
  3. Proguanil
  4. Quinine
  5. Artesonate
  6. 1. What is the key?
  7. 2. What does Shuffner’s dot in RBC indicate?

Ans. 1. The key is B. Mefloquine. Probably wrong key as mefloquine can not eradicate hepatic cycle! Primaquine is the drug that can eradi- cate hepatic cycle. None of the given option can iradicate liver cycle!! Ans. 2. Shuffners dot indicates, it is plasmodium ovale or plasmodium vivex infestation.

72.      A 35yo woman had an uneventful lap chole 18h ago. She has a pulse=108bpm, temp 37.8C. There are signs of reduced air entry at the right base but the CXR doesn’t show an obvious abnormality. What is the most appropriate management strategy?

  1. Cefuroxime PO
  2. Ceftriaxone IV
  3. Chlorpheniramine PO
  4. Chest physiotherapy
  5. Reassure
  6. 1. What is the key?
  7. 2. What is the diagnosis?

Ans. 1. The key is D. Chest physiotherapy.

Ans. 2. Atelactasis. [Atelectasis can be seen in chest x-ray but some- times it becomes undetectable in chest x-ray as we are suspecting in this case].

 

73.       A 20yo pop star singer complains of inability to raise the pitch of her voice. She attributes this to the thyroid surgery she under- went a few months back. What is the most likely dx?

  1. Thyroid storm
  2. Bilateral recurrent laryngeal nerve injury
  3. Unilateral recurrent laryngeal nerve injury
  4. External laryngeal nerve injury
  5. Thyroid cyst

Ans. The key is D. External laryngeal nerve injury. [External laryngeal nerve functions to tense the vocal cords by activating the cricothyroid muscle, increasing pitch].

74.      A 28yo woman at 39wk gestation is in labor. She develops abdominal pain and HR=125bpm, BP=100/42mmHg, temp=37.2C and saturation=99%. Exam: lower abdomen is exquisitely tender. CTG=prv normal, now showing reduced variability and late decelera-

tion develops with slow recovery. She has had 1 prv LSCS for a breech baby. Choose the most appropriate CS complication for this lady?

  1. Endometritis
  2. UTI
  3. Urinary tract injury
  4. Pleurisy
  5. Uterine rupture

Ans. The key is E. Uterine rupture. [Features of shock, exquisitely tender lower abdomen with abdominal pain, previously normal CTG now becoming non-reassuring and previous history of LSCS makes the diagnosis of uterine rupture].

75.      An 8m infant presented with FTT and constipation. Exam: large tongue and fam hx of prolonged neonatal jaundice. What is the most likely dx?

  1. Downs syndrome
  2. Fragile X syndrome
  3. Praderwilli syndrome
  4. DiGeorge syndrome
  5. Congenital hypothyroidism
  6. 1. What is the key?
  7. 2. What are the points in favour?

Ans. 1. The key is E. Congenital hypothyroidism.

Ans. 2. Points in fevour:i) FTT (failure to thrive) ii) constipation iii) macroglossia iv) prolonged neonatal jaundice.

76.        A 3m infant has presented with recurrent infections. He has abnormal facies and CXR shows absent thymic shadow. What is the most likely dx?

  1. Downs syndrome
  2. Fragile X syndrome
  3. DiGeorge syndrome
  4. Marfans syndrome
  5. 1. What is the key?
  6. 2. What are the points in favour?

Ans. 1. The key is C. DiGeorge syndrome.

Ans. 2. Points in favour: i) Early age of onset ii) abnormal facies iii) absent thymic shadow on Chest X-ray iii) history of recurrent infection [Infections are common in children due to problems with the immune system’s T-cell-mediated response that in some patients is due to

an absent or hypoplastic thymus]. [in newborn can be recognized by convulsions from hypocalcaemia due to malfunctioning parathyroid glands and low level of parathyroid hormones].

77.       A 30yo man presents with deep penetrating knife wound. He said he had TT when he left school. What will you do for him now?

  1. Human Ig only
  2. Human Ig and TT
  3. Full course of tetanus vaccine only
  4. Human Ig and full course of tetanus vaccine
  5. Antibiotic

Ans. The key is B. Human Ig and TT. It is a wrong key!! According to UK greenbook correct key is A. Human Ig only. [For tetanus vaccine and management uk protocol is a bit different from other countries].

78.      A 32yo previously healthy woman has developed pain and swell- ing of both knees and ankles with nodular rash over her shins. As part of the inv a CXR has been performed. What is the single most likely CXR appearance?

  1. Apical granuloma
  2. Bilateral hilar lymphadenopathy
  3. Lobar consolidation
  4. Pleural effusion
  5. Reticular shadowing in the bases
  6. 1. What is the key?
  7. 2. What is the name of this condition? What are the points in favour?

Ans. 1. The key is B. bilateral hilar lymphadenopathy.

Ans. 2. The name is Lofgren’s syndrome. It is the triad of i) erythema nodosum ii) bilateral hilar lymphadenopathy and iii) arthralgia.

79.       A neonate’s CXR shows double bubble sign. Exam: low set ears, flat occiput. What is the most likely dx?

  1. Downs syndrome
  2. Fragile X syndrome
  3. Turner’s syndrome
  4. DiGeorge syndrome
  5. 1. What is the key?
  6. 2. What double bubble sign indicate?

Ans. 1. The key is A. Down’s syndrome.

Ans. 2. Double bubble sign indicate duodenal atresia.

 

80.      A 19yo boy complains of itching on the site of insect bite. What is the single most appropriate management?

  1. Penicillin oral
  2. Doxycycline oral
  3. Oral antihistamine
  4. Oral ciprofloxacin
  5. Reassurance

Ans. The key is C. Oral antihistamine.

81.       A man presents with scrotal swelling, the swelling is cystic and is non-tender. It is located in the upper pole of the posterior part of the testis. What is the most likely dx?

  1. Epididymal cyst
  2. Testicular ca
  3. Hydrocele
  4. Teratoma
  5. Testicular torsion

Ans. The key is A. Epididymal cyst. [the location of upper pole of the posterior part of testis is the common site for epididymal pathology].

82.      A young footballer has collapsed during a game. During initial evaluation: RR=14/min, pulse=88bpm, BP=110/70mmHg. He seems to be sweating and muttering some incomprehensible words. What is the most imp next step?

  1. CT
  2. MRI
  3. Blood sugar
  4. Body temp
  5. IV fluids

Ans. The key is C. Blood sugar. [Normal vital signs, sweating and collapse during exercise gives suspicion of hypoglycemia which is very easy to treat but fatal if left unrecognized. So most important next step is to check the blood sugar].

 

83.       A 45yo waitress complains of pelvic pain which worsens

pre-menstrually and on standing and walking. She also complains of post-coital ache. Select the most likely cause leading to her symp- toms?

  1. PID
  2. Endometritiosis
  3. Pelvic congestion syndrome
  4. Adenomyosis
  5. Premature ovarian failure

Ans. The key is C. Pelvic congestion syndrome. [Women with pelvic congestion syndrome experience a constant pain that may be dull and aching, but is occasionally more acute. The pain is worse at the end of the day and after long periods of standing, and sufferers get relief when they lie down. The pain is worse during or after sexual intercourse, and can be worse just before the onset of the menstrual period. Cause is attributed to female hormone estrogen induced vas- cular (venous) changes. So it is usually seen in females].

 

84.     A 37yo female had a fall with outstretched hand, presented with dinner fork deformity and tenderness over the right arm. What is the single most associated nerve injury?

  1. Axillary nerve
  2. Radial nerve
  3. Musculocutaneous nerve
  4. Median nerve
  5. Ulnar nerve

Ans. The key is D. Median nerve. [Median nerve is the nerve injured in Colle’s fracture].

85.      A mother comes with her 15m child. Which of the following will bother you?

  1. Shies away from strangers
  2. Can walk but not run
  3. Vocabulary consists of only 2 meaningless words
  4. She can’t make a sentence
  5. None
  6. 1. What is the key?
  7. 2. How many words is told clearly by a 15 month old child?

Ans. 1. The key is C. Vocabulary consists of only 2 meaningless words. Ans. 2. At 15 months the child can clearly say 5 words and his first meaningful clear word he says at 12 months.

86.      A 35yo lady who has been using IUCD for one year now com- plains of pelvic pain and heavy painful periods. Select the most likely cause leading to her symptoms?

  1. PID
  2. Endometriosis
  3. Adenomyosis
  4. Fibroids
  5. Asherman syndrome
  6. 1. What is the key?
  7. 2. What points favour your diagnosis?

Ans. 1. The key is A. PID.

Ans. 2. The given picture may have D/D of PID or fibroid. As IUCD is a risk factor for PID, it is the most likely diagnosis of given picture. Menorrhagia for fibroids are usually painless].

87.      The dx cells of Hodgkin disease are:

  1. T-cells
  2. R-S cells
  3. B-cells
  4. Macrophages
  5. Auer rods

Ans. The key is B. R-S cells. [Diagnostic cell in Hodgkins disease is Reed-Sternberg cells].

88.      A 16yo girl is admitted after taking a paracetamol OD 4 h ago. She has consumed large amounts of alcohol. Her plasma parac- etamol conc is just below the conc that would suggest tx. What should be the tx option for her?

  1. Refer to psychiatry ward
  2. Refer to medical ward
  3. N-acetylcystine
  4. Serum plasma paracetamol
  5. No further investigation

Ans. The key is A. Refer to psychiatry ward. [Short term acute alcohol consumption causes enzyme inhibition as in present case and even then paracetamol level is under tx level. So the patients drug level is in safe side but simultaneous drug overdose and alcohol consumption needs psychiatric evaluation and hence the option here is A].

89.      A 64yo woman has been on HRT for 9yrs. She had regular withdrawal bleeds until 3 yrs ago and since then has been taking a no bleed prep. Recently she noticed a brown vaginal discharge. Choose the single most appropriate initial inv?

  1. Cervical smear
  2. High vaginal swab
  3. TFT
  1. Transvaginal US
  2. 1. What is the key?
  3. 2. Why this test will be done?

Ans. 1. The key is D. Transvaginal US.

Ans. 2. To determine the endometrial thickness!

In a postmenopausal woman with vaginal bleeding, the risk of cancer is approximately 7.3% if her endometrium is thick (> 5 mm) and < 0.07% if her endometrium is thin ( 5 mm).

In postmenopausal women without vaginal bleeding, the risk of can- cer is approximately 6.7% if the endometrium is thick (> 11 mm) and 0.002% if the endometrium is thin ( 11 mm).

90.      A young girl complains of episodic headaches preceded by forti- fication spectra. Each episode last for 2-3 days. During headache pt prefers quiet, dark room. What is the tx of choice for acute stage?

  1. Paracetamol
  2. Aspirin
  3. Sumatriptan
  4. Gabapentin
  5. Cafergot

Ans. The key is B. Aspirin. This is a wrong key! Correct key is A. Parac- etamol. [Aspirin can cause Reye’s syndrome in children, so should not be given. If paracetamol or ibuprofen fail only then triptan should be given].

 

 

 

 

 

91.        A 60yo pt recovering from a surgery for toxic goiter is found to be hypotensive, cyanosed in the the RR. Exam: tense neck. There is blood oozing from the drain. What is the most likely dx?

  1. Thyroid storm
  2. Reactionary hemorrhage
  3. Secondary hemorrhage
  4. Primary hemorrhage
  5. Tracheomalacia

Ans. The key is B. Reactionary haemorrhage. [in the recovery room, cyanosis, hypotension, tense neck, woozing of blood from drain; all these goes in favour of reactionary haemorrhage. “reactionary,” hem- orrhage that occurs within the first 24 hours after surgery].

 

92.      A 33yo man is hit by a car. He loses consciousness but is found to be fine by the paramedics. When awaiting doctors review in the ED he suddenly becomes comatose. What is the most likely dx?

  1. SAH
  2. Subdural hemorrhage
  3. Intracerebral hemorrhage
  4. Extradural hemorrhage

Ans. The key is D. Extradural haemorrhage. [Age 33 (younger age), considerable head trauma, and lucid interval (present in both extradu- ral and subdural) are the points in favour].

93.        A 77yo male presents with hx of enuresis and change in behav- ior. Exam: waddling gait. What is the most likely dx?

  1. Subdural hemorrhage
  2. Brain tumor
  3. Normal pressure hydrocephalus
  4. Psychotic depression

Ans. The key is C. Normal pressure hydrocephalus. [age (usually occurs in 60s or 70s), loss of bladder control (enuresis), waddling gait and behavior change are all features of normal pressure hydrocepha- lus].

94.       A 29yo teacher is involved in a tragic RTA. After that incident, he has been suffering from nightmares and avoided driving on the motorway. He has been dx with PTSD. What is the most appropriate management?

  1. CBT
  2. Diazepam
  3. Citalopram
  4. Dosalepin
  5. Olanzepin

Ans. The key is A. CBT. [CBT is the treatment of choice in PTSD].

 

 

95.        A 5yo child presents with fever. He looks pale. His parents say he always feels tired. On exam: orchidomegaly & splenomegaly. Labs: WBC=1.7, Hgb=7.1, Plt=44. What is the dx?

  1. ALL
  2. CLL
  3. AML
  4. CML
  5. Hodgkins

Ans. The key is A. ALL. [normally in ALL CBC shows raised WBC, low RBC and low platelet; but it is also possible to all cell lines to be depressed, as is the presented case].

96.        A 6wk child is brought in with vomiting, constipation and de- creased serum K+. What is the dx?

  1. Pyloric stenosis
  2. Duodenal atresia
  3. Hirschsprung disease
  4. Achalasia cardia
  5. Tracheo-esophageal fistula

Ans. The key is A. Pyloric stenosis. [why not duodenal atresia? Pyloric stenosis is much more commoner than duodenal atresia; in duodenal atresia the vomitus should contain bile, which is not the case in pyloric stenosis].

97.        A 17 yo girl had an episode of seizure. Contraction of muscles started from around the interphalangeal joints, which spread to the muscles of wrist and elbow. Choose possible type of seizure?

  1. Grand mal
  2. Tonic clonic
  3. Myoclonic
  4. Absent

Ans. The key is C. Myoclonic. [seizers associated with contraction of specific muscle group is seen in myoclonic seizers].

98.         46yo man, known case of chronic GN presents to OPD. He feels well. BP = 140/90mmHg. Urine dipstick: protein ++, blood ++ and serum creatinine=106mmol/L. Which medication can prevent the progression of this dx?

  1. ACEi
  2. Diuretics
  3. Cytotoxic meds
  4. Longterm antibiotics
  5. Steroids

Ans. The key is A. ACEI. [renal impairment is delayed by ACEI].

99.        A 23 yo girl presented with perioral paresthesia and carpopedal spasm 20 mins after a huge argument with her boyfriend. What is the next step for this pt?

  1. SSRI
  2. Diazepam
  3. Rebreath into a paper bag
  4. Propranolol
  5. Alprazolam
  6. 1. What is the key?
  7. 2. What is the likely diagnosis?

Ans. 1. The key is C. Rebreathing in paper bag. [hyperventilation caus- es CO2 washout and respiratory alkalosis. An increase in pH, alkalo-

sis, promotes increased protein binding, which decreases free calcium levels and this low calcium causes perioral persthesia and carpopedal spasm. If you continue breathing and rebreathing in paper bag it will allow CO2 concentration to rise in paper bag and as you rebreath this again and again you will regain some washed out CO2 and thus relief to this alkalosis].

Ans. 2. The girl may have anxiety disorder when it precipitates leads to hyperventilation syndrome. X

100.     A 25 yo woman has been feeling anxious and nervous for the last few months. She also complains of palpitations and tremors. Her symptoms last for a few minutes and are very hard to control. She tells you that taking alcohol initially helped her relieve her symptoms but now this effect is wearing off and she has her symptoms even after drinking alcohol. What is the dx?

  1. Panic disorder
  2. Depression
  3. OCD
  4. Alcohol addiction
  5. GAD

Ans. The key is A. Panic disorder.

101.      A 2yo child is very naughty. His teacher complains that he is easily distracted. His parents say that he can’t do a particular task for a long time. He sometimes hurts himself and breaks many things. This causes many troubles at home. What is the dx?

  1. ASD
  2. Dyslexia
  3. ADHD
  4. Antisocial personality disorder
  5. Oppositional defiant

Ans. The key is C. ADHD (Attention deficit hyperreactive disorder).

102.      A 79 yo lady who is otherwise well recently started abdomi- nal pain. She is afebrile and complains that she passed air bubbles during urination. A urethral catheter showed fecal leakage in the urinary bag. What is the likely pathology?

  1. Diuretics
  2. CD
  3. Rectosigmoid tumor
  4. Large bowel perforation
  5. UC

Ans. The key is B. CD. [debate came that Crohn’s disease cannot occur in 79 yrs but this is not the case! “Crohn’s disease can occur at any age, but is most frequently diagnosed in people ages 15 – 35. About 10% of patients are children under age 18”. [http://www.nytimes.com/ health/guides/disease/crohns-disease/risk-factors.html]. So I think

it can occur in this age also and the feature like fistula is a common

 

association of CD].

 

 

103.     A 2 month child with diarrhea and vomiting for 6 days is brought in looking lethargic. What is the appropriate initial inv?

  1. BUE
  2. Random blood sugar
  3. CBC
  4. CXR
  5. AXR

Ans. The key is A. BUE. [Electrolyte imbalance should be checked in this baby presenting with diarrhea and vomiting for considerable time].

104.   A 72 yo man fell while shopping and hurt his knee. His vitals are fine. He speaks in a low voice and is very slow to give answers. What is the most probable dx?

  1. Alzheimers
  2. Vascular demetia
  3. TIA
  4. Pseudo-dementia
  5. Picks dementia

Ans. The key is A. Alzheimers

  1. A 47 yo man met with a He has multiple injuries. Pelvic fx is confirmed. He has not passed urine in the last 4 hrs. What is the next appropriate management for this pt?

 

  1. Urethral catheter
  2. Suprapubic catheter
  3. IV fluids
  4. IV furosemide
  5. Insulin
  6. 1. What is the key?
  7. 2. What is the reason of this management?

Ans. 1. The key is B. Suprapubic catheter.

Ans. 2. In pelvic fracture there is chance of urethral rupture and hence displacement of urethral catheter while try to place it.

106.    A 49 yo pt presents with right hypochondriac pain. Inv show a big gallstone. What is the most appropriate management?

  1. Lap Cholecystectomy
  2. Reassure
  3. Low fat diet
  4. Ursodeoxycholic acid
  5. Emergency laparotomy
  1. 1. What is the key?
  2. 2. Points in favour?

Ans. 1. The key is A. Lap Cholecystectomy.

Ans. 2. i) as symptomatic only reassurence is not appropriate ii) as big stone ursodyoxycholic acid is less effective iii) less invasiv is preferred so laparoscopic rather than laparotomy.

 

 

107.     In a man who is neglected and alcohol dependent, whith high suicidal risk, which factor can increase this risk further?

  1. Alcohol dependence
  2. SSRI
  3. S]moking
  4. Agoraphobia
  5. Court involvement

Ans. The key is A. Alcohol dependence. This is wrong key!! Correct key should be B. SSRI. [He is already alcohol dependent. So further (additional) risk factor is SSRI (SSRI has well known risk of develop- ing suicidal ideation]. Arrest and court involvement is also a risk factor and in a neglected person it is also a strong risk factor.

108.    A 71 yo man presents with coarse tremor. He is on some meds. Which one can be the reason for the tremor?

  1. Lithium
  2. Diazepam
  3. Fluoxetine
  4. Imipramine
  5. Haloperidol

Ans. The key is A. Lithium. Actually in therapeutic dose lithium causes fine tremor but in toxic dose it causes coarse tremor. So the probable answer is lithium.

109.     A young woman complains of diarrhea, abdominal cramps and mouth ulcers. AXR shows distended transverse colon with globet cell depletion on rectal biopsy. What is the most probable dx?

  1. CD
  2. UC
  3. Bowel Ca
  4. Bowel obstruction
  5. IBS
  6. 1. What is the key?
  7. 2. What are points in favour?

Ans. 1. The key is B. UC.

Ans. 2. In UC there is goblet cell depletion and less mucous produc- tion in contrast with CD where there may be goblet cell hyperplasia

and mucous secretion is not reduced. Please note aphthous ulcer can develop in both CD and UC.

110.    After eating a cookie at a garden party, a child began to cough and went blue. The mother also noticed that there were swollen patches on the skin. What is the dx?

  1. Allergic reaction
  2. Aspiration of food
  3. Cyanotic heart disease
  4. Trachea-esophageal fistula
  5. Achalasia cardia

Ans. The key is A. Allergic reaction.

 

 

111.    A 70 yo man presents with balance difficulties, vomiting and nausea. Which of the following is the best inv?

  1. MRI cerebellum
  2. CT cerebellum
  3. Skull XR
  4. LP
  5. Blood culture

Ans. The key is A. MRI cerebellum. [Balance difficulties, vomiting and nausea suggests cerebellar lesion. In posterior fossa lesion MRI is preferred].

112.    A 2 yo pt presents with colicky pain which radiates from loin to groin. He complains of similar episodes in the past. Inv has been done and 7mm stone was found in the ureter. What is the most appropriate management?

  1. Percutaneous nephrolithotomy
  2. Open surgery
  3. Ureteroscopy or laser
  4. Conservative tx
  5. ESWL
  6. 1. What is the key?
  7. 2. What treatments are recommended for different sized stones in Adults?

Ans. 1. The key is E. ESWL. [Given its minimally invasive features, extracorporeal shock wave lithotripsy (ESWL) has become a prima- ry mode of treatment for the pediatric patients with reno-ureter-

al stones].

Ans. 2. Stones < 5mm: pass spontaneously, Increase fluid intake.

 

Stones 5mm-10mm /pain not resolving: medical expulsive therapy—> Nifedipine or Tamsulosin(and/or prednisolone).

 

Stones 10mm-2cm: ESWL or Ureteroscopy using dormia basket.

Stones > 2cm/large/multiple/complex: Percutaneous nephrolithoto- my.

113.      A footballer has been struck in the groin by a kick and a pres- ents with severe pain and mild swelling in the scrotum. What is the most appropriate next step?

  1. USG
  2. Doppler
  3. Exploratory surgery
  4. IV fluids
  5. Antibiotics

Ans. The key is C. Exploratory surgery. [To exclude torsion].

114.    A 47 yo ex-soldier suffers from low mood and anxiety. He can’t forget the images he faces before and has always had flashbacks. He is not able to watch the news because there are usually some reports about war. What is he suffering from?

  1. Depression
  2. PTSD
  3. Panic attack
  4. Agoraphobia
  5. GAD

Ans. The key is B. PTSD. [repeated flashbacks and tendency to avoid the thoughts of stressor is diagnostic of PTSD].

115.     A 36 yo woman has recently spent a lot of money on buying clothes. She goes out almost every night with her friends. She be- lieves that she knows better than her friends, so she should choose the restaurant for eating out. She gave hx of having low mood at 12 yo. What’s the dx?

  1. Mania
  2. Depression
  3. Bipolar affective disorder
  4. Borderline personality disorder
  5. Dysthymia

Ans. The key is C. Bipolar affective disorder. [Initial depressive episode (may be befor a long) followed by mania is bipolar affective disorder].

116.     A 28 yo female presents with a 3m hx of diarrhea. She com- plains of abdominal discomfort and passing stool 20x/day. Exam=fe- brile. Barium enema shows cobblestone mucosa. What is the most likely dx?

  1. Ameoba
  2. Colon Ca
  3. GE
  4. CD
  5. UC

Ans. The key is D. CD. [Hx of diarrhea, abdominal discomfort, and patient being febrile indicate gut inflammation and cobblestone appearance on barium enema is suggestive of CD].

117.     A child is brought in with high grade fever, runny nose and bark- like cough. He is also drooling. What is the most appropriate tx for this child?

  1. Corticosteroids
  2. Paracetamol
  3. Adrenaline nebulizer
  4. IV antibiotics
  5. Intubation under GA

Ans. The key is E. Intubation under GA. This is a wrong key! In epi- glottitis there is no cough and bark like cough is diagnostic of croup! So the correct key should be a. Corticosteroids. [high fever, bark-like cough in a child suggest croup which is treated with corticosteroids. Drooling is sign ofepigottitis which may be a bad recall here].

118.     A 78yo lady on warfarin for atrial fibrillation lives in a care home. She presents with hx of progressive confusion for three days. She was also noticed to have bruises on her arms. INR = 7. What is the most probable dx?

  1. Alzheimers
  2. Delirium
  3. Subdural hemorrhage
  4. Vascular dementia
  5. Pick’s dementia

Ans. The key is C. Subdural haemorrhage. [Age 78 yrs, living in a care home where unnoticed trivial injury is possible (like fall), warfarine and high INR is potential risk factor of subdural haemorrhage suggested by bruises on arms also].

119.      A 28 yo drug user presents to the ED with collapse and anuria. His serum K+ = 7.5mmol/L. CXR = early pulmonary edema. What is the next appropriate management?

  1. Urgent hemodialysis
  2. IV calcium gluconate
  3. IV insulin + dextrose
  4. Furosemide
  5. IV NS 9%

Ans. The key is B. IV calcium gluconate. [IV calcium gluconate is given to protect the heart from lethal arrhythmia or cardiac arrest from high potassium level and used to buy time while definitive management

is being taken. Actually calcium gluconate don’t lower the potassium level that much but protect the heart from arrhythmia].

120.    A 32 yo woman suffers an episode of severe occipital headache with vomiting and loss of consciousness. She is brought to the hos- pital where she is found to be conscious and completely alert. Exam: normal pulse & BP. No abnormal neurological signs. What is the next step in management?

  1. Admission for observation
  2. CT brain
  3. MRI head
  4. Reassurance and discharge
  5. XR skull

Ans. The key is B. CT brain. [basilar migraine can cause severe headache and LOC. But there occurs no neurological deficit and on recovering from unconsciousness becomes completely alert. But to di- agnose basilar migraine there should at least history of two migraine attacks with aura. As here diagnostic criteria of basilar migraine is not fulfilled we can not discharge the patient without neuroimaging like CT or MRI].

121.     A 25 yo woman was brought to the ED by her boyfriend. She has many superficial lacerations on her forearm. She is so distressed and constantly says her boyfriend is going to end the relationship. She denies trying to end her life. What is the most likely dx?

  1. Acute psychosis
  2. Severe depression
  3. Psychotic depression
  4. Borderline personality disorder
  5. Schizophrenia

Ans. The key is D. Borderline personality disorder. [ Borderline per- sonality disorder: Act impulsively and develop intense but short-lived emotional attachment to others. They are usually attention seekers but not suicidal].

 

122.     A young woman was brought to the hospital. On exam she has low temperature and tremor. She says when she closes her eyes, she can see colors. What drug has been used?

  1. Amphetamines
  2. LSD
  3. Cocaine
  4. Heroine
  5. Ecstasy

Ans. The key is B. LSD.

123.      A lady comes in severe liver disease and hematemesis. Her INR is >10. What should she be given?

  1. FFP
  2. Steroids
  3. Whole blood
  4. IV fluids
  5. Vit K

Ans. The key is A. FFP.

124.    After eating a cookie at a garden party, a child began to cough and went blue. The mother also noticed that there were swollen patches on the skin. What is the initial management?

  1. OTC antihistamine
  2. Oxygen
  3. Bronchodilators
  4. Epinephrine IM
  5. Nebulized epinephrine

Ans. The key is D. Epinephrine IM [anaphylaxis with partially blocked airway].

125.     A 63 yo female is noted to have left pupil irresponsive to light and is dilated. What is the most probably dx?

  1. Pontine hemorrhage
  2. Subdural hemorrhage
  3. Cerebellar hemorrhage
  4. Extradural hemorrhage
  5. Subarachnoid hemorrhage

Ans. The key is D. Extradural hemorrhage. This is a wrong key! Correct key is B. Subdural hemorrhage.

126.     A 28yo business executive presents at the GP asking for some help because she has been arguing with her boyfriend frequently. She is worried about her weight, and she thinks she may be fat. She has been on a diet and lost 7 kgs in the last 2 months on purpose. She is eating less. She used to do a lot of exercise. Now she says she’s feeling down, has some insomnia and feels tired and without energy. She has not showed up at work. She is worried because re- cently she got a loan to buy a luxury car. She can’t be fired. She com- plains about her low mood. She thinks this is weird because she used to be extremely productive. She used to work showing an excellent performance at the office. She even received compliments from her boss. How, she says her boyfriend is angry because her apartment is a chaos. Usually she spends a lot of time cleaning it, even upto 3 AM. She liked it to be perfect, but not it’s a mess. On exam: BMI=23, no other signs. What is the most probably dx?

  1. Anorexia nervosa
  2. Bipolar disease
  3. Binge eating disorder
  4. Hyperthyroidism
  5. Schizophrenia

Ans. The key is B. Bipolar disorder. [Now she is depressed but before hypomanic which makes the likely dx of bipolar disorder].

 

 

127.     A woman brought her husband saying she wants the ‘thing’ on his forehead removed. The husband is refusing tx saying it improves his thinking. What is the most appropriate next step?

  1. Assess his mental capacity to refuse tx
  2. Remove lesion
  3. Refer to ED
  4. Mini-mental state exam
  5. Refuse surgery and send pt back

Ans. The key is A. Assess his mental capacity to refuse treatment.

 

 

128.     A 37 yo man who has many convictions and has been impris- oned many times has a hx of many unsuccessful relationships. He has 2 boys but doesn’t contact them. What is the most probable dx?

  1. Borderline personality disorder
  2. Schizophrenia
  3. Avoidant personality disorder
  4. Histrionic personality disorder
  5. Antisocial behavior disorder

Ans. The key is E. Antisocial behavior disorder. [Antisocial personality disorder is a particularly challenging type of personality disorder, char- acterised by impulsive, irresponsible and often criminal behavior].

129.     A 60 yo man has a pathological rib fx. He also complains of recurrent infection. BMA is done. Labs: Ca2+ = 3.9mmol/L and ALP

= 127u/L. what type of cell would be found in abdundance in the marrow smear?

  1. Plasma cell
  2. Myeloid cell
  3. Bence-jones protein
  4. Megakaryocytes
  5. Reticulocytes
  6. 1. What is the key.
  7. 2. What is the diagnosis?
  8. 3. What are the points in favour of diagnosis?

Ans. 1. The key is A. Plasma cell.

Ans. 2. The diagnosis of multiple myeloma.

Ans. 3. Points in favour: i) age 60 yrs ii) pathological rib fracture (from metastases) iii) recurrent infection (due to B cell dysfunction (manifested as hypogammaglobulinemia), numerical and functional abnormalities of T cells, and dysfunction of natural killer cells), iv) raised calcium level.

130.     A child presents with blue marks on the sclera, short stature and heart murmur. What is the dx?

  1. Osteogenesis imperfect
  2. Hypopituitarism
  3. VSD
  4. Achondrogenesis
  5. Dwarfism

Ans. The key is A. Osteogenesis imperfecta.

131.     A 5month child can’t speak but makes sounds. She can hold things with palm, not fingers. Can’t sit independently but can hold her hand and sit when propped up against pillows. How’s the childs development?

  1. Normal
  2. Delayed speech
  3. Delayed sitting
  4. Delayed motor development

Ans. The key is A. normal

 

132.     A 27 yo woman has hit her neck in an RTA without complains of tingling or motor loss. What is the next most appropriate inv?

  1. MRI
  2. XR
  3. CT cervical
  4. Diagonal XR

Ans. The key is B. X-ray. [As there is no neurological deficit we can ex- clude any fracture by x-ray first]. [Diagonal x ray means ,oblique view of cervical spine. By this view we can assess facet joint arthopathy.

This doesn’t related to RTA].

133.      A young female who has many superficial lacerations was brought into the ED by her boyfriend for superficially lashing her upper arm. She is adamant and screaming that she is not suicidal but scared her boyfriend wants to leave her. What is the dx?

  1. Acute psychosis
  2. Severe depression
  3. Obsessive
  4. Bipolar
  5. Borderline personality
  6. Schizophrenia

Ans. The key is acute psychosis. Probably this is wrong key! Correct key should be E. Borderline personality disorder. [Borderline person- ality disorder (BPD), also known as emotionally unstable personality disorder, is a long term pattern of abnormal behavior characterized by unstable relationships with other people, unstable sense of self, and unstable emotions.[3][4] There is often an extreme fear of aban- donment, frequent dangerous behavior, a feeling of emptiness, and self-harm].

134.    A 22yo woman was brought by her boyfriend with multiple su- perficial lacerations. There are scars of old cuts on her forearms. She is distressed because he wants to end the relationship. She denies suicide. What is the most likely dx?

  1. Acute psychosis
  2. Borderline personality disorder
  3. Severe depression
  4. Schizoid personality
  5. Psychotic depression

Ans. The key is B. Borderline personality. [Borderline personality dis- order (BPD), also known as emotionally unstable personality disorder, is a long term pattern of abnormal behavior characterized by unstable relationships with other people, unstable sense of self, and unstable emotions.[3][4] There is often an extreme fear of abandonment, fre- quent dangerous behavior, a feeling of emptiness, and self-harm].

135.     A 31yo single man lives with his mother. He usually drives to work. He always thinks when the traffic lights change, his mother is calling him, so he drives back home. What is the dx?

  1. OCD
  2. GAD
  3. Schizophrenia
  4. Bipolar
  5. Cyclothymia

Ans. The key is C. Schizophrenia. [ delusion of reference – he thinks that the changing traffic lights are giving message to him].

136.     A 56yo woman is known case of pernicious anemia. She refuses to take hydroxycobalamin IM as she is needle shy. She asks for oral medication. Why will oral meds be not effective?

  1. Intrinsic factor def
  2. Malabsorption
  3. Irritated gastric mucosa
  4. Lack of gastric acidity

Ans. The key is A. Intrinsic factor def. [Vitamin B12 cannot be ab- sorbed without intrinsic factor].

137.      An 11m baby had an apnea event. The parents are worried that if something like this happens in the future, how they are to deal. Advise them about infant CPR.

  1. Index and middle finger compression
  2. Compression with palm of one hand
  3. Compression with palm of two hands
  4. Compression with rescue breaths 30:2
  5. Compression with rescue breaths 15:2

Ans. The key is A. Index and middle finger compression.

138.     A teacher brings in a child who says she fell down after hitting a table. On probing further, you decide that it was most probably an absence seizure. What led you to this dx?

  1. The child had not eaten since morning
  2. The child suddenly went blank and there was up-rolling of eyes
  3. The child started moving his fingers uncontrollably before he fell
  4. The child’s body became rigid and then started to jerk

Ans. The key is B. The child suddenly went blank and there was up-rolling of eyes.

139.     A man has discharge from his left ear after a fight. Where is the discharge coming from?

  1. CSF
  2. Inner ear
  3. Outer ear
  4. Brain

Ans. The key is A. CSF. [probable fracture base of skull].

140.     A 40 yo manic depressive is noted to have high serum levels of lithium and profound hypokalemia. His GP had started him on anti-HTNs. Choose the single most likely cause?

  1. Verapamil
  2. Amiodarone
  3. Ranitidine
  4. Lithium
  5. Thiazide

Ans. The key is E. Thiazide. [Thiazide was prescribed for Hpt and when lithium was prescribed its level increased due to thiazide and thiazide also caused hypokalemia resulting the given picture].

141.      A 74yo man presents with weakness in his arm and leg from which he recovered within a few days and short term memory loss. He has an extensor plantar response. He has similar episodes 2 years ago and became unable to identify objects and to make proper judg- ment. What is the most appropriate dx?

  1. Alcoholic dementia
  2. Pick’s dementia
  3. Huntington’s disease
  4. Alzheimer’s disease
  5. Vascular dementia

Ans. The key is E. Vascular dementia. [hemiparesis, memory im- pairment, extensor planter reflex, inability to identify objects, poor judgment are features of strokes and thus vascular dementia].

142.     A nurse comes to you saying that she has recently developed the habit of washing her hands after every 15-20 mins. She is unable to conc on her work and takes longer than before to finish tasks as she must constantly wash her hands. What is the most appropriate management?

  1. CBT
  2. SSRI
  3. ECT
  4. Antipsychotics
  5. Desensitization
  6. 1. What is the key?
  7. 2. What is the diagnosis?

 

Ans. 1. The key is A. CBT.

Ans. 2. The diagnosis is OCD.

143.      A 61yo man underwent a surgery in which ileal resection had been done. He complains of fatigue, headache, and heart racing. Labs: MCV=108fL, Hgb=8.9g/dL. What is the most likely dx?

  1. Vit B12 def
  2. Iron def
  3. Folate def
  4. Hemolytic anemia
  5. Anemia of chronic disease
  6. 1. What is the key?
  7. 2. What are the points in favour?

Ans. 1. The key is Vit. B12 deficiency.

Ans. 2. Vit B12 is absorbed mostly in ileum. [As ileal resection is done B12 is not absorbed leading to megaloblastic anaemia].

144.    A 7yo is brought by his mother who says that he was well at birth but has been suffering from repeated chest and GI infections since then. She also says that he is not growing well for this age. What is the likely condition of this child?

  1. CF
  2. SCID
  3. Primary Tcell immunodeficiency
  4. Primary Bcell immunodeficiency
  5. Malabsorption
  6. 1. What is the key?
  7. 2. What are the points in favour?

Ans. 1. The key is A. cystic fibrosis.

Ans. 2. CF involved in production of sweat, respiratory mucous, di- gestive fluid and mucous. These secretion becomes thick than normal predisposing to lung and GI infections since birth.

145.     A 3yo child has a high temp for 4 days and he had not seen a doctor. Then mother notices rashes on buccal mucosa and some around the mouth. What is the most appropriate dx?

  1. Measles
  2. Roseola infectiosum
  3. Rubella
  4. Chicken pox
  5. Impetigo

Ans. The key is B. Roseola infectiosum. It is a wrong key! The correct key should be A. Measles! [Rash on buccal mucosa (koplicks spot) are characteristic of measles. Also as the rash developed after 4 days fever the dx is measles!].

 

 

146.     A 70yo lady presents with fever for 3d and confusion. There is no significant PMH. What is themost probable dx?

  1. Delirium
  2. Hypoglycemia
  3. Alzheimers
  4. DKA

Ans. The key is A. Delirium. Delirium is an acute confusional state and declined cognitive function which involves changes in arousal (hyper- active, hypoactive or mixed), perceptual deficits, altered sleep-wake cycle, and psychotic features such as hallucinations and delusions.

147.     An obese mother suffers from OSAS. Which of the following inv is best for her?

  1. ABG
  2. Overnight pulse-oximetry
  3. Polysomnography
  4. EEG

Ans. The key is B. Overnight pulse-oxymetry. [It is already a diagnosed case of OSAS. So no need for reconfirmation with polysomnography. If like to know the current status or monitor overnight pulse oxymetry is good].

148.     A 28yo business man came to the sexual clinic. He was worried that he has HIV infection. 3 HIV tests were done and all the results are negative. After a few months, he comes back again and claims that he has HIV. What is the dx?

  1. Somatization
  2. Hypochondriac
  3. Mancheusens
  4. OCD
  5. Schizophrenia

Ans. The key is B. Hypochondriac. [Worry about having a serious illness].

149.     A 6wk child presents with progressive cyanosis, poor feeding, tachypnea over the first 2 wks of life and holosystolic murmur. What is the most appropriate condition?

  1. ASD
  2. VSD
  3. Tricuspid atresia
  4. PDA
  5. TOF
  6. 1. What is the key?
  7. 2. What are the points in favour?

Ans. 1. The key is E. Tetralogy of Fallot. It is a wrong key!! Correct answer is C. tricuspid atresia.

Ans. 2. Points in favour: i) tachypnoea over first 2 wks of life ii) pro- gressive cyanosis iii) poor feeding iv) holosystolic murmur of VSD.

 

 

 

 

150.    A 29yo woman who was dx to have migraine presents with severe onset of occipital headache. She lost her consciousness. CT=normal. Neurological exam=normal. What is the most appropri- ate management?

  1. Repeat CT
  2. MRI
  3. LP
  4. XR
  5. No inv required
  6. 1. What is the key?
  7. 2. What is the diagnosis?
  8. 3. What are the points in favour?

Ans. 1. The key is E. No investigation required. Ans. 2. The diagnosis is basilar migraine.

Ans. 3. Points in favour i) history of migraine ii) severe occipital head- ache iii) LOC iv) CT normal v) neurological examination is normal.

 

151.     A 19yo man has been happier and more positive than usual, with more energy than he has ever felt before for no particular reason. He has been getting more work done at the office today and has been socializing with his friends as usual. What is the most likely dx?

  1. Atypical depression
  2. Marked depression
  3. Bipolar syndrome
  4. Psychosis
  5. Hypomania
  6. 1. What is the key?
  7. 2. What are the points in favour of this diagnosis?

Ans. 1. The key is E. Hypomania.

Ans. 2. i) elevated mood ii) more energy than before iii) getting more work done at the office (loss of inhibition). These features are com- mon for both mania and hypomania!! Then why it is not mania? It is not mania as in mania you will get psychotic symptoms like i) delusion of grandeur ii) auditory hallucinations, which are absent here!

152.     A 35yo female attempts suicide 10x. There is no hx of psychiat- ric problems and all neurological exams are normal. What is the best tx?

  1. Problem focused tx
  2. CBT
  3. Antipsychotic
  4. Antidepressant
  5. ECT

Ans. The key is A. Problem focused tx. [patient is not psychotic and with normal neurology! So she may getting some problem in family life, finance, job or somewhere like this which she is not able to cope with and that is leading to her suicidal thoughts].

 

 

153.      A 57yo man presents with weight loss, tiredness, fever and ab- dominal discomfort. Exam: spleen palpable up to the umbilicus. Labs: WBC=127, Hgb=8.7, Plt=138. What is the most likely dx?

  1. CML
  2. AML
  3. CLL
  4. AML
  5. Polycythemia
  6. 1. What is the key?
  7. 2. Why it is so diagnosed?

Ans. 1. The key is A. CML.

Ans. 2. Points in favour of CML: i) Age 57 years ii) weight loss iii) abdominal discomfort iv) anaemia v) fever vi) marked splenomegaly.

154.    A baby born at 34 weeks with a heart murmur is kept in the incu- bator for almost 4 weeks. There is no murmur at discharge. What is the likely cause of this murmur?

  1. PDA
  2. TOF
  3. Aneurysm of sinus of Valsalva
  4. Aorto-pulmonary septal defect
  5. AVM
  6. 1. What is the key?
  7. 2. Why it was present in this baby?
  8. 3. If it is present after birth what is the management?

Ans. 1. The key is A. PDA.

Ans. 2. As it is more common in premature baby!

Ans. 3. Management:

  • indomethacin closes the connection in the majority of cases
  • if associated with another congenital heart defect amenable to sur- gery then prostaglandin E1 is useful to keep the duct open until after surgical

155.     A 6yo girl who has previously been well presented with a hx of tonic-clonic seizures lasting 4mins. Her mother brought her to the hospital and she appeared well. She is afebrile and didn’t lose

consciousness during the episode of seizure. She has no neurologic deficit. What is the most appropriate inv for her?

  1. ABG
  2. Serum electrolytes
  3. ECG
  4. Blood glucose
  5. 1. What is the key?
  6. 2. What are the points in favour?

Ans. 1. The key is B. Serum electrolyte.

Ans. 2. In epilepsy patient becomes unconscious. The child has no previous illness and she was conscious during the episode of seizure. So electrolyte imbalance may be the cause.

 

 

156.     A 60yo woman was found by her son. She was confused and had urinary incontinence. She has recovered fully after 6h with no neurological complaints. What is the most likely dx?

  1. Stroke
  2. Vestibular insufficiency
  3. TIA
  4. Intracranial hemorrhage

Ans. The key is C. TIA.

157.     A 34yo woman presents 3 weeks after childbirth. She has had very low mood and has been suffering from lack of sleep. She also has thought of harming her little baby. What is the most appropriate management for this pt?

  1. ECT
  2. CBT
  3. IV haloperidol
  4. Paroxetine
  5. Amitryptiline

Ans. The key is A. ECT. This is wrong key! Correct key is C. IV halo- peridol. [Dx is postpartum psychosis. Patient.info, Wikipedia says antipsychotic and if needed additional mood stabilizer may be used as 1st line. NHS says antidepressant, antipsychotic and mood stabilizer (any one or in combination) as 1st line treatment. As antipsychotic has given priority likely correct answer is C. IV haloperidol. 2nd line is CBT, and last resort is ECT].

158.     A 65yo woman presents with headache. She also complains of dizziness and tinnitus. She has Recently realized she has visual problems. There is hx of burning sensation in fingers and toes. On exam: splenomegaly, itchy after hot bath. Labs: RBC=87, Hgb=31.9, Plt=796. What is the dx?

  1. CML
  2. CLL
  3. Polycythemia vera
  4. Myelofibrosis
  5. NHL
  6. 1. What is the key?
  7. 2. What are the points in favour?

Ans. 1. The key is C. Polycythemia vera.

Ans. 2. Points in favour: i) hyperviscosity symptoms (headache, dizzi- ness, tinnitus, visual problem) ii) pruritus, typically after a hot bath, iii) splenomegaly iv) RBC=87, Hb=31.9, Plt=796.

159.     A 29yo male brought to ED in unconscious state. There is no significant past hx. Which of the following should be done as the initial inv?

  1. CT
  2. Blood glucose
  3. ABG
  4. MRI
  5. CBC

Ans. The key is B. Blood glucose.

 

160.    A 45yo woman comes with red, swollen and exudating ulcer on the nipple and areola of right breast with palpable lump under the ulcer. What do you think is causing this skin condition?

  1. Inflammatory cells releasing cytokines
  2. Infiltration of the lymphatics by the carcinomatous cells
  3. Infiltration of the malignant skin cells to the breast tissue

Ans. The key is B. Infiltration of the lymphatics by the carcinomatous cells.

161.     A 20yo young lady comes to the GP for advice regarding cervical ca. she is worried as her mother past away because of this. She would like to know what is the best method of contraception in her case?

  1. POP
  2. Barrier method
  3. IUCD
  4. COCP
  5. IUS

Ans. The key is A. POP. Probably wrong key! Correct key should be B. Barrier method! [spermatozoa itself acts as a carcinogen!!! So barrier method is the best protection from the given option!!].

162.     A 66yo man, an hour after hemicolectomy has an urine output of 40ml. However, an hour after that, no urine seemed to be draining from the catheter. What is the most appropriate next step?

  1. IV fluids
  2. Blood transfusion
  3. Dialysis
  4. IV furosemide
  5. Check catheter

Ans. The key is E. Check catheter.

163.     A 24yo pt presented with anaphylactic shock. What would be the dose of adrenaline?

  1. 0.5ml of 1:1000
  2. 0.5ml of 1:10000
  3. 1ml of 1:500
  4. 5ml of 1:1000
  5. 0.05ml of 1:100

Ans the key is A. o.5 ml of 1:1000. [in cardiac arrest 1 ml of 1:1000 iv].

164.    A 44yo woman complains of heavy bleeding per vagina. Trans- vaginal US was done and normal. Which of the following would be the most appropriate inv for her?

  1. Hysterectomy
  2. Endometrial biopsy
  3. CBC
  4. High vaginal swab
  5. Coagulation profile

Ans. The key is E. Coagulation profile. [Transvaginal US is normal i.e. no endometrial hyperplasia, no fibroid or obvious cause for heavy bleeding was found. So now most appropriate investigation should be coagfulation profile].

165.     A 60yo woman presented to OPD with dysphagia. No hx of weight loss or heartburn. No change in bowel habits. While doing endoscopy there is some difficulty passing through the LES, but no other abnormality is noted. What is the single most useful inv?

  1. CXR
  2. MRI
  3. Esophageal biopsy
  4. Esophageal manometry
  5. Abdominal XR
  6. 1. What is the key?
  7. 2. What is the diagnosis?
  8. 3. What is the treatment.

Ans. 1. The key is D. Oesophageal manometry.

Ans. 2. Achalasia cardia

Ans. 3. i) Oral medication: Nitrates or CCB ii) Balloon dilatation of the spincter iii) Oesophagomyotomy.

166.     A 24yo woman presents with deep dyspareunia and severe pain in every cycle. What is the initial inv?

  1. Laparoscopy
  2. Pelvic US
  3. Hysteroscopy
  4. Vaginal Swab
  5. 1. What is the key?
  6. 2. What is the likely diagnosis?
  7. 3 What is the treatment?

Ans. 1. The key is B. Pelvic US.

Ans. 2. The likely diagnosis is endometriosis.

Ans. 3. Treatment: There is no cure for endometriosis, but a number of treatments may improve symptoms. This may include pain medi- cation [NSAIDs such as naproxen], hormonal treatments [COCP, or mirena], or surgery [Surgical removal of endometriosis when other measures fail].

167.     A 38yo woman, 10d postpartum presents to the GP with hx of passing blood clots per vagina since yesterday. Exam:

BP=90/40mmhg, pulse=110bpm, temp=38C, uterus tender on pal- pation and fundus 2cm above umbilicus, blood clots +++. Choose the single most likely dx/

  1. Abruption of placenta 2nd to pre-eclampsia
  2. Concealed hemorrhage
  3. Primary PPH
  4. Secondary PPH
  5. Retained placenta
  6. Scabies
  7. 1. What is the key?
  8. 2. How the condition is defined?

Ans. 1. The key is D. Secondary PPH.

Ans. 2. Secondary PPH: Secondary PPH is defined as abnormal or excessive bleeding from the birth canal between 24 hours and 12 weeks postnatally. [www.rcog.org.uk/en/guidelines-research-services/ guidelines/gtg52/].

168.     A 32yo female with 3 prv 1st trimester miscarriages is dx with antiphospholipid syndrome. Anticardiolipin antibodies +ve. She is now 18wks pregnant. What would be the most appropriate manage- ment?

  1. Aspirin
  2. Aspirin & warfarin
  3. Aspirin & heparin
  4. Heparin only
  5. Warfarin only

Ans. The key is C. Aspirin & heparin.

169.     A 23yo presents with vomiting, nausea and dizziness. She says her menstrual period has been delayed 4 weeks as she was stressed recently. There are no symptoms present. What is the next appropri- ate management?

  1. Refer to OP psychiatry
  2. Refer to OP ENT
  3. CT brain
  4. Dipstick for B-hCG
  5. MRI brain
  6. 1. What is the key?
  7. 2. What is the likely diagnosis?

Ans. 1. The key is D. Dipstick for B-hCG.

Ans. 2. Likely diagnosis is pregnancy. [Features like vomiting, nausea and dizziness are consistent with early pregnancy supported by delayed menstruation].

 

170.    A 16yo girl came to the sexual clinic. She complains of painful and heavy bleeding. She says she doesn’t a regular cycle. What is the most appropriate management?

  1. Mini pill
  2. Combined pill
  3. IUS
  4. Anti-prostoglandins
  5. Anti-fibrinolytics

Ans. The key is B. Combined pill.

171.     A 36yo man walks into a bank and demands money claiming he owns the bank. On being denied, he goes to the police station to report this. What kind of delusions is he suffering from?

  1. Delusion of reference
  2. Delusion of control
  3. Delusion of guilt
  4. Delusion of persecution
  5. Delusion of grandeur

Ans. The key is E. Delusion of grandeur.

 

 

172.     Which method of contraception can cause the risk of ectopic pregnancy?

  1. COCP
  2. IUCD
  3. Mirena
  4. POP

Ans. The key is B. IUCD.

173.      A woman has pernicious anemia. She has been prescribed parenteral vitamin B12 tx but she is needle phobic. Why is oral tx not preferred for this pt?

  1. IM B12 is absorbed more
  2. Intrinsic factor deficiency affects oral B12 utilization
  3. IM B12 acts faster
  4. IM B12 needs lower dosage
  5. Pernicious anemia has swallowing difficulties

Ans. The key is B. Intrinsic factor deficiency affects oral B12 utilization.

174.    An old man comes to the doctor complaining that a part of this body is rotten and he wants it removed. What is the most likely dx?

  1. Guilt
  2. Hypochondriasis
  3. Munchausen’s
  4. Nihilism
  5. Capgras syndrome

Ans. The key is D. Nihilism. [nihilism (medical term is nihilistic delu- sion): parts of the body do not exist or are dead]

Guilt: an emotion that occurs when a person feels that they have violated a moral standard.

Hypochondriasis: worry about having a serious illness.

Munchausen’s: a psychiatric factitious disorder wherein those affected feign disease, illness, or psychological trauma to draw attention,sym- pathy, or reassurance to themselves.

Capgras syndrome: a delusion that a friend, spouse, parent, or other close family member (or pet) has been replaced by an identical-look- ing impostor.

  1. A 31yo woman who is 32weeks pregnant attends the antenatal cliniLabs: Hgb=10.7, MCV=91. What is the most appropriate man- agement for this pt?

 

  1. Folate supplement
  2. Ferrous sulphate 200mg/d PO
  3. Iron dextran
  4. No tx req

Ans. The key is D. No tx required. [According to NICE, cut offs for iron supplements:

at booking (8-10 weeks)- if less than 11 at 28 weeks and further- if less than 10.5 if less than these values=> give iron].

 

 

176.     A 47yo man who is a chronic alcoholic with established liver damage, has been brought to the hospital after an episode of heavy drinking. His is not able to walk straight and is complaining of double vision and is shouting obscenities and expletives. What is the most likely dx?

  1. Korsakoff psychosis
  2. Delirium tremens
  3. Wernickes encephalopathy
  4. Tourettes syndrome
  5. Alcohol dependence

Ans. The key is C. Wernicke’s encephalopathy. [triad of i) ophthalmo-

plegia, ii) ataxia iii) confusion].

177.      A 32yo woman of 39wks gestation attends the antenatal day unit feeling very unwell with sudden onset of epigastric pain a/w nausea and vomiting. Temp 36.7C. Exam: RUQ tenderness. Bloods: mild anemia, low plts, elevated LFT and hemolysis. What is the most likely dx?

  1. Acute fatty liver of pregnancy
  2. Acute pyelonephritis
  3. Cholecystitis
  4. HELLP syndrome
  5. Acute hepatitis

Ans. The key is D. HELLP syndrome. [H=hemolysis, EL=elevated liver enzyme, LP=low platelet count].

178.     A 57yo woman presents with dysuria, frequency and urinary incontinence. She complains of dyspareunia. Urine culture has been done and is sterile. What is the most appropriate step?

  1. Oral antibiotics
  2. Topical antibiotics
  3. Topical estrogen
  4. Oral estrogen
  5. Oral antibiotics and topical estrogen

Ans. The key is C. Topical estrogen. [There may be UTI like symptoms and dyspareunia in atrophic vaginitis for which topical oestrogen can be used].

179.      A pt came to the ED with severe lower abdominal pain. Vitals: BP=125/85mmHg, Temp=38.9C. Exam: abdomen rigid, very un- comfortable during par vaginal. She gave a past hx of PID 3 years ago which was successfully treated with antibiotics. What is the appropriate inv?

  1. US
  2. Abdomen XR
  3. CT
  4. High vaginal
  5. Endocervical swab

Ans. The key is A. US. [Patient had previous PID. Current symptoms of severe cervical motion tenderness with significant rise of tempera- ture is very much suggestive of pelvic abscess].

 

180.     A pregnant woman with longterm hx of osteoarthritis came to the antenatal clinic with complaints of restricted joint movement and severe pain in her affected joints. What is the choice of drug?

  1. Paracetamol
  2. Steroid
  3. NSAID
  4. Paracetamol+dihydrocoiene
  5. Pethadine

Ans. The key is A. Paracetamol.

181.      A 24yo 18wk pregnant lady presents with pain in her lower abdomen for the last 24h. She had painless vaginal bleeding. Exam: abdomen is tender, os is closed. What is the most probable dx?

  1. Threatened miscarriage
  2. Inevitable miscarriage
  3. Incomplete miscarriage
  4. Missed miscarriage
  5. Spontaneous miscarriage

Ans. The key is A. Threatened miscarriage. [gestational age 18 weeks, lower abdominal pain, tender abdomen, closed os and painless vaginal bleeding indicates threatened abortion].

182.     A 2yo child playing in the garden had a clean cut. She didn’t have any vaccinations. Also, there is no contraindication to vaccinations. Parents were worried about the vaccine side effects. What will you give?

  1. Clean the wound and dress it
  2. Give TT only
  3. Give DPT only
  4. Give DPT and tetanus Ig
  5. Give complete DPT vaccine course

Ans. The key is E. Give complete DPT vaccine course.

183.      A 32yo female who has had 3 prv miscarriages in the 1st tri- mester now comes with vaginal bleeding at 8wks. US reveals a viable fetus. What would be the most appropriate definitive management?

  1. Admit
  2. Aspirin
  3. Bed rest 2 weeks
  4. Cervical cerclage
  5. No tx

Ans. The key is B. Aspirin. [Early miscarriage is more common in antiphospholipid syndrome and treated with heparin or aspirin when become pregnant].

184.    A 6yo girl started wetting herself up to 6x/day. What is the most appropriate tx?

  1. Sleep alarms
  2. Desmopressin
  3. Reassure
  4. Behavior training
  5. Imipramine

Ans. The given key is B. Desmopressin. This is wrong key. Correct key is D. Behavioral training. [This is not nocturnal enuresis but diurnal enuresis. In diurnal enuresis Desmopressin is not recommended.

Behavioral training is mainstay of treatment in diurnal enuresis].

185.     A 27yo 34wk pregnant lady presents with headache, epigastric pain and vomiting. Exam: pulse=115, BP=145/95mmHg, proteinuria

++. She complains of visual disturbance. What is the best medication for the tx of the BP?

  1. 4g MgSO4 in 100ml 9%NS in 5mins
  2. 2g MgSO4 IV bolus
  3. 5mg hydralazine IV
  4. Methyldopa 500mg/8h PO
  5. No tx

Ans. The given key is A. 4g MgSO4 in 100ml 0.9%NS in 5mins. It is a wrong key. Correct key is E. No tx. [Here, question specifically asked for tx of BP. In case of BP of 145/95 mmHg no treatment for BP is needed. Ref: NICE guideline].

 

 

186.     A 24yo lady who is 37wk pregnant was brought to the ED. Her husband says a few hours ago she complained of headache, visual disturbance and abdominal pain. On arrival at the ED she has a fit. What is the next appropriate management for this pt?

  1. 4g MgSO4 in 100ml 9%NS in 5mins
  2. 2g MgSO4 IV bolus
  3. 2g MgSO4 in 500ml NS in 1h
  4. 4g MgSO4 IV bolus
  5. 10mg diazepam in 500ml 9%NS in 1h

Ans. The key is A. 4g MgSO4 in 100ml 0.9%NS in 5mins [NICE]. [Dx is eclumpsia].

 

 

187.     What is the pathological change in Barret’s esophagitis?

  1. Squamous to columnar epithelium
  2. Columnar to squamous epithelium
  3. Dysplasia
  4. Metaplasia
  1. Hyperplasia

Ans. The key is A. Squamous to columner epithelium. [Both A and D are correct answer as Barrett’s oesophagus is a change known as metaplasia].

188.     A 34yo male presents with hx of headache presents with ataxia, nystagmus and vertigo. Where is the site of the lesion?

  1. Auditory canal
  2. 8th CN
  3. Cerebellum
  4. Cerebral hemisphere
  5. Brain stem

Ans. The key is C. Cerebellum. [Features described are consistent with cerebellar lesion].

 

 

189.     A 24yo girl comes to the woman sexual clinic and seeks advice for contraception. She is on sodium valproate.

  1. She can’t use COCP
  2. She can use COCP with extra precaution
  3. She can use COCP if anticonvulsant is changed to
  4. She can use COCP with estrogen 50ug and progesterone higher dose
  5. She can use COCP

Ans. The key is E. She can use COCP. [sodium valproate has no effect on COCP].

190.     A 27yo lady came to the ED 10 days ago with fever, suprapubic tenderness and vaginal discharge. PID was dx. She has been on the antibiotics for the last 10days. She presents again with lower abdom- inal pain. Temp=39.5C. what is the most appropriate next manage- ment?

  1. Vaginal swab
  2. Endocervical swab
  3. US
  4. Abdominal XR
  5. Laparoscopy

Ans. The key is C. US. [Initial presentation was of PID. But recurrance of symptoms suggests resistant condition like abscess formation].

 

191.     An 18yo man complains of fatigue and dyspnea, he has left parasternal heave and systolic thrill with a harsh pan-systolic mur- mur at left parasternal edge. What is the most probable dx?

  1. TOF
  2. ASD
  3. VSD
  4. PDA
  5. TGA

Ans. The key is C. VSD.

192.     A young girl presenting with fever, headache, vomiting, neck stiffness and photophobia. She has no rashes. What is the most appropriate test to confirm dx?

  1. Blood culture
  2. Blood glucose
  3. LP
  4. CXR
  5. CT

Ans. The key is C. LP. [Case of meningitis. LP will confirm the diagno- sis].

193.      A 65yo HTN man wakes up in the morning with slurred speech, weakness of the left half of his body and drooling. Which part of the brain is affected?

  1. Left parietal lobe
  2. Right internal capsule
  3. Right midbrain
  4. Left frontal lobe

Ans. The key is B. Right internal capsule. [As symptoms are on left side lesion is on right side of the brain. So answer should be either b) right internal capsule or c) right midbrain. If it was midbraine there would have cranial nerve involvement. On the other hand given picture is very much consistent with lacunar infarction of internal capsule!]

  1. A 27yo presents with abdominal pain, bleeding, vomiting and diarrhea. Her LMP was 7wks Exam: abdominal tenderness, BP=90/60mmHg. What is the next appropriate management?
  2. Immediate laparotomy
  3. Laparoscopy
  4. Salpingotomy
  5. Salpingectomy
  6. MTX
  7. 1. What is the key?
  8. 2. What is the diagnosis?
  9. 3. Justify the key.

Ans. 1. The key is A. Immediate laparotomy.

Ans. 2. The diagnosis is ruptured ectopic pregnancy .

Ans. 3. In ruptured ectopic pregnancy if there is shock we should go for immediate laparotomy.

195.      A woman presents with complains of abdominal pain, unsteadi- ness, numbness of lower limb and palpitations. All inv are normal. What is the dx?

  1. Manchausen
  2. Somatization
  3. Hypochondriac
  4. Bipolar

Ans. The key is B. Somatization. [This is multiple, recurrent, medically unexplained symptoms usually starting early in life. Usually patient presents with one symptom at a time. Investigations are normal].

 

 

196.      A 34yo African-caribbean man with a hx of sarcoidosis has presented with bilateral kidney stones. What is the most likely cause for this pt’s stones?

  1. Hypercalcemia
  2. Hyperuricemia
  3. Diet
  4. Recurrent UTIs
  5. Hyperparathyroidism
  6. 1. What is the key?
  7. 2. Why this occurs?

Ans. 1. The key is A. Hypercalcemia.

Ans. 2. Hypercalcemia in sarcoidosis is due to the uncontrolled syn- thesis of 1,25-dihydroxyvitamin D3 by macrophages. 1,25-dihydroxyvi- tamin D3 leads to an increased absorption of calcium in the intestine and to an increased resorption of calcium in the bone.

 

 

197.      Which of the following is NOT a physiological change during pregnancy?

  1. Tidal volume 500ml
  2. RBC vol 64L
  3. Cardiac output 5L/min
  4. Uterus weight 1kg
  5. ESR up by 4x

Ans. The key is A. Tidal volume 500 ml.

198.      A 10yo boy presents with nose bleed. What measure should be taken to stop the bleeding?

  1. Press base of the nose
  2. Ice packs
  3. Press soft parts of the nose
  4. Start tranexemic acid
  5. IV fluids

Ans. The key is C. Press soft parts of the nose.

 

 

199.      An MI pt who is already on aspirin no longer smokes and his cholesterol, ECG, echo and BP are normal. Choose the best option for him:

  1. Give statin
  2. Give statin+warfarin
  3. Low cholesterol diet
  4. Statin+ACEi

Ans. The key is D. Statin + ACEi.

[Offer all people who have had an acute MI treatment with the follow- ing drugs:

ACE (angiotensin-converting enzyme) inhibitor dual antiplatelet therapy (aspirin plus a second antiplatelet agent) beta-blocker sta- tin. [2007, amended 2013] [NICE guideline].

200.     A 46yo man is being treated for a pleural effusion. A chest drain has been sited just below the 4th rib in the mid-axillary line on his right side. What single structure is at particular risk of injury?

  1. Arzygos vein
  2. Diaphragm
  3. Intercostal artery
  4. Internal thoracic artery
  5. Liver

Ans. The key is C. Intercostal artery. [Most vulnerable structure is intercostal nerve, then intercostal artery then intercostals vein. As intercostal nerve is not in option intercostal artery is the answer here].

 

 

201.      What advice would you give for the parents of a child with repeated UTI?

  1. Surgery
  2. Prophylactic antibiotics
  3. Increase fluids
  4. Toilet training
  5. Laxatives

 

Ans. The given key is A. Surgery. This is a wrong key. Correct option is B. Prophylactic antibiotic.[For repeated UTI prophylactic antibiotic should be given].

202.      A pt presents with complete anuria following prolonged hypotension and shock in a pt who bled profusely from a placental abruption. What is the most probable dx?

  1. Post viral infection
  2. Acute papillary necrosis
  3. Acute cortical necrosis
  4. HUS
  5. Renal vein thrombosis
  6. 1. What is the key?
  7. 2. What is the reason for this?

Ans. 1. The key is C. Acute cortical necrosis.

Ans. 2. There are 2 reasons for this acute cortical necrosis. i) signifi- cant diminished arterial perfusion of the kidneys due to spasm of the feeding artery secondary to profuse bleeding from placental abruption

  1. ii) DIC secondary to placental abruption.

203.      An alcoholic 56yo man had ascetic fluid analysis done which was found to be yellow color. What is the most appropriate cause?

  1. Alcoholic hepatitis
  2. Decompensated cirrhosis
  3. TB peritonitis
  4. Pyogenic peritonitis
  5. Neoplasm
  6. 1. What is the key?
  7. 2. How this diagnosis is made?

Ans. 1. The key is B. Decompansated cirrhosis.

Ans. 2. If the patient experiences serious problems described below his disease has progressed from compensated cirrhosis to decompen- sated cirrhosis:

Bleeding varices (internal bleeding) Ascites (fluid in the belly) Encephalopathy (confusion)

Jaundice (yellowing of eyes and skin).

204.   A 15yo boy presents with testicular pain for 2days. There is no hx of trauma. Exam: temp=38.5C, right hemi-scrotum tenderness. What is the single most appropriate management?

  1. Give antibiotics
  2. Give analgesia
  3. Reassure
  4. US scrotum
  5. Exploratory surgery
  6. 1. What is the key?
  7. 2. What is the diagnosis?
  8. 3. What are the points in favour?

Ans. 1. The key is A. Give antibiotics.

Ans. 2. The diagnosis is epididymo-orchitis.

Ans. 3. Points in favour: i) No history of trauma ii) testicular pain with fever points towards epididymo-orchitis.

205.    A 58yo lady presented with urinary incontinence. She looks anx- ious for her condition. Urine culture is sterile. Her urodynamic study is normal. What is the next step?

  1. Antibiotics
  2. Topical estrogen
  3. Systemic estrogen
  4. Duloxetine
  5. Pelvic floor exercise

Ans. The key is E. Pelvic floor exercise.

206.    A 45yo lady came to family planning clinic for contraception ad- vice. She is not keen to be pregnant for the next 3yrs. Her recent US showed multiple small submucosal fibroid. What is the best method of contraception for her?

  1. Etonogestrol
  2. COCP
  3. IUS
  4. POP
  5. IUCD

Ans. The key is C. IUS. [IUS gives 3-5 yrs long contraception. It also helps to shrink the fibroid].

207.    A child presents with eczema. She was given two creams by the GP – emollient and steroid. What advice would you give her regard- ing application of the cream?

  1. Sparingly use both the cream
  2. First use emollient, then steroid
  3. Apply steroid then emollient
  4. Mix emollient & steroid before use
  5. Emollient at night with steroid

Ans. The key is B. First use emollient, then steroid. [emmolient 30 minutes before steroid].

208.    All the following drugs do not cause bronchoconstriction except?

  1. Atenolol
  2. Salbutamol
  3. Salmetrol
  4. Ipratropium bromide
  5. Cocaine

Ans. The key is A. Atenolol.

 

 

209.    A 28 yo female who delivered 6 weeks ago feels sad and has no interest to feeding the baby. She has been eating poorly and having difficulty sleeping. She feels weak throughout the day and

has stopped taking the baby out of the house. She also says that the baby has evil eyes. What is the most likely diagnosis?

  1. Postpartum blues
  2. Postpaetum depression
  3. Postpurtum psychosis
  4. Schizophrenia
  5. Psychotic depression
  6. 1. What is the key?
  7. 2. What are the points in favour?

Ans. 1. The key is C. Postpartum psychosis.

Ans. 2. Points in favour: i) features of depression: feels sad, poor eating, difficulty sleep, feeling weak ii) delusional ideas: thinks baby has evil eyes and not taking the baby out of the house. These points to postpartum psychosis. [Postpartum psychosis starts within 2 wks (occasionally later) of delivery and it can take 6 -12 months or more to recover from postpartum psychosis].

 

 

210.    A 44yo man presents with periorbital and pedal edema. 24h urine shows 8g of protein/d and serum cholesterol=7mmol/L. Renal biopsy results are awaited. What would be the most likely dx?

  1. Minimal change disease
  2. Glomerulonephropathy
  3. Membranous glomerulonephropathy
  4. FSGS
  5. IgA nephropathy
  6. Mesangiocapillary

Ans. The given key is C. Membranous glomerulonephritis. This is wrong key. Correct key is D. FSGS. [FSGS is most common cause of

 

nephrotic syndrome in adults].

211.     A 53yo man presents complaining of weight loss, lethargy, increasing abdominal discomfort and gout for the past yr. Exam: spleen palpated 5cm below left costal margin, no fluid wave. CBC: Hgb=10.5g/dL, WBC=200 – 85% neutrophils, plts=100, Na+=140m- mol/L, K+ 4mmol/L, create=151umol/L, urea=7mmol/L. Serum B12 increased. Philadelphia chromosome +ve. What is the most likely dx?

  1. CML
  2. CLL
  3. AML
  4. ALL
  5. Lymphoma
  6. 1. What is the key?
  7. 2. What are points in favour of this diagnosis?

Ans. 1. The key is A. CML.

Ans. 2. Points in favour: i) wt loss ii) lethargy iii) abdominal discomfort

  1. iv) splenomegaly v) gout [Elevated uric acid and vitamin B12 levels are found in 25% of patients of CML]. A blood picture is suggestive and

+ve Philadelphia chromosome is diagnostic.

 

 

212.    In a group of cancer pts, 10 died that wasn’t treated while 5 died in the tx group. Which statement is correct?

  1. Absolute risk =10
  2. Relative risk =10
  3. Relative risk =5
  4. Absolute risk=5
  5. Relative risk=2

Ans. Given key is E. relative risk = 2. [RR= Number of death in not treated group/number of death in treated group].

213.      A 67yo woman has presented with hard, irregular, poorly defined 5cm lump in her right breast. She has a bruise on the surface and there is no discharge. What is the most likely dx?

  1. Fibroadenosis
  2. Fat necrosis
  3. Fibroadenoma
  4. Duct ectasia
  5. Ca breast
  6. 1. What is the key?
  7. 2. Please justify the key.

Ans. 1. The key is B. Fat necrosis.

Ans. 2. Fat necrosis usually occurs following trauma or surgery. Given case is a fat necrosis of breast as there is no discharge and there is a

bruise indicating prior trauma.

214.    A 67yo female who had undergone a radical mastectomy now comes with the complaint of swelling and redness in her right upper limb. Involvement of which of the following structures explain these symptoms?

  1. Epitrochlear LN
  2. Cephalic vein
  3. Subclavian artery
  4. Axillary group of LN
  5. Long thoracic nerve

Ans. The key is D. Axillary group of LN. [Axillary clearance compromise lymphatic flow and may results in swelling of upper limb].

215.     A 50yo smoker and heavy drinker presents with complaints of racing heart. A 24h ECG comes out normal. What is your next step in management?

  1. ECHO
  2. Reassure
  3. Stress test

Ans. The key is B. Reassure. [Smoking and alcohol excess can cause palpitation without any recognizable arrhythmia and for this no treat- ment is required].

 

 

216.     A 47yo man comes to the GP with a swelling in his left groin which disappears on lying down. The swelling was bluish in color and felt like a bag of worms. He also complains of a mass in the left loin along with hematuria occasionally. What could be the possible dx?

  1. Left sided RCC
  2. Varicosity 2nd to liver disease
  3. Testicular tumor
  4. UTI
  5. IVC obstruction
  6. 1. What is the key?
  7. 2. What is the condition described?
  8. 3. What is the link between these two conditions?

Ans. 1. The key is A. Left sided Renal cell carcinoma.

Ans. 2. Left sided varicocele.

Ans. 3. Most common secondary cause of left sided varicocele is RCC. Newly diagnosed varicocele over the age of 40yrs are very much suggestive of RCC. Varicocele is common on left side as left testicular veins drain to the left renal vein, while the right testicular vein drain directly into IVC.

217.     A man presents with muffled hearing and feeling of pressure in ear with tinnitus and vertigo. He also complains of double vision when looking to the right. What is the most appropriate dx?

  1. Meniere’s disease
  2. Acoustic neuroma
  3. Acute labyrinthytis
  4. Meningioma
  5. Otosclerosis
  6. 1. What is the key?
  7. 2. Justify the key.

Ans. 1. The key is B. Acoustic neuroma.

Ans. 2. Hearing loss, feeling of pressure in the ear with tinnitus, vertigo and involvement of cranial nerve i.e. right abducent nerve are suggestive of acoustic neuroma.

218.     In 85% of the population this artery is dominant. What is the single most appropriate option?

  1. Left ant descending artery
  2. Coronary sinus
  3. Circumflex artery
  4. Left main stem, post descending artery
  5. Right coronary artery
  6. 1. What is the key?
  7. 2. Justify the key.

Ans. 1. The key is E. Right coronary artery.

Ans. 2. If the posterior discending artery is supplied by the circumflex artery then it is left dominant and if posterior descending artery is supplied by the right coronary artery then it is right dominant. As in 85% of population posterior descending artery is supplied by right coronary artery it is called the dominant that is right coronary artery is dominant.

 

 

219.     A 54 yo lady presents with sudden, severe pain in the left half of her skull. She also complains of pain around her jaw. What is the next likely step?

  1. CT
  2. MRI
  3. Fundoscopy
  4. ESR
  5. Temporal artery biopsy
  6. 1. What is the key?
  7. 2. What is the diagnosis?
  8. 3. What are the points in favour of your diagnosis?

Ans. 1. The key is ESR.

 

Ans. 2. The diagnosis is Giant cell arteritis or temporal arteritis.

Ans. 3. Points in favour: i) Age >50yrs ii) Female sex iii) Severe pain in the left half of skull iv) Pain around the jaw (jaw claudication).

220.     A teenage girl who was ‘fine’ until her boyfriend said he didn’t want the relationship anymore. She took 10 tablets of paracetamol in front of his mother after taking alcohol. What should you do?

  1. Refer to psychiatry
  2. Counselling
  3. GP to sort out family issues
  4. Return to work to relieve her anger

Ans. The key is A. Refer to psychiatry. [1o tablets of paracetamol is not a life threatening toxic dose and simultaneous drug overdose and alcohol consumption needs psychiatric evaluation].

221.     A 6yo fell on outstretched hand while playing. He feels tender at the elbow but otherwise well. What is the most likely dx?

  1. Spiral fx
  2. Green stick fx
  3. Compound fx
  4. Supracondylar fx
  5. Pulled elbow

Ans. The key is B. Green stick fracture.

222.     A man has a BP of 160/90mmHg, proteinuria++. KUB US are equally reduced in size with smooth borders and normal pelvic calyceal system. What is the cause of HTN in the pt?

  1. Chronic glomerulonephritis
  2. Chronic pyelonephritis
  3. Bilateral renal artery stenosis
  4. Essential HTN
  5. Polycystic kidney

Ans. The key is bilateral renal artery stenosis. This is probably a wrong key. The correct key should be A. Chronic glomerulonephritis. [In bilateral renal artery stenosis BP is very high (much more than 90/160 mmHg].

 

 

223.      A lady presents with abdominal pain, dysuria, dyspareunia and vaginal discharge. What si your next step?

  1. Laparoscopy
  2. High vaginal swab
  3. Hysteroscopy
  4. Laparotomy
  5. US

Ans. The key is B. High vaginal swab. [Probable diagnosis is PID].

224.    An old lady 72yo staying at a nursing home for a few years, a known HTN on reg tx presented with sudden dysphagia while eating with drooling of saliva and req urgent inv. What would be your next step?

  1. Ba swallow
  2. Chest CT
  3. Endoscopy
  4. Laryngoscopy
  5. CXR
  6. Endoscopy with biopsy

Ans. The key is C. Endoscopy. [Probable impacted food bolus (usually meat)which can be visualized and removed with the aid of endoscopy].

225.     A man presents with outward deviation of his right eye and diplopia. Which nerve is affected?

  1. Left trochlear
  2. Left oculomotor
  3. Right trochlear
  4. Right abducens
  5. Right oculomotor

Ans. The given key is B. Left oculomotor! It is wrong key! Answer should be E. Right oculomotor.

226.     A 60yo pt who has had a MI a week back presents with dyspnea and pericardial rub. ECG shows ST elevation. CXR: loss of margin at costo-vertebral angle. What is the single most likely cause?

  1. Cardiac tamponade
  2. Mitral regurge
  3. Dressler’s syndrome
  4. Atrial fib
  5. Emboli
  6. 1. What is the key?
  7. 2. Why it is not reinfarction as there is ST elevation?

Ans. 1. The key is C. Dressler’s syndrome.

Ans. 2. There is pericardial rub there is pericarditis and in pericarditis there is widespread ST elevation. So the condition is not new MI but Dressler’s syndrome.

227.     A 12yo girl presented with tics, LOC, no residual sign and no post-ictal phase. EEG abnormality in temporal lobe. The girl had a rapid recovery. What is the most probably dx?

  1. Generalized tonic-clonic
  2. Myoclonic
  3. Partialàgeneralized seizure
  4. Atonic seizure
  5. Febrile convulsion

Ans. The key is C. Partial –> generalized seizure

228.     A 48yo woman who has been taking medications for asthma for a long time has now presented with decreasing vision. What is the most probable cause for her decrease in vision?

  1. Inhaled salbutamol
  2. Inhaled steroids
  3. Aminophylline
  4. Beta-blockers
  5. Oral steroids
  6. 1. What is the key?
  7. 2. Justify the key.

Ans. 1. The key is E. Oral steroid.

Ans. 2. Prolonged steroid use leads to cataract formation.

229.     A 34yo man after a car crash is in the ED and deteriorating. His GCS has fallen from 13 to 7. What is the most appropriate next step?

  1. CT
  2. Burr hole
  3. MRI
  4. Intubation
  5. IV fluids

Ans. The key is D. Intubation. [ABC protocol].

230.     A pt with alternating swings or episodes from elation and depression had underwent tx and gotten better. What medication needed to be continued so he can stay well?

  1. Anxiolytics
  2. Mood stabilizers
  3. Antidepressants
  4. Antipsychotics

Ans. The key is B. Mood stabilizers [bipolar disorder treated with mood stabilizers].

 

231.     A 40yo male with pre-existing glumerulonephritis having proteinuria and hematuria suddenly deteriorates and presents with oliguria and serum K+=7.8mmol/L, urea=13mmol/L, creat=342m- mol/L, GFR=19mL/h. The best management would be?

  1. Calcium supplement
  2. Calcium resonate enema 30g
  3. 10units insulin with 50% dextrose
  4. Nebulized salbutamol
  5. 10ml of 10% calcium gluconate
  6. Hemodialysis urgent
  7. 1. What is the key?
  8. 2. Justify the key.

Ans. 1. The key is E. 10 ml of 10% calcium gluconate.

Ans. 2. To prevent cardiac arrhythmia. [Actually calcium gluconate neither shifts K+ to cells nor reduces serum K+ level that much. It just prevents cardiac arrest or life threatening cardiac arrhythmia and buys time till definitive measures are taken].

232.     34yo man was brought to the ED after a RTA. BP=50/0mmHg and chest wall not movingsymmetrically, RR=34bpm. What would be initial action?

  1. IV fluid infusion
  2. Intubation and ventilation
  3. CT chest
  4. Transfer to ITU

Ans. The key is B. Intubation and ventilation [ABC protocol].

233.      A pt complains of SOB, wheeze, cough and nocturnal waking. He has dry scaly shin with rashes that are itchy. What is the single most likely dx?

  1. Scabies
  2. Eczema
  3. Rheumatism
  4. Dermatitis
  5. Psoriasis

Ans. The key is B. Eczema. [Asthma may be associated with atopy].

234.    A 54yo woman has presented with episodes of abdominal ache, vomiting and postural hypotension. She also has a dark pigmen- tation of her skin. A dx of Addison’s disease was made. What is the most likely electrolyte abnormality expected in this pt?

  1. High Na+, Low K+
  2. Low Na+, High K+
  3. Low Na+, Low K+
  4. High Na+, High K+
  5. Low Na+, Normal K+

Ans. The key is B. Low Na+, High K+. [ with Addison disease, the sodium, chloride, and carbon dioxide levels are often low, while the potassium level is high].

 

 

235.     An 8yo returned from Spain with severe pain in one ear. Exam: pus in auditory canal, tympanic membrane looks normal. What is the tx option?

  1. Gentamicin topical
  2. Amoxicillin PO
  3. Analgesia
  4. Amoxicillin IV
  5. 1. What is the key?
  6. 2. What is the diagnosis?

Ans. 1. The key is A. Gentamycin topical.

Ans. 2. Diagnosis is otitis externa.

236.     A 6wk child is very sick-looking. Bloods: Na+=124, K+=2.8. Dehy- drated. What would you choose to resuscitate?

  1. 18% NS + 4% dextrose + 20mmol KCl
  2. 9% NS
  3. 0.45% NS
  4. 0.45% NS + 5% dextrose
  5. 0.45% NS + 5% dextrose + 20 mmol KCl

Ans. The given key is E. But it is wrong key! The correct key is B. 0.9% NS. Explanation: Rsuscitation is mostly done with 0.9% NS or ringers lactate, or hartmans solution. Here is hypokalemia. To treat hypoka- laemia the cut off value is below 2.5 mmol/L and absence of anuria during resuscitation. Maintenance is with fluid E.

237.      A 68yo man gets repeated attacks of LOC and TIA. What is the most likely cause for this?

  1. Atrial fib
  2. Mitral stenosis
  3. Aortic stenosis
  4. HOCM
  5. Carotid artery stenosis

Ans. The key is E. Carotid artery stenosis.

238.      Pt presented with hemoptysis 7d post-tonsillectomy. What is the next step?

  1. Packing
  2. Oral antibiotics + discharge
  3. Admit + IV antibiotics
  4. Return to theatre and explore
  5. Ice cream and cold fluids

Ans. The key is C. Admit + IV antibiotic. [infection is a common cause of secondary haemorrhage. Patient should be admitted to observe the course of bleeding and treatment is given with IV antibiotics].

239.      A child was admitted following a RTA with initial GCS=15. Then during the night the noticed GCS reduced to 13. What is the manage- ment?

  1. Refer to neuro-surgeon
  2. IV fluids
  3. Oxygen
  4. CT brain
  5. Skull XR

Ans. The key is D. CT brain. [probable intracranial haemorrhage].

240.     A 57yo woman who is suffering from HTN, presented to the hospital with complaints of recurrent falls when trying to get out of bed or getting up from sitting. She is on some anti-HTN therapy with no other med prbs. What is the cause of her fall?

  1. CCB
  2. Vertibrobasiliar insufficiency
  3. Thiazide
  4. Hypoglycemia
  5. Infection

Ans. The key is C. Thiazide. [It causes postural hypotension by volume depletion].

241.      A 56yo woman with MS presents with drooping of the left side of her lips. She also has loss of sensation over her face, hearing im- pairment and some in-coordination of her movements. What is the most likely anatomical site affected?

  1. Cerebellum
  2. Cerebrum
  3. Spinal cord
  4. Brain stem
  5. Optic nerve

Ans. The key is D. Brain stem. Features of 5, 7, 8th cranial nerve and cerebellum involvement suggestive of brainstem lesion.

 

242.     A 68yo male presented with swelling in the lower pole of the parotid gland for the last 10yrs. Exam: firm in consistency. What’s the most probable dx?

  1. Pleomorphic adenoma
  2. Adenolymphoma
  3. Mikulicz’s disease
  4. Parotiditis
  5. Frey’s syndrome

Ans. The key is A. Pleomorphic adenoma. [Pleomorphic adenoma (most common) – also called benign mixed tumour: is the most com- mon tumour of the parotid gland and causes over a third of subman- dibular tumours. They are slow-growing and asymptomatic, having a malignant potentiality].

 

 

243.      A 28yo shipyard worker was admitted for pain in calf while at work which has been increasing over the last 3m. There is no hx of HTN or DM but he is a smoker. Exam: loss of posterior tibial and dorsalis pedis pulsation along with a non-healing ulcer at the base of the right 1st MTP joint. What is the most probably dx?

  1. Thromboangitis obliterans
  2. Sciatica
  3. DVT
  4. Baker’s cyst
  5. Embolus
  6. 1. What is the key?
  7. 2. What are the points in favour?

Ans. The key is A. Thromboangitis obliterans.

Ans. 2. i) young age ii) smoker iii) pain in cuff iv) loss of posterior tibial and dorsalis pedis pulsation v) non-healing ulcer at the base of the right 1st MTP joint all are suggestive of Buerger’s disease.

244.    A 35yo lady presents with painful ulcers on her vulva, what is the appropriate inv which will lead to the dx?

  1. Anti-HSV antibodies
  2. Dark ground microscopy of the ulcer
  3. Treponema palladium antibody test
  4. Rapid plasma regain test
  5. VDRL

Ans. The key is A. Anti-HSV antibodies. [Genital Herpes may be as- ymptomatic or may remain dormant for months or even years. When symptoms occur soon after a person is infected, they tend to be severe. They may start as multiple small blisters that eventually break open and produce raw, painful sores that scab and heal over within

a few weeks. The blisters and sores may be accompanied by flu-like symptoms with fever and swollen lymph nodes.

There are three major drugs commonly used to treat genital herpes symptoms: acyclovir (Zovirax), famciclovir (Famvir), and valacyclo- vir(Valtrex). These are all taken in pill form. Severe cases may be treated with the intravenous (IV) drug acyclovir].

245.     A 53yo man presents with a longstanding hx of a 1cm lesion on his arm. It has started bleeding on touch. What is the most likely dx?

  1. Basal cell carcinoma
  2. Kaposi’s sarcoma
  3. Malignant melanoma
  4. Squamous cell carcinoma
  5. Kerathoacanthoma

Ans. The key is D. Squamous cell carcinoma. [SSCs Arises in squa- mous cells. SCCs may occur on all areas of the body including the mucous membranes and genitals, but are most common in areas frequently exposed to the sun, such as the rim of the ear, lower lip, face, balding scalp, neck, hands, arms and legs. SCCs often look like scaly red patches, open sores, elevated growths with a central depres- sion, or warts; they may crust or bleed. A tissue sample (biopsy) will be examined under a microscope to arrive at a diagnosis. Squamous cell carcinomas detected at an early stage and removed promptly are almost always curable and cause minimal damage].

246.     A 47yo man with hx of IHD complains of chest pain with SOB on exertion over the past few days. ECG normal, Echo= increased EF and decreased septal wall thickness. What is the most likely dx?

  1. Dilated CM
  2. Constrictive pericarditis
  3. Amyloidosis
  4. Subacute endocarditis

Ans. The key is A. Dilated CM. [In dilated cardiomyopathy ejection fraction is decreased (but here increased which goes in favour of constrictive pericarditis). On the other hand decreased septal wall thickness favours the diagnosis of dilated cardiomyopathy. So it seems to be a bad recall!!].

247.     An elderly pt who is known to have DM presents to the hospital with drowsiness, tremors and confusion. What inv should be done to help in further management?

  1. Blood sugar
  2. ECG
  3. Standing and lying BP
  4. Fasting blood sugar
  5. CT

Ans. The key is A. Blood sugar.

248.     A 28yo pregnant woman with polyhydramnios and SOB comes for an anomaly scan at 31 wks. US= absence of gastric bubble. What is the most likely dx?

  1. Duodenal atresia
  2. Esophageal atresia
  3. Gastrochiasis
  4. Exomphalos
  5. Diaphragmatic hernia

Ans. The key is B. Oesophageal atresia.

249.     A 1m boy has been brought to the ED, conscious but with cool peripheries and has HR=222bpm. He has been irritable and feeding poorly for 24h. CXR=borderline enlarged heart with clear lung fields. ECG=regular narrow complex tachycardia, with difficulty identifying p wave. What is the single most appropriate immediate tx?

  1. Administer fluid bolus
  2. Administer oxygen
  3. Oral beta-blockers
  4. Synchronized DC cardio-version
  5. Unilateral carotid sinus massage
  6. 1. What is the key?
  7. 2. Justify the key.
  8. 3. What is the diagnosis?

Ans. 1. The key is D. Synchrnized DC cardioversion.

Ans. As the patient is in probable hemodynamic instability (suggested by cool peripheries due to low BP or inadequate CO) so we should go for DC cardioversion.

Ans. 3. Probable diagnosis is SVT.

250.     A 7yo child presented with chronic cough and is also found to be jaundiced on examination. What is the most likely dx?

  1. Congenital diaphragmatic hernia
  2. Congenital cystic adenematoid malformation
  3. Bronchiolitis
  4. RDS
  5. Alpha 1 antitrypsin deficiency
  6. 1. What is the key?
  7. 2. Justify the key.

Ans. 1. The key is E. Alpha 1 antitrypsin deficiency.

Ans. 2. Unexplained liver disease with respiratory symptoms are very suggestive of AATD.

 

251.     A 35yo construction worker is dx with indirect inguinal hernia. Which statement below bestdescribes it?

  1. Passes through the superficial inguinal ring only
  2. Lies above and lateral to the pubic tubercle
  3. Does not pass through the superficial inguinal ring
  4. Passes through the deep inguinal ring

Ans. The key is D. Passess through the deep inguinal ring.

252.     A woman has numerous painful ulcers on her vulva. What is the cause?

  1. Chlamydia
  2. Trichomonas
  3. Gardenella
  4. HSV
  5. EBV

Ans. The key is D. HSV.

253.      A 72 yo man has been on warfarin for 2yrs because of past TIA and stroke. What is the most important complication that we should be careful with?

  1. Headache
  2. Osteoporosis
  3. Ear infection
  4. Limb ischemia
  5. Diarrhea

Ans. The given key is E. Diarrhoea which is considered as a wrong key and A. Headache is the correct key. [Headache is the warning sign of hemorrhagic stroke].

254.    A 55yo man has been admitted for elective herniorraphy. Which among the following can be the reason to delay his surgery?

  1. Controlled asthma
  2. Controlled atrial fib
  3. DVT 2yrs ago
  4. Diastolic BP 90mmHg
  5. MI 2 months ago

Ans. The key is E. MI 2 months ago [better go for surgery 6 months post MI as surgery before this has higher mortality rate].

255.     A 65yo known case of liver ca and metastasis presents with gastric reflux and bloatedness. On bone exam there is osteoporosis. He also has basal consolidation in the left lung. What is the vnext appropriate step?

  1. PPI IV
  2. Alendronate
  3. IV antibiotics
  4. Analgesic
  5. PPI PO

Ans. Here is two key C. IV antibiotics and E. PPI PO. Correct key is C. IV antibiotics. [Pneumonia should be treated first].

256.     A 66yo man has the following ECG. What is the most appropri- ate next step in management?

 

 

  1. Metoprolol
  2. Digoxin
  3. Carotid sinus massage
  4. Adenosine
  5.  

Ans. The key is A. Metoprolol. [P waves are replaced by fibrillatory f-waves. Irregular R-R intervals. Dx atrial fibrillation].

257.      A 22yo sexually active male came with 2d hx of fever with pain in scrotal area. Exam: scrotal skin is red and tender. What is the most appropriate dx?

  1. Torsion of testis
  2. Orchitis
  3. Inguinal hernia
  4. Epididymo-orchitis
  5. 1. What is the key?
  6. 2. How will you differentiate torsion from epididymo-orchitis?

Ans. 1. The key is D. Epididymo-orchitis.

Ans 2. In epididymo-orchitis there should be fever, elevation of testes reduces pain (positive prehn sign), In torsion testis lies at a higher level. In torsion urinalysis negative but in epididymo-orchitis it is posi- tive. Epididymo-orchitis usually occurs in sexually active man.

258.     A man on warfarin posted for hemicolectomy. As the pt is about to undergo surgery. What option is the best for him?

  1. Continue with warfarin
  2. Continue with warfarin and add heparin
  3. Stop warfarin and add aspirin
  4. Stop warfarin and add heparin
  5. Stop warfarin

Ans. The key is D. Stop warfarin and add heparin.

259.     A 65yo known alcoholic is brought into hospital with confusion, aggressiveness and ophthalmoplegia. He is treated with diazepoxide. What other drug would you like to prescribe?

  1. Antibiotics
  2. Glucose
  3. IV fluids
  4. Disulfiram
  5. Vit B complex

Ans. The key is E. Vitamin B complex.

260.    A 32yo woman has severe right sided abdominal pain radiating into the groin which has lasted for 3h. She is writhering in pain. She has no abdominal signs. What is the most likely cause of her abdom- inal pain?

  1. Appendicitis
  2. Ruptured ectopic pregnancy
  3. Salpingitis
  4. Ureteric colic
  5. Strangulated hernia
  6. 1. What is the key?
  7. 2. Abdominal pain radiating to groin, at which level of stone does it indicate?

Ans. 1. The key is D. Ureteric colic.

Ans. 2. It indicate stone at lower ureter. [i) Pain from upper ureteral stones tends to radiate to the flank and lumbar areas. ii) Midureteral calculi cause pain that radiates anteriorly and caudally. This miduret- eral pain in particular can easily mimic appendicitis on the right or acute diverticulitis on the left. iii) Distal ureteral stones cause pain that tends to radiate into the groin or testicle in the male or labia majora in the female.

 

261.      A 39yo coal miner who smokes, drinks and has a fam hx of blad- der cancer is suffering from BPH. The most important risk factor for his bladder carcinoma is?

  1. Fam hx
  2. Smoking
  3. Exposure to coal mine
  4. BPH
  5. 1. What is the key?
  6. 2. What are the risk factors for bladder cancer?

Ans. 1. The key is B. smoking.

Ans. 2. Risk factors of bladder cancer: i) Smoking ii) Exposure to chemicals used in dye industry iii) Whites are more likely to develop bladder cancer iv) Risk increases with age v) More common in men vi) Chronic bladder irritation and infections (urinary infections, kidney and bladder stones, bladder catheter left in place a long time.) vii) Personal history of bladder or other urothelial cancer viii) Family history ix) Che- motherapy or radiotherapy x) Pioglitazone for more than one year and certain herb xi) Arsenic in drinking water xii) Low fluid consumption.

262.     A 34yo woman is referred to the endocrine clinic with a hx of thyrotoxicosis. At her 1st appointment she is found to have a smooth goiter, lid lag and bilateral exophthalmos with puffy eyelids and conjunctival injection. She wants to discuss the tx of her thyroid prb as she is keen to become pregnant. What is the most likely tx you would advise?

  1. 18m of carbimazole alone
  2. 18m of PTU (propylthyouracil) alone
  3. A combo od anti-thyroid drug an0d thyroxine
  4. Radioactive iodine
  5. Thyroidectomy
  6. 1. What is the key?
  7. 2. What is the reason?

Ans. 1. The key is B. 18m of PTU alone.

Ans. 2. Other drug option i.e Carbimazole is teratogenic [can cause i) spina bifida ii) cardiovascular malformations, hypospadius etc] if be- come pregnant. PTU is on the other hand relatively safe in pregnancy.

263.      A child living with his stepfather is brought by the mother with multiple bruises, fever and fractures. What do you suspect?

  1. NAI
  2. Malnutrition
  3. Thrombocytopenia
  4. HIV

Ans. The key is A. NAI. [H/O living with stepfather, multiple bruises, fever and fractures are suggestive of NAI].

264.    A young man who was held by the police was punched while in custody. He is now cyanosed and unresponsive. What is the 1st thing you would do?

  1. IV fluids
  2. Clear airway
  3. Turn pt and put in recovery position
  4. Give 100% oxygen
  5. Intubate and ventilate

Ans. The key is B. Clear airway. [ABC protocol].

265.     A HTN male loses vision in his left eye. The eye shows hand movement and a light shined in the eye is seen as a faint light. Fun- dus exam: flame shaped hemorrhages. The right eye is normal. What is the cause of this pts unilateral blindness?

  1. HTN retinopathy
  2. CRA thrombosis
  3. CRV thrombosis
  4. Background retinopathy
  5. Retinal detachmen

Ans. The key is A. HTN retinopathy [it is a wrong key. Correct key is C. central retinal vein thrombosis].

266.     A mentally retarded child puts a green pea in his ear while eat- ing. The carer confirms this. Otoscopy shows a green colored object in the ear canal. What is the most appropriate single best approach to remove this object?

  1. By magnet
  2. Syringing
  3. Under GA
  4. By hook
  5. By instilling olive oil

Ans. The key is Under GA. [Pea is not a magnetic material and hence it cannot be removed by magnet, it will swell up if syringing is attempted, as hook placement is likely with risk of pushing the pea

deeper it is not also suitable in a mentally retarded child, and olive oil is not of help in case of pea. So to avoid injury it is better to remove under GA].

267.     A pt presents with longstanding gastric reflux, dysphagia and chest pain. On barium enema, dilation of esophagus with taper- ing end is noted. He was found with Barrett’s esophagus. He had

progressive dysphagia to solids and then liquids. What is the single most appropriate dx?

  1. Achalasia
  2. Esophageal spasm
  3. GERD
  4. Barrett’s esophagus
  5. Esophageal carcinoma

Ans. The key is E. Oesophageal carcinoma. [there is dilatation in oesophagus which is seen both in achalasia and carcinoma. Dyspha- gia to solid initaially is very much suggestive of carcinoma and also barrett’s change is a clue to carcinoma]

268.     A 48yo lady presents with itching, excoriations, redness, bloody discharge and ulceration around her nipple. What is the most likely dx?

  1. Paget’s disease of the breast
  2. Fibrocystic dysplasia
  3. Breast abscess
  4. Duct papilloma
  5. Eczema

Ans. The key is A. Paget’s disease of the breast.

 

269.      Pt with widespread ovarian carcinoma has bowel obstruction and severe colic for 2h and was normal in between severe pain for a few hours. What is the most appropriate management?

  1. PCA (morphine)
  2. Spasmolytics
  3. Palliative colostomy
  4. Oral morphine
  5. Laxatives
  6. 1. What is the key?
  7. 2. Why we should go for this option?

Ans. 1. The key is C. Palliative colostomy.

Ans. 2. Cancer or chemotherapy induced obstructions are unlikely to respond to conservative management [NBM, IV fluid, nasogastric suction] and hence only analgesia will not relieve it. So in such cases we have to go for palliative colostomy.

 

270.    A 70yo man admits to asbestos exposure 20yrs ago and has attempted to quit smoking. He has noted weight loss and hoarse- ness of voice. Choose the single most likely type of cancer a.w risk factors present.

  1. Basal cell carcinoma
  2. Bronchial carcinoma
  3. Esophageal carcinoma
  4. Nasopharyngeal carcinoma
  5. Oral carcinoma
  6. 1. What is the key?
  7. 2. What are the conditions related to asbestos exposure?

Ans. 1. The key is B. Bronchial carcinoma. [Asbestos exposure is a risk factor for lung cancer and also has a synergistic effect with cigarette smoke. Horseness can be from involvement of recurrent laryngeal nerve].

Ans. 2. Conditions related to asbestos exposure: i) Pleural plaques (after a latent period of 20-40 yrs) ii) Pleural thickening iii) Asbestosis (latent period is typically 15-30 yrs) iv) Mesothelioma (prognosis is very poor) v) Lung cancer.

271.     A 32yo woman had progressive decrease in vision over 3yrs. She is now dx as almost blind. What would be the mechanism?

  1. Cataract
  2. Glaucoma
  3. Retinopathy
  4. Uveitis
  5. Keratitis
  6. 1. What is the key ?
  7. 2. Why you made this diagnosis?

Ans. 1. The key is B. Glaucoma. This is wrong key! Correct option is retinopathy.

Ans. 2. Cataract is unlikely at this age. Uveitis and iritis doesn’t have such degree of vision loss and iritis and anterior uveitis have pain, redness and photophobia. Open angle glaucoma mostly occurs after the age of 50yrs. Answer should be retinopathy (example retinitis pigmentosa).

272.     A child during operation and immediately after showed glycos- uria, but later his urine sugar was normal. Choose the most probable dx.

  1. Pre-diabetic state
  2. Normal finding
  3. Low renal tubular threshold
  4. DM
  5. 1. What is the key?
  6. 2. Why glycosuria occurred?

Ans. 1. The key is B. Normal finding.

Ans. 2. Stress during operation can cause transient hyperglycemia causing glycosuria secondary to stress induced rise of cortisole which becomes normal after some time.

273.      A pt presented with hx of swelling in the region of the sub-man- dibular region, which became more prominent and painful on chew- ing. He also gave hx of sour taste in the mouth, the area is tender on palpation. Choose the most probable dx?

  1. Chronic recurrent sialadenitis
  2. Adenolymphoma
  3. Mikulicz’s disease
  4. Adenoid cystic carcinoma
  5. Sub-mandibular abscess

Ans. The key is A. Chronic recurrent sialadenitis. [pain, swelling, more pain on chewing, tenderness, and submandibular region suggests diagnosis of submandibular chronic recurrent sialadenitis, usually secondary to sialolithiasis or stricture].

274.     ECG of an 80yo pt of ICH shows saw-tooth like waves, QRS complex of 80ms duration, ventricular rate=150/min and regular R-R interval. What is the most porbable dx?

  1. Atrial fib
  2. Atrial flutter
  3. SVT
  4. Mobitz type1 second degree heart block
  5. Sinus tachycardia

Ans. The key is B. Atrial flutter. [Saw-tooth like waves, normal QRS complex of 80 ms (normal range 70-100 ms), ventricular rate of 150/ min and regular R-R interval is diagnostic of atrial flutter].

275.     A 50 yo woman who was treated for breast cancer 3 yrs ago now presents with increase thirst and confusion. She has become drowsy now. What is the most likely metabolic abnormality?

  1. Hypercalcemia
  2. Hyperkalemia
  3. Hypoglycemia
  4. Hyperglycemia
  5. Hypocalcemia
  6. 1. What is the key?
  7. 2. Justify the key

Ans. 1. The key is A. Hypercalcemia.

Ans. 2. Increased thirst, confusion, drowsiness these are features of hypercalcemia. Any solid organ tumour can produce hypercalcemia. Here treated Ca breast is the probable cause of hypercalcemia.

276.      A 29yo woman presents to her GP with a hx of weight loss, heat intolerance, poor conc and palpitations. Which of the following is most likely to be a/w dx of thyroiditis a/w viral infection?

  1. Bilateral exophthalmos
  2. Diffuse, smooth goiter
  3. Reduced uptake on thyroid isotope scan
  4. Positive thyroid peroxidase antibodies
  5. Pretibial myxedema
  6. 1. What is the key?
  7. 2. What is the diagnosis?
  8. 3. Whats are the points in favour?

Ans. 1. The key is C. Reduced uptake on thyroid isotope scan.

Ans. 2. The diagnosis is De Quervain’s or subacute thyroiditis.

Ans. 3. Viral or subacute thyroiditis: diagnostic criteria: i) Features of hyperthyroidism present. ii) Pain thyroid, not mentioned. iii) Investiga- tions: high esr (60-100) not mentioned, Reduced uptake of radioac- tive iodine by the gland.

277.      A lady, post-colostomy closure after 4days comes with fluctu- ating small swelling in the stoma. What is the management option for her?

  1. Local exploration
  2. Exploratory laparotomy
  3. Open laparotomy
  4. Reassure

Ans. The key is A. Local exploration.

278.     A 65yo female pt was given tamoxifen, which of the following side effect caused by it will concern you?

  1. Fluid retention
  2. Vaginal bleeding
  3. Loss of apetite
  4. Headache and dizziness
  5. Anorgasm
  6. 1. What is the key?
  7. 2. What is the reason to select this key?

Ans. 1. The key is B. Vaginal bleeding.

Ans. 2. Tamoxifen can promote development of endometrial carcino- ma. So vaginal bleeding will be of concern for us.

 

279.      A 39yo man with acute renal failure presents with palpitations. His ECG shows tall tented T waves and wide QRS complex. What is the next best step?

  1. Dialysis
  2. IV calcium chloride
  3. IV insulin w/ dextrose
  4. Calcium resonium
  5. Nebulized salbutamol
  6. 1. What is the key?
  7. 2. What is the diagnosis?
  8. 3. What is the significant of tall tented T waves and wide QRS complex?

Ans. 1. The key is B. IV calcium chloride (both IV calcium gluconate or IV calcium chloride can be used when there is ECG changes).

Ans. 2. The ECG changes are suggestive of Hyperkalemia.

Ans. 3. At potassium level of >5.5mEq/L occurs tall tented T waves and at potassium level >7mEq/L occurs wide QRS complex with bizarre QRS morphology.

 

 

280.     A 54yo pt 7 days after a total hip replacement presents with acute onset breathlessness and raised JVP. Which of the following inv will be most helpful in leading to a dx?

  1. CXR
  2. CTPA
  3. V/Q scan
  4. D-Dimer
  5. Doppler US of legs
  6. 1. What is the key?
  7. 2. Justify the key.

Ans. 1. The key is B. CTPA.

Ans. 2. The patient has a +ve two level PE Wells score (if it was negative we should do D-Dimer) and there is no renal impairment or history suggestive of allergy to contrast media (if these present we should have go for VQ scan) the investigation of choice is PTCA. NICE guideline.

281.      A 7yo girl has been treated with penicillin after sore throat, fever and cough. Then she develops skin rash and itching. What is the most probable dx?

  1. Erythema nodosum
  2. Erythema multiforme
  3. SJS
  4. Erythema marginatum
  5. Erythema gangernosum
  1. What is the key?
  2. What common drugs causes this to occur?

Ans. 1. The key is B. Erythema multiforme.

Ans. 2. Common drugs causing erythma multiforme are: antibiotics (including, sulphonamides, penicillin), anticonvulsants (phenytoin,bar- biturates), aspirin, antituberculoids, and allopurinol.

282.     A 60yo man presented with a lump in the left supraclavicular region. His appetite is decreased and he has lost 5kg recently. What is the most probably dx?

  1. Thyroid carcinoma
  2. Stomach carcinoma
  3. Bronchial carcinoma
  4. Mesothelioma
  5. Laryngeal carcinoma

Ans. The key is B. Stomach carcinoma. [Mentioned lump in the left supraclavicular region is Vershow’s gland, has long been regarded as strongly indicative of the presence of cancer in the abdomen, specifi- cally gastric cancer].

 

 

283.      A 64yo man has presented to the ED with a stroke. CT shows no hemorrhage. ECG shows atrial fib. He has been thrombolysed and he’s awaiting discharge. What prophylactic regimen is best for him?

  1. Warfarin
  2. Heparin
  3. Aspirin
  4. Statins
  5. Beta blockers

Ans. The key is A. Warfarine. [Atrial fibrillation: post stroke- following a stroke or TIA warfarine should be given as the anticoagulant of choice. NICE guideline].

284.    A 54yo man after a CVA presents with ataxia, intention tremors and slurred speech. Which part of the brain has been affected by the stroke?

  1. Inner ear
  2. Brain stem
  3. Diencephalon
  4. Cerebrum
  5. Cerebellum
  6. 1. What is the key?
  7. 2. What are the features of the condition?

Ans. 1. The key is E. Cerebellum.

Ans. 2. i) Ataxia ii) slurred speech or dysarthria iii) dysdiodokokinesis

  1. iv) intention tremor v) nystagmus.

285.     A 57yo man with blood group A complains of symptoms of vom- iting, tiredness, weight loss and palpitations. Exam: hepatomegaly, ascites, palpable left supraclavicular mass. What is the most likely dx?

  1. Gastric carcinoma
  2. Colorectal carcinoma
  3. Peptic ulcer disease
  4. Atrophic gastritic
  5. Krukenburg tumor

Ans. The key is A. Gastric carcinoma. [i) blood group A is associated with gastric cancer ii) vomiting, tiredness, weight loss are general features of gastric cancer iii) palpitation from anemia of cancer iv) hepatomegaly (metastasis) and ascites are late features of gastric cancer. v) palpable left supraclavicular mass- is Vershow’s gland, has long been regarded as strongly indicative of gastric cancer].

286.     A 21yo girl looking unkempt, agitated, malnourished and ner- vous came to the hospital asking for painkillers for her abdominal pain. She is sweating, shivering and complains of joint pain. What can be the substance misuse here?

  1. Alcohol
  2. Heroin
  3. Cocaine
  4. LSD
  5. Ecstasy

Ans. The key is B. Heroin. [agitation, nervousness, abdominal cramp, sweating, shivering and piloerection, arthralgia these are features of heroin withdrawal].

287.      A child presents with increasing jaundice and pale stools. Choose the most appropriate test?

  1. US abdomen
  2. Sweat test
  3. TFT
  4. LFT
  5. Endomyseal antibodies

Ans. The key is A. US abdomen. [This is a picture suggestive of obstructive jaundice. LFT can give clue like much raised bilirubin, AST and ALT not that high and raised alkaline phosphatase but still USG is diagnostic in case of obstructive jaundice].

 

288.     A 32yo man presents with hearing loss. AC>BC in the right ear after Rhine test. He also complains of tinnitus, vertigo and numb- ness on same half of his face. What is the most appropriate inv for his condition?

  1. Audiometry
  2. CT
  3. MRI
  4. Tympanometry
  5. Weber’s test

Ans. The key is C. MRI. [features are suggestive of acaustic neuroma, so MRI is the preferred option].

289.     A 56 yo lady with lung cancer presents with urinary retention, postural hypotension, diminished reflexes and sluggish pupillary reaction. What is the most likely explanation for her symptoms?

  1. Paraneoplastic syndrome
  2. Progression of lung cancer
  3. Brain metastasis
  4. Hyponatremia
  5. Spinal cord compression

Ans. The key is A. Paraneoplastic syndrome. [Features given are well known features of autonomic neuropathy which can be a result of paraneoplastic syndrome].

290.     An old woman having decreased vision can’t see properly at night. She has changed her glasses quite a few times but to no ef- fect. She has normal pupil and cornea. What is the most likely dx?

  1. Cataract
  2. Glaucoma
  3. Retinal detachment
  4. Iritis
  5. GCA

Ans. The key is B. Glaucoma. It is a wrong key. Correct key should be A. Cataract. [Age and normal pupil and cornea are suggestive of

cataract. If it was glaucoma pupil would be a bit dilated and/or oval in shape].

 

 

291.      A pt comes with sudden loss of vision. On fundoscopy the optic disc is normal. What is the underlying pathology?

  1. Iritis
  2. Glaucoma
  3. Vitreous chamber
  4. Retinal detachment
  5. 1. What is the key?
  6. 2. What are the causes of sudden painless loss of vision?

Ans. 1. The Key is D. Retinal detachment. [At an early stage optic disc remains normal in retinal detachment].

Ans. 2. Causes of sudden painless loss of vision:

Retinal detachment Vitreous haemorrhage Retinal vein occlusion Retinal artery occlusion Optic neuritis Cerebrovascular accident

292.     A child was woken up from sleep with severe pain in the testis. Exam: tenderness on palpation and only one testis was normal in size and position. What would be your next step?

  1. Analgesia
  2. Antibiotics
  3. Refer urgently to a surgeon
  4. Reassurance
  5. Discharge with analgesics

Ans. The key is A. Analgesia. [According to some US sites it is analge- sia but no UK site support this! So for Plab exam the more acceptable option is C. Refer urgently to a surgeon].

293.       A child suffering from asthma presents with Temp 39C, drool- ing saliva on to the mother’s lap, and taking oxygen by mask. What sign will indicate that he is deteriorating?

  1. Intercostal recession
  2. Diffuse wheeze
  3. Drowsiness

Ans. The key is A. Intercostal recession. This is wrong key. Correct key is C. Drowsiness. [Intercostal recession is a sign of severe asthma but it can be seen at a lesser degree as well. So drowsiness is more appropriate answer].

294.     A 12yo boy presents with painful swollen knee after a sudden fall. Which bursa is most likely to be affected?

  1. Semimembranous bursa
  2. Prepatellar bursa
  3. Pretibial bursa
  4. Suprapatetaller bursa

Ans. The key is B. Prepatellar bursa. [A fall onto the knee can damage the prepatellar bursa. This usually causes bleeding into the bursa sac causing swellen painful knee. Prepatellar bursitis that is caused by

an injury will usually go away on its own. The body will absorb the blood in the bursa over several weeks, and the bursa should return to normal. If swelling in the bursa is causing a slow recovery, a needle

may be inserted to drain the blood and speed up the process. There is a slight risk of infection in putting a needle into the bursa].

295.      A 61yo man has been referred to the OPD with frequent episodes of breathlessness and chest pain a/w palpitations. He has a regular pulse rate=60bpm. ECG=sinus rhythm. What is the most appropriate inv to be done?

  1. Cardiac enzymes
  2. CXR
  3. ECG
  4. Echo
  5. 24h ECG

Ans. The key is E. 24h ECG.

Indications of 24 h ambulatory holter monitoring:

To evaluate chest pain not reproduced with exercise testing

To evaluate other signs and symptoms that may be heart-related, such as fatigue, shortness of breath, dizziness, or fainting

To identify arrhythmias or palpitations

To assess risk for future heart-related events in certain conditions, such as idiopathic hypertrophic cardiomyopathy, post-heart attack with weakness of the left side of the heart, or Wolff-Parkinson-White syndrome

To assess the function of an implanted pacemaker

To determine the effectiveness of therapy for complex arrhythmias

296.      A woman dx with Ca Breast presents now with urinary freq. which part of the brain is the metastasis spread to?

  1. Brain stem
  2. Pons
  3. Medulla
  4. Diencephalon
  5. Cerebral cortex

Ans. The key is D. Diencephalon. [diencephalon is made up of four distinct components: i) the thalamus ii) the subthalamus iii) the hy- pothalamus and iv) the epithalamus. Among these the hypothalamus has crucial role in causing urinary frequency].

 

297.      A man is very depressed and miserable after his wife’s death. He sees no point in living now that his wife is not around and apol- ogises for his existence. He refuses any help offered. His son has brought him to the ED. The son can’t deal with the father any more. What is the most appropriate next step?

  1. Voluntary admission to psychiatry ward
  2. Compulsory admission under MHA
  3. Refer to social services
  4. Alternate housing
  5. ECT

Ans. The key is B. Compulsory admission under MHA. [This patient is refusing any help offered! And his son cannot deal with him anymore! In this situation voluntary admission to psychiatry ward is not pos- sible and the option of choice is “compulsory admission under MHA”. The point here is the man has felt himself in danger by self neglect].

298.      A 31yo man has epistaxis 10 days following polypectomy. What is the most likely dx?

  1. Nasal infection
  2. Coagulation disorder
  3. Carcinoma

Ans. The key is A. Nasal infection. [Infection is one of the most im- portant cause of secondary hemorrhage].

299.      A woman had an MI. She was breathless and is put on oxygen mask and GTN, her chest pain has improved. Her HR=40bpm. ECG shows ST elevation in leads I, II, III. What is your next step?

  1. LMWH
  2. Streptokinase
  3. Angiography
  4. Continue current management
  5. None

Ans. The key is B. Streptokinase. This is wrong key. Correct key is

  1. Angiography. [Now a days PCI is considered to have much better outcome than thrombolysis].

300.      A 67yo male presents with polyuria and nocturia. His BMI=33, urine culture = negative for nitrates. What is the next dx inv?

  1. PSA
  2. Urea, creat and electrolytes
  3. MSU culture and sensitivity
  4. Acid fast urine test
  5. Blood sugar

Ans. The key is E. Blood sugar. [Age at presentation and class1 obesity favours the diagnosis of type2 DM].

301.      A pt from Africa comes with nodular patch on the shin which is reddish brown. What is the most probable dx?

  1. Lupus vulgaris
  2. Erythema nodosum
  3. Pyoderma gangrenosum
  4. Erythema marginatum
  5. Solar keratosis

Ans. The key is B. Erythema nodosum. [Causes of erythema nodo- sum: MOST COMMON CAUSES- i) streptococcal infection ii) sarcoid- osis. Other causes- tuberculosis, mycoplasma pneumonia, infectious mononucleosis, drugs- sulfa related drug, OCP, oestrogen; Behcet’s disease, CD, UC; lymphoma, leukemia and some others].

 

 

 

 

302.      A 29yo lady came to the ED with complaints of palpitations that have been there for the past 4 days and also feeling warmer than usual. Exam: HR=154bpm, irregular rhythm. What is the tx for her condition?

  1. Amiadarone
  2. Beta blockers
  3. Adenosine
  4. Verapamil
  5. Flecainide

Ans. The key is B. Beta blockers [The probable arrhythymia is AF secondary to thyrotoxicosis. Some of the symptoms of hyperthy- roidism (such as tremor and palpitations, which are caused by excess thyroid hormone acting on the cardiac and nervous system) can be improved within a number of hours by beta-blockers. So to rapid control the symptoms of thyrotoxicosis Beta blocker should be used which will improve the arrhythmia as well].

303.     A T2DM is undergoing a gastric surgery. What is the most appropriate pre-op management?

  1. Start him in IV insulin and glucose and K+ just before surgery
  2. Stop his oral hypoglycemic on the day of the procedure
  3. Continue regular oral hypoglycemic
  4. Stop oral hypoglycemic the prv night and start IV insulin with glucose and K+ before surgery
  5. Change to short acting oral hypoglycemic

Ans. The key is D. Stop oral hypoglycemic the prv night and start IV insulin with glucose and K+ before Surgery.

304.     A 19yo boy is brought by his mother with complaint of lack of interest and no social interactions. He has no friends, he doesn’t talk much, his only interest is in collecting cars/vehicles having around 2000 toy cars. What is the most appropriate dx?

  1. Borderline personality disorder
  2. Depression
  3. Schizoaffective disorder
  4. Autistic spectrum disorder

Ans. The key is D. Autistic spectrum disorder.

Autism spectrum disorders affect three different areas of a child’s life: Social interaction

Communication — both verbal and nonverbal Behaviors and interests

In some children, a loss of language is the major impairment. In oth- ers, unusual behaviors (like spending hours lining up toys) seem to be the dominant factors.

305.    A 45yo man who is diabetic and HTN but poorly compliant has chronic SOB, develops severe SOB and chest pain. Pain is sharp, increased by breathing and relieved by sitting forward. What is the single most appropriate dx?

  1. MI
  2. Pericarditis
  3. Lung cancer
  4. Good pastures syndrome
  5. Progressive massive fibrosis

Ans. The key is B. Pericarditis. [Nature of pain i.e. sharp pain increased by breathing and relieved by sitting forward is suggestive of pericar- ditis].

Nature of pericardial pain: the most common symptom is sharp, stab- bing chest pain behind the sternum or in the left side of your chest. However, some people with acute pericarditis describe their chest pain as dull, achy or pressure-like instead, and of varying intensity.

The pain of acute pericarditis may radiate to your left shoulder and neck. It often intensifies when you cough, lie down or inhale deeply. Sitting up and leaning forward can often ease the pain.

 

306.    A 6m boy has been brought to ED following an apneic episode at home. He is now completely well but his parents are anxious as his cousin died of SIDS (Sudden Infant Death Syndrome) at a similar age. The parents ask for guidance on BLS for a baby of his age. What is the single most recommended technique for cardiac compres- sions?

  1. All fingers of both hands
  2. All fingers of one hand
  3. Heel of one hand
  4. Heel of both hand
  5. Index and middle fingertips of one hand

Ans. The key is E. Index and middle fingertips of one hand.

307.     A 70yo man had a right hemicolectomy for ceacal carcinoma 6days ago. He now has abdominal distension and recurrent vomit- ing. He has not opened his bowels since surgery. There are no bowel sounds. WBC=9, Temp=37.3C. What is the single most appropriate next management?

  1. Antibiotic therapy IV
  2. Glycerine suppository
  3. Laparotomy
  4. NG tube suction and IV fluids
  5. TPN

Ans. The key is D. NG tube suction and IV fluids. [The patient has developed paralytic ileus which should be treated conservatively].

308.    A 60yo man with a 4y hx of thirst, urinary freq and weight loss presents with a deep painless ulcer on the heel. What is the most appropriate inv?

  1. Ateriography
  2. Venography
  3. Blood sugar
  4. Biopsy for malignant melanoma
  5. Biopsy for pyoderma

Ans. The key is C. Blood sugar. [The patient probably developed diabetic foot].

309.    A 16yo boy presents with rash on his buttocks and extensor surface following a sore throat. What is the most probable dx?

  1. Measles
  2. Bullous-pemphigoig
  3. Rubella
  4. ITP
  5. HSP

Ans. The key is D. ITP. It’s probably a wrong key! The correct key should be E. HSP. [In HSP rash typically found in buttocks, legs and

feets and may also appear on the arms, face and trunk. But in ITP it mostly occurs in lower legs. HSP usually follow a sorethroat and ITP follow viral infection like flue or URTI. HSP is a vasculitis while ITP is deficiency of platelets from more destruction in spleen which is immune mediated].

310.    A 34yo man with a white patch on the margin of the mid-third of the tongue. Which is the single most appropriate LN involved?

  1. External iliac LN
  2. Pre-aortic LN
  3. Aortic LN
  4. Inguinal LN
  5. Iliac LN
  6. Submental LN
  7. Submandibular LN
  8. Deep cervical LN

Ans. The key is G. Submandibular LN.

311.     A 50yo lady presents to ED with sudden severe chest pain radi- ating to both shoulder and accompanying SOB. Exam: cold peripher- ies and paraparesis. What is the single most appropriate dx?

  1. MI
  2. Aortic dissection
  3. Pulmonary embolism
  4. Good pastures syndrome
  5. Motor neuron disease

Ans. The key is B. Aortic dissection. [Cold peripheries due to reduced blood flow to dista parts of dissection and reduced perfusion of nerves resulted in paraparesis. Usual management for type A dissection is surgery and for type B is conservative].

312.    A 54yo myopic develops flashes of light and then sudden loss of vision. That is the single most appropriate tx?

  1. Pan retinal photo coagulation
  2. Peripheral iridectomy
  3. Scleral buckling
  4. Spectacles
  5. Surgical extraction of lens

Ans. The key is C. Scleral buckling. [It is a case of retinal detachment with treatment option of scleral buckling].

313.      A 40yo chronic alcoholic who lives alone, brought in the ED having been found confused at home after a fall. He complains of a headache and gradually worsening confusion. What is the most likely dx?

  1. Head injury
  2. Hypoglycemia
  3. Extradural hematoma
  4. Subdural hematoma
  5. Delirium

Ans. The key is D. Subdural hematoma. [subdural hematoma may be acute or chronic. In chronic symptoms may not be apparent for several days or weeks. Symptoms of subdural hematomas are: fluctuating level of consciousness, ± insidious physical or intellectual slowing, sleepiness, headache, personality change and unsteadiness. Tx. Irriga- tion/evacuation e.g. via barr twist drill and barr hole craniostomy 1st line. Craniotomy if the clot organized 2nd line].

314.      A 54yo man with alcohol dependence has tremor and sweating 3days into a hosp admission for a fx femur. He is apprehensive and fearful. What is the single most appropriate tx?

  1. Acamprossate
  2. Chlordiazepoxide
  3. Lorazepam
  4. Lofexidine
  5. Procyclidine

Ans. The key is B. Chlordiazepoxide. C. Lorazepam is also correct key! [Dx alcohol withdrawal symptom. According to NICE 1st line treat- ment is oral lorazepam and if symptom persists or oral medication is declined, give parenteral lorazepam, haloperidol or olanzapine. Accord- ing to OHCM 1st line treatment is chlordiazepoxide.

Chlordiazepoxide should only be used at the lowest possible dose and for a maximum of up to four weeks. This will reduce the risks of developing tolerance, dependence and withdrawal].

315.     A 5yo child complains of sore throat and earache. He is pyrexial. Exam: tonsils enlarged and hyperemic, exudes pus when pressed upon. What is the single most relevant dx?

  1. IM
  2. Acute follicular tonsillitis
  3. Scarlet fever
  4. Agranulocytosis
  5. Acute OM

Ans. The key is B. Acute follicular tonsillitis. [Tonsillitis is usually caused by a viral infection or, less commonly, a bacterial infection.

The given case is a bacterial tonsillitis (probably caused by group A streptococcus). There are four main signs that tonsillitis is caused by a bacterial infection rather than a viral infection. They are:

 

a high temperature

white pus-filled spots on the tonsils no cough

swollen and tender lymph nodes (glands).

316.     A man with a fam hx of panic disorder is brought to the hosp with palpitations, tremors, sweating and muscles tightness on 3 occasions in the last 6 wks. He doesn’t complain of headache and his BP is WNL. What is the single most appropriate long-term tx for him?

  1. Diazepam
  2. Olanzapine
  3. Haloperidol
  4. Fluoxetine
  5. Alprazolam

Ans. The key is D. Fluoxetine. [Recommended treatment for panic disorder is i) CBT ii) Medication (SSRIs or TCA). NICE recommends a total of seven to 14 hours of CBT to be completed within a four month period. Treatment will usually involve having a weekly one to two hour session. When drug is prescribed usually a SSRI is preferred. Antide- pressants can take two to four weeks before becoming effective].

 

 

317.      A 28yo man presents with rapid pounding in the chest. He is completely conscious throughout. The ECG was taken (SVT). What is the 1st med to be used to manage this condition?

  1. Amiodarone
  2. Adenosine
  3. Lidocaine
  4. Verapamil
  5. Metoprolol

Ans. The key is B. Adenosine. [Management of SVT: i) vagal manoeu- vres (carotid sinus message, valsalva manoeuvre) transiently increase AV-block, and unmask the underlying atrial rhythm. If unsuccessful then the first medicine used in SVT is adenosine, which causes tran- sient AV block and works by i) transiently slowing ventricles to show the underlying atrial rhythm ii) cardioverting a junctional tachycardia to sinus rhythm. OHCM].

318.     A 56yo woman who is depressed after her husband died of cancer 3m ago was given amitryptaline. Her sleep has improved and she now wants to stop medication but she still speaks about her husband. How would you manage her?

  1. CBT
  2. Continue amitryptaline
  3. Psychoanalysis
  4. Bereavement counselling
  5. Antipsychotic

Ans. The key is B. Continue amitriptyline. [depression is important feature of bereavement. Patient may pass sleepless nights. As this patients sleep has improved it indicate she has good response to antidepressant and as she still speaks about her husband there is chance to deterioration of her depression if antidepressant is stopped. For depressive episodes antidepressants should be continued for at least 6-9 months].

319.     A 64yo man presents with a hx of left sided hemiparesis and slurred speech. He was absolutely fine 6h after the episode. What is the most appropriate prophylactic regimen?

  1. Aspirin 300mg for 2 weeks followed by aspirin 75mg
  2. Aspirin 300mg for 2 weeks followed by aspirin 75mg and dipyrida- mole 200mg
  3. Clopidogrel 75mg
  4. Dipyridamole 200mg
  5. Aspirin 300mg for 2 weeks

Ans. The key is B. Aspirin 300mg for 2 weeks followed by aspirin 75mg and dipyridamole 200mg. It is wrong key! Current recommend- ed secondary prophylaxis is C. Clopidogrel 75mg.

 

 

 

 

320.     A 63yo lady with a BMI=32 comes to the ED with complaints of pigmentation on her legs. Exam: dilated veins could be seen on the lateral side of her ankle. Which of the following is involved?

  1. Short saphenous vein
  2. Long saphenous vein
  3. Deep venous system
  4. Popliteal veins
  5. Saphano-femoral junction

Ans. The key is A. Short saphenous vein. [short saphenous vein travels lateral aspect of ankle while great or long saphenous vein travels medial aspect of ankle].

321.      A 55yo man presents with hx of weight loss and tenesmus. He is dx with rectal carcinoma. Which risk factors help to develop rectal carcinoma except following?

  1. Smoking
  2. Family hx
  3. Polyp
  4. Prv carcinoma
  5. High fat diet
  6. High fibre diet

Ans. The key is F. High fibre diet. [except high fiber diet all others are risk factors to develop rectal carcinoma].

322.     A pt presents with a painful, sticky red eye with a congested conjunctiva. What is the most suitable tx?

  1. Antibiotic PO
  2. Antihistamine PO
  3. Antibiotic drops
  4. Steroid drops
  5. IBS

Ans. The key is C. Antibiotic drops. [bacterial conjunctivitis is treated with antibiotic drops].

323.      A 45yo woman complains of pain in her hands precipitated by exposure to the cold weather. She has breathlessness on walking. When she is eating, she can feel food suddenly sticking to the gullet. It seems to be in the middle of the esophagus but she can’t localize exactly where it sticks. It is usually relieved with a drink of water. Choose the single most likely cause of dysphagia from the options?

  1. Esophageal carcinoma
  2. Systemic sclerosis
  3. SLE
  4. Pharyngeal carcinoma
  5. Globus hystericus

Ans. The key is B. Systemic sclerosis. [Raynods phenomena, pulmo- nary involvement, oesophageal dysmotility are suggestive of systemic sclerosis].

 

 

324.    A 3yo child brought to the ED with a swelling over the left arm. XR shows multiple callus formation in the ribs. Exam: bruises on childs back. What is the most appropriate next step?

  1. Check child protection register
  2. Coagulation profile
  3. Skeletal survey
  4. Serum calcium
  5. DEXA scan

Ans. The key is C. Skeletal survey. [Skeletal survey is a series of

x-ray which is usually used in NAI]. [after survey should think of child protection].

 

325.     A 35yo woman has had bruising and petechiae for a week. She has also had recent menorrhagia but is otherwise well. Blood: Hgb=11.1, WBC=6.3, Plt=14. What is the single most likely dx?

  1. Acute leukemia
  2. Aplastic anemia
  3. HIV infection
  4. ITP
  5. SLE

Ans. The key is D. ITP. [As the patient is otherwise well acute leu- kemia, HIV and SLE is unlikely. Normal wbc count excludes aplastic anemia. So likely diagnosis is ITP].

326.     A 30yo man complains of episodes of hearing music and some- times threatening voices within a couple of hours of heavy drinking. What is the most likely dx?

  1. Delirium tremens
  2. Wernicke’s encephalopathy
  3. Korsakoff’s psychosis
  4. Alcohol hallucinosis
  5. Temporal lobe dysfunction

Ans. The key is D. Alcoholic hallucinosis. [Alcohol hallucinosis can occur during acute intoxication or withdrawal. It involves auditory and visual hallucinations, most commonly accusatory or threatening voices].

327.      A pt had TIA which he recovered from. He has a hx of stroke and exam shows HR in sinus rhythm. He is already on aspirin 75mg and anti-HTN drugs. What other action should be taken?

  1. Add clopidogrel only
  2. Increase dose of aspirin to 300mg
  3. Add warfarin
  4. Add clopidogrel and statin
  5. Add statin only

Ans. The key is D. Add clopidogrel and statin. This is wrong key! Correct key should be E. Add statin only. [He who is already on aspirin need no change in aspirin dose. Clopidogrel is now drug of choice for secondary prevention in TIA but if someone is already on aspirin he should continue it as same dose and should be considered to shift to clopidogrel in next few visits. So for given case correct option is add statin only].

328.     A 40yo woman suddenly collapsed and died. At the post-mor- tem autopsy, it was found that there a bleed from a berry aneurysm from the circle of Willis. In which space did the bleeding occur?

  1. Subarachnoid
  2. Subdural
  3. Extradural
  4. Subparietal
  5. Brain ventricles

Ans. The key is A. Subarachnoid.

329.      A schizophrenic pt hears people only when he is about to fall asleep. What is the most likely dx?

  1. Hypnopompic hallucinations
  2. Hyponogogic hallucinations
  3. Hippocampal hallucinations
  4. Delirious hallucinations
  5. Auditory hallucinations

Ans. The key is B. Hypnogogic hallucinations.

330.     A pt who came from India presents with cough, fever and en- larged cervical LN. Exam: caseating granulomata found in LN. What is the most appropriate dx?

  1. Lymphoma
  2. TB adenitis
  3. Thyroid carcinoma
  4. Goiter
  5. Thyroid cyst

Ans. The key is B. TB adenitis. [caseating granulomata are diagnostic of TB].

331.      A 44yo man comes with hx of early morning headaches and vomiting. CT brain shows ring enhancing lesions. What is the single most appropriate option?

  1. CMV
  2. Streptococcus
  3. Toxoplasmosis
  4. NHL
  5. Pneumocystis jerovii

Ans. The key is C. Toxoplasmosis.

332.     A 72yo man is found to be not breathing in the CCU with the following rhythm. What is the most likely dx?

 
  
  1. SVT
  2. VT
  3. VF
  4. Atrial fib
  5. Atrial flutter

Ans. The key is C. VF.

333.      A 65yo man with difficulty in swallowing presents with an aspi- ration pneumonia. He has a bovine cough and fasciculating tongue. Sometimes as he swallows food it comes back through his nose. Choose the single most likely cause of dysphagia from the given option?

  1. Bulbar palsy
  2. Esophageal carcinoma
  3. Pharyngeal pouch
  4. Pseudobulbar palsy
  5. Systemic sclerosis

Ans. The key is A. Bulbar palsy. [Dysphagia, nasal regurgitation, fas- ciculating tongue these are features of bulbar palsi due to lower motor neuronal lesion of IX-XII nerves].

334.     A 16yo teenager was brought to the ED after being stabbed on the upper right side of his back. Erect CXR revealed homogenous opacity on the lower right lung, trachea was centrally placed. What is the most probable explanation for the XR findings?

  1. Pneumothorax
  2. Hemothorax
  3. Pneumonia
  4. Tension pneumothorax
  5. Empyema

Ans. The key is B. Hemothorax. [In blunt trauma there may be he- mo-pneumothorax but in sharp wound like stabbing there may occur only hemothorax].

 

335.      A 55yo woman complains of retrosternal chest pain and dys- phagia which is intermittent and unpredictable. The food suddenly sticks in the middle of the chest, but she can clear it with a drink of water and then finish the meal without any further problem. A

barium meal shows a ‘corkscrew esophagus’. What is the single most likely dysphagia?

  1. Esophageal candidiasis
  2. Esophageal carcinoma
  3. Esophageal spasm
  4. Pharyngeal pouch
  5. Plummer-vinson syndrome

Ans. The key is C. Esophageal spasm. [chest pain, unpredictable inter- mittent dysphagia and food suddenly sticks in the middle of the chest which can be cleared with a drink of water indicates uncoordinated irregular esophageal peristalsis which is characteristic of esophageal spasm! Also “cork-screw esophagus” seen in barium swallow is diag- nostic of esophageal spasm].

336.      A 38yo female presents with sudden loss of vision but fun- doscopy is normal. She had a similar episode about 1 y ago which resolved completely within 3m. Exam: mild weakness of right upper limb and exaggerated reflexes. What is the single most appropriate tx?

  1. Pan retinal photo coagulation
  2. Pilocarpine eye drops
  3. Corticosteroids
  4. Peripheral iridectomy
  5. Surgical extraction of lens

Ans. The key is C. Corticosteroid. [Sudden loss of vision, remission and relapse of optic neuritis and focal neurological symptoms and exaggerated reflexes all points towards multiple sclerosis. Treatment option is corticosteroids].

337.      A 15yo boy presents with a limp and pain in the knee. Exam: leg is externally rotated and 2cm shorter. There is limitation of flexion, abduction and medial rotation. As the hip is flexed external rotation is increased. Choose the most likely dx?

  1. Juvenile rheumatoid arthritis
  2. Osgood-schlatter disease
  3. Reactive arthritis
  4. Slipped femoral epiphysis
  5. Transient synovitis of the hip

Ans. The key is D. Slipped femoral epiphysis. [The given presentation is classic for slipped femoral epiphysis].

338.      A 64yo woman has difficulty moving her right shoulder on recovering from surgery of the posterior triangle of her neck. What is the single most appropriate option?

  1. Accessory nerve
  2. Glossopharyngeal nerve
  3. Hypoglossal nerve
  4. Vagus nerve
  5. Vestibule-cochlear nerve

Ans. The key is A. Accessory nerve. [Accessory nerve lesion caus- es weakness of the trapezius muscle and can produce a drooping

shoulder, winged scapula, and a weakness of forward elevation of the shoulder].

339.      A 37yo man with an ulcer on the medial malleolus. Which of the following LN is involved?

  1. External iliac LN
  2. Pre-aortic LN
  3. Aortic LN
  4. Inguinal LN
  5. Iliac LN
  6. Submental LN
  7. Submandibular LN
  8. Deep cervical LN

Ans. The key is D. Inguinal LN.

340.     A pt presents with weight loss of 5kgs despite good appetite. He also complains of palpitations,sweating and diarrhea. He has a lump in front of his neck which moves on swallowing. What isthe most appropriate dx?

  1. Lymphoma
  2. TB adenitis
  3. Thyroid Ca
  4. Goiter
  5. Thyroid cyst

Ans. The key is D. Goiter.

341.      A 76yo woman has become tired and confused following an in- fluenza like illness. She is also breathless with signs of consolidation of the left lung base. What is the most likely dx?

  1. Drug toxicity
  2. Delirium tremens
  3. Infection toxicity
  4. Hypoglycemia
  5. Electrolyte imbalance
  6. The key is Infection toxicity. [Infection toxicity or toxic shock syn- drome is likely here as preceding flue like illness points towards toxin

(enterotoxin type B) from Staphylococcus aureus].

342.     A young pt is complaining of vertigo whenever she moves sideways on the bed while lying supine. What would be the most appropriate next step?

  1. Head roll test
  2. Reassure
  3. Advice on posture
  4. Carotid Doppler
  5. CT

Ans. The key is A. Head roll test. [this is a case of “benign paroxysmal positional vertigo” for which the diagnosis is made by head roll test].

343.      A 32yo man has OCD. What is the best tx?

  1. CBT
  2. SSRI
  3. TCA
  4. MAO inhibitors
  5. Reassure

Ans. The key is B. SSRI. This is wrong key! The correct key is CBT].

344.    A 65yo woman says she died 3m ago and is very distressed that nobody has buried her. When she is outdoors, she hears people say that she is evil and needs to be punished. What is the most likely explanation for her symptoms?

  1. Schizophrenia
  2. Mania
  3. Psychotic depression
  4. Hysteria
  5. Toxic confusional state

Ans. The key is C. Psychotic depression. [Psychotic depression, also known as depressive psychosis, is a major depressive episode that is accompanied by psychotic symptoms (hallucinations, delusions).

In this patient nihilistic delusion favours the diagnosis of psychotic depression. It can occur in the context of bipolar disorder or majorde- pressive disorder].

 

345.     A 50yo woman presents following a fall. She reports pain and weakness in her hands for several months, stiff legs, swallowing difficulties, and has bilateral wasting of the small muscles of her hands. Reflexes in the upper limbs are absent. Tongue fasciculations are present and both legs show increased tone, pyramidal weakness and hyper-reflexia with extensor plantars. Pain and temp sensation are impaired in the upper limbs. What is the most likely dx?

  1. MS
  2. MND
  3. Syringobulbia
  4. Syringomyelia
  5. Myasthenia gravis

Ans. The key is C. Syringobulbia. [In MS there are characteristic re- lapse and remission which is absent here; In MND there is no sensory deficit; Syringomyelia doesn’t cause cranial nerve lesion and in myas- thenia there is muscular weakness without atrophy. Here the features described well fits with syringobulbia].

346.     Which of the following formulas is used for calculating fluids for burn pts?

  1. 4 x weight(lbs) x area of burn = ml of fluids
  2. 4 x weight(kgs) x area of burn = L of fluids
  3. 4 x weight(kgs) x area of burn = ml of fluids
  4. 4 x weight(lbs) x area of burn = L of fluids
  5. 5 x weight(kgs) x area of burn = dL of fluids

Ans. The key is C. 4 x weight(kgs) x area of burn = ml of fluids.

347.      A 65yo male presents with dyspnea and palpitations. Exam: pulse=170bpm, BP=120/80mmHg. Carotid massage has been done as first instance. What is the next step of the management?

  1. Adenosine
  2. Amlodipine
  3. DC cardioversion
  4. Lidocaine
  5. Beta blocker

Ans. The key is A. Adenosine. [The likely diagnosis is SVT. 1st vagal manoeuvres, if fails iv adenosine.

  • Vagal manoeuvres (carotid sinus massage, Valsalva manoeuvre) transiently increase

AV block, and may unmask an underlying atrial rhythm.

  • If unsuccessful, give adenosine, which causes transient AV block; OHCM, 9th edition].

348.     A 48yo farmer presented with fever, malaise, cough and SOB. Exam: tachypnea, coarse endinspiratory crackles and wheeze throughout, cyanosis. Also complaint severe weight loss. His CXR shows fluffy nodular shadowing and there is PMN leukocytosis. What is the single most appropriate dx?

  1. Ankylosing spondylitis
  2. Churg-strauss syndrome
  3. Cryptogenic organizing
  4. Extrinsic allergic alveolitis
  5. Progressive massive fibrosis

Ans. The key is D. Extrinsic allergic alveolitis.

349.      A 35yo lady is admitted with pyrexia, weight loss, diarrhea and her skin is lemon yellow in color. CBC = high MCV. What is the most probably dx?

  1. Aplastic anemia
  2. Pernicious anemia
  3. Leukemia
  4. ITP
  5. Lymphoma

Ans. The key is B. Pernicious anemia. [It may be graves with per- nicious anemia. Lemon yellow pallor occurs in pernicious anemia. Hyperthyroidism may cause persistently raised body temperature. Both are autoimmune disease which favours this association].

350.     A 72yo woman who had a repair of strangulated femoral hernia 2 days ago becomes noisy, aggressive and confused. She is febrile, CBC normal apart from raised MCV. What is the most likely dx?

  1. Electrolyte imbalance
  2. Delirium tremens
  3. Wernicke’s encephalopathy
  4. Infection toxicity
  5. Hypoglycemia

Ans. The key is B. Delirium tremens. [Electrolyte imbalance may cause confusion but not aggressiveness; infection toxicity will cause high fe- ver, low BP, rash etc which are absent here (fever here is hyperthermia of delirium tremens which is seen in some cases). Abstinance from alcohol in the hospital caused delirium tremens (chronic alcoholism is supported by high MCV) here].

351.      An old lady had UTI and was treated with antibiotics. She then developed diarrhea. What is the single most likely tx?

  1. Co-amoxiclav
  2. Piperacillin + tazobactam
  3. Ceftriaxone
  4. Vancomycin

Ans. The key is D. Vancomycin. [Pseudomembranous colitis is treated with metronidazole or vancomycin].

 

 

352.     A 56yo man has symptoms of sleep apnea and daytime head- aches and somnolence. Spirometry shows a decreased tidal volume and vital capacity. What is the single most appropriate dx?

  1. Ankylosing spondylitis
  2. Churg-strauss syndrome
  3. Good pasture syndrome
  4. Motor neuron disease
  5. Progressive massive fibrosis
  6. Spinal cord compression

Ans. The key is D. Motor neuron disease. [involvement of respiratory muscles in MND is associated with poor respiration causing sleep apnoea].

353.      A 55yo man presents with mild headache. He has changed his spectacles thrice in 1 yr. there is mild cupping present in the disc and sickle shaped scotoma present in both eyes. What is the single most appropriate tx?

  1. Pan retinal photo coagulation
  2. Pilocarpine eye drops
  3. Corticosteroids
  4. Scleral buckling
  5. Analgesics alone

Ans. The key is B. Pilocarpine. [This is a case of open angle glaucoma, treatment is with pilocarpine].

 

 

354.    A 55yo woman was found collapsed at home, paramedics revived her but in the ambulance she had a cardiac arrest and couldn’t be saved. The paramedic’s report tells that the woman was immobile lately due to hip pain and that they found ulcers on the medial side of ankle. She had DM and was on anti-diabetics. What is the cause of her death?

  1. Acute MI
  2. DKA
  3. Pulmonary embolism
  4. Acute pericarditis
  5. Cardiac tamponade

Ans. The key is C. Pulmonary embolism. [Immobilization due to hip pain may resulted in DVT and later pulmonary embolism. Please note ulcer in medial side indicate undiagnosed DVT and its presence favour the presence of DVT and followed by pulmonary embolism].

 

355.     An 18yo previously well student is in his 1st year at uni. He has been brought to the ED in an agitated, deluded and disoriented state. What is the most probable reason for his condition?

  1. Drug toxicity
  2. Delirium tremens
  3. Infection toxicity
  4. Electrolyte imbalance
  5. Head injury

Ans. The key is A. Drug toxicity. [Young age and 1st yr in university is likely to point towards drug toxicity].

356.      A young adult presents to the ED after a motorcycle crash. The pt has bruises around the left orbital area. GCS=13, examination notes alcoholic breath. Shortly afterwards, his GCS drops to 7. What is the single most important initial assessment test?

  1. MRI brain
  2. CT brain
  3. CXR
  4. CT angio brain
  5. Head XR

Ans. The key is B. CT brain. [Likely cause is epidural hematoma].

357.      A 30yo female attends OPD with a fever and dry cough. She says that she had headache, myalgia and joint pain like one week ago. Exam: pulse=100bpm, temp=37.5C. CXR: bilateral patchy consol- idation. What is the single most likely causative organism?

  1. Pneumococcal pneumonia
  2. Legionella
  3. Mycoplasma
  4. Klebsiella
  5. Chlamydia pneumonia

Ans. The key is C. Mycoplasma. [Myalgia, joint pain, bilateral patchy consolidations points towards mycoplasma pneumonia].

358.      A 46yo man is being investigated for indigestion. Jejunal biopsy shows deposition of macrophages containing PAS (Periodic acid-schiff) +ve granules. What is the most likely dx?

  1. Bacterial overgrowth
  2. Celiac disease
  3. Tropical sprue
  4. Whipple’s disease
  5. Small bowel lymphoma

Ans. The key is D. Whipple’s disease. [periodic acid-schiff +ve granules containing macrophages in jejunal biopsy is diagnostic of whipples disease].

359.      A 32yo woman of 38wks gestation complains of feeling unwell with fever, rigors and abdominal pains. The pain was initially located in the abdomen and was a/w urinary freq and dysuria. The pain has now become more generalized specifically radiating to the right loin. She says that she has felt occasional uterine tightening. CTG is reassuring. Select the most likely dx?

  1. Acute fatty liver of pregnancy
  2. Acute pyelonephritis
  3. Roung ligament stretching
  4. Cholecystitis
  5. UTI

Ans. The key is B. Acute pyelonephritis. [Fever, rigor, abdominal pain a/w frequency, dysurea and radiation to the rt loin suggests rt sided pyelonephritis].

360.     A 32yo pt presents with cervical lymphadenopathy and spleno- megaly. What is the single most appropriate option?

  1. Hemophilus
  2. Streptococcus
  3. Toxoplasmosis
  4. NHL
  5. Pneumocystis jerovcii

Ans. The key is D. NHL. [ Here only two points are mentioned- cervical lymphadenopathy and splenomegaly! This combination makes NHL as the most likely cause though splenomegaly is a relatively uncom- mon feature of it!!! This combination does not fit in other options!]

361.      A 62yo man who was admitted for surgery 3days ago sudden- ly becomes confused. His attn span is reduced. He is restless and physically aggressive and picks at his bed sheets. What single aspect of the pt’s hx recovered in his notes is most likely to aid in making the dx?

  1. Alcohol consumption
  2. Head trauma
  3. Hx of anxiety
  4. Prescribed med
  5. Obvious cognitive impairment

Ans. The key is A. Alcohol consumption. [abstinence from alcohol in the hospital lead to delirium tremens].

362.     A 10yo girl presents with pallor and features of renal failure. She has hematuria as well as proteinuria. The serum urea and creat are elevated. These symptoms started after an episode of bloody diarrhea 4days ago. What is the most probable dx?

  1. TTP
  2. HUS
  3. ITP
  4. HSP
  5. ARF

Ans. The key is B. HUS. [Most cases of hemolytic uremic syndrome develop in children after two to 14 days of diarrhea often bloody, due to infection with a certain strain of E. coli. Features may be i) abdom- inal pain, ii) pale skin, iii) hematuria and proteinuria, iv) features of renal failure like- nausea/vomiting, swelling of face, hand, feet or entire body etc. v) elevated urea and creatinine etc.].

363.      A 40yo woman has had intermittent tension, dizziness and anxiety for 4months. Each episode usually resolves after a few hours. She said she takes alcohol to make her calm. She is in a loving relationship and has no probs at work or home. What is the next step in her management?

  1. Collateral info
  2. CT brain
  3. CBC
  4. LFT
  5. TFT

Ans. The key is A. Collateral info.

364.    A 45yo IV drug abuser is brought into the ED with complaint of fever, shivering, malaise, SOB and productive cough. Exam:

temp=39C, pulse=110bpm, BP=100/70mmHg. Inv: CXR=bilateral cav- itating bronchopneumonia. What is the single most likely causative organism?

  1. Mycoplasma
  2. Staphylococcus
  3. Chlamydia pneumonia
  4. Pseudomonas
  5. PCP

Ans. The key is B. Staphylococcus. [Among the given causes Staphy- lococcus and PCP are recognized cause of cavitating pneumonia. This case is with productive cough which goes more with staphylococcus as PCP is not productive but rather associated with dry cough. Drug abuse can support both staphylococcus and PCP].

 

365.     A 71yo woman looks disheveled, unkempt and sad with poor eye contact. She has recently lost her husband. Which of the following describes her condition?

  1. Anxiety
  2. Hallucination
  3. Mania
  4. High mood
  5. Low mood

Ans. The key is E. Low mood.

366.     A 62yo male comes to the GP complaining of double vision while climbing downstairs. Which of the following nerve is most likely involved?

  1. Abducens nerve
  2. Trochlear nerve
  3. Oculomotor nerve
  4. Optic nerve
  5. Trigeminal nerve

Ans. The key is B. Trochlear nerve. [oculomotor may cause palsy of inferior rectus, medial rectus and superior rectus causing double vision in multiple gaze! But trochlear involving superior oblique only causes diplopia in downgaze only. So the answer is Trochlear nerve].

 

 

367.     L1 level, what is the most appropriate landmark?

  1. Mcburney’s point
  2. Stellate ganglion
  3. Deep inguinal ring
  4. Termination of the spinal cord
  5. Transpyloric plane

Ans. The given key is D. Termination of the spinal cord. This is a wrong key. The correct key is E. Transpyloric plane. [The termination of the spinal cord is between L1 and L2 (variable between people). L1 land- mark- duodenum first part, superior mesenteric artery, hila of both kidneys, upper border of pancreas, splenic artery, pylorus and fundus of gall bladder].

368.     A 32yo woman presents to the ED with headache and vomiting. She was decorating her ceiling that morning when the headache began, felt mainly occipital with neck pain. Some 2hs later she felt nauseated, vomited and was unable to walk. She also noticed that her voice had altered. She takes no reg meds and has no significant PMH. Exam: acuity, field and fundi are normal. She has upbeat nystagmus in all directions of gaze with normal facial muscles and tongue movements. Her uvulas deviated to the right and her speech is slurred. Limb exam: left arm past-pointing and dysdiadochokinesis with reduced pin prick sensation in her right arm and leg. Although

power is normal, she can’t walk as she feels too unsteady. Where is the most likely site of lesion?

  1. Right medial medulla
  2. Left medial pons
  3. Left cerebellar hemisphere
  4. Right lateral medulla
  5. Left lateral medulla

Ans. The key is Left lateral medulla. [ There is a loss of pain and temperature sensation on the contralateral (opposite) side of the body and ipsilateral (same) side of the face. There is associated cerebellar symptoms and other cranial nerve involvement. The condition is known as Lateral Medullary Syndrome].

369.     A 28yo female presents with 1 wk hx of jaundice and 2d hx of altered sleep pattern and moods. She was dx with hypothyroidism for which she is receiving thyroxine. TFT showed increased TSH. PT=70s. What is the most probable dx?

  1. Acute on chronic liver failure
  2. Hyper-acute liver failure
  3. Autoimmune hepatitis
  4. Acute liver failure
  5. Drug induced hepatitis

Ans. The key is C. Autoimmune hepatitis. [Autoimmune hepatitis may present as acute hepatitis, chronic hepatitis, or well-established cirrhosis. Autoimmune hepatitis rarely presents as fulminant hepatic failure. One third may present as acute hepatitis marked by fever, hepatic tenderness and jaundice. Non specific features are anorexia, weight loss and behavioural change (here altered sleep pattern and moods). There may be coagulopathy (here PT=70s.) leading to epi- staxis, gum bleeding etc. Presence of other autoimmune disease like hypothyroidism supports the diagnosis of autoimmune hepatitis].

 

 

370.     A 55yo man has a chronic cough and sputum, night sweats and weight loss. What is the single most likely causative organism?

  1. Coagulase +ve cocci in sputum
  2. Gram -ve diplococci in sputum
  3. Gram +ve diplococci in sputum
  4. Pneumocystis carinii in sputum
  5. Sputum staining for mycobacterium tuberculosis

Ans. The key is E. Sputum staining for mycobacterium tuberculosis. [Chronic cough and sputum, night sweats and weight loss are classic features of tuberculosis].

371.     A 20yo pregnant 32wks by date presents to the antenatal clinic with hx of painless vaginal bleeding after intercourse. Exam: P/A – soft and relaxed, uterus=dates, CTG=reactive. Choose the single most likely dx?

  1. Abruption of placenta 2nd to pre-eclampsia
  2. Antepartum hemorrhage
  3. Placenta previa
  4. Preterm labor
  5. Placenta percreta

Ans. The key is C. Placenta previa.

372.     A 30yo man presents to the ED with difficulty breathing. He has returned from India. Exam: throat reveals grey membranes on the tonsils and uvula. He has mild pyrexia. What is the single most relevant dx?

  1. Diphtheria
  2. IM
  3. Acute follicular tonsillitis
  4. Scarlet fever
  5. Agranulocytosis

Ans. The key is A. Diphtheria. [history of travel to india, grey mem- brane in tonsil and uvula, low grade fever, and dyspnoea support the diagnosis of diphtheria].

373.      A 23yo man comes to the ED with a hx of drug misuse. He rec- ognizes that he has a prb and is willing to see a psychiatrist. Which of the following terms best describes this situation?

  1. Judgement
  2. Thought insertion
  3. Thought block
  4. Mood
  5. Insight

Ans. The key is E. Insight. [in psychiatry, the patient’s aware- ness and understanding of the origins and meaning of his atti-

tudes, feelings, and behavior and of his disturbing symptoms (self-un- derstanding) is known as insight].

 

 

374.     A pt with hodgkins lymphoma who is under tx develops high fever. His blood results show WBC <2800 and has a chest infection. Choose the most likely tx?

  1. Co-amoxiclav
  2. Piperacillin+tazobactam
  3. Erythromycin
  4. Piperacillin+Co-amoxiclav
  5. Penicillin+tazobactam

 

Ans. The key is B. Piperacillin+tazobactam. [Here patients WBC is

<2800, i.e. patient has leucopenia (probable neutropenia). Piperacillin/ Tazobactam may be used in the management of neutropenic patients with fever suspected to be due to a bacterial infection as in patient with postchemotherapy neutropenia. Even febrile neutropenia can be seen in patients with cancer per-se!].

375.      A 25yo woman presents with urinary freq, dysuria and fever. Urine microscopy shows 20-50 RBC and 10-20 WBC in each field. What is the most probable dx?

  1. Schistosmiasis
  2. Kidney trauma
  3. Ureteric calculus
  4. Bladder calculi
  5. Cystitis

Ans. The key is E. Cystitis. [Hematuria and significant WBC in urine (>10 per HPF) makes cystitis the most likely diagnosis].

376.      A 65yo presents with dyspareunia after sex. She isin meno- pause. She complains of bleeding after sex. What is the most probably dx?

  1. Cervical ca
  2. Endometrial ca
  3. Ovarian ca
  4. Breast ca
  5. Vaginal ca

Ans. The key is B. Endometrial carcinoma. [Any post menopausal bleeding is endometrial carcinoma unless proven otherwise].

377.      A 45yo man underwent an emergency splenectomy following a fall from his bicycle. He smokes 5 cigarettes/day. Post-op, despite mobile, he develops swinging pyrexia and a swollen painful left calf.

His CXR shows lung atelectasis and abdominal US demonstrates a small sub-phrenic collection. What is the single most likely risk factor for DVT in this pt?

  1. Immobility
  2. Intraperitoneal hemorrhage
  3. Smoking
  4. Splenectomy
  5. Sub-phrenic collection

Ans. The key is D. Splenectomy. [Splenectomized patients have an increased risk of developing deep vein thrombosis and pulmonary embolism].

378.      A 6m baby had LOC after which he had jerky movement of hands and feet. What is the most probable dx?

  1. Infantile spasm
  2. Absence
  3. Partial simple seizure
  4. Atonic seizure
  5. Partial complex

Ans. The given key is A. Infantile spasm which is a wrong key! Correct key is E. Partial complex. [In partial simple seizer there is no LOC (loss of consciousness). Infantile spasm though may be associated with occasional LOC but its nature is more generalized rather than the focal nature described here. So the likely option is E. Partial complex seizure].

379.      A 24yo primigravida who is 30wk pregnant presents to the la- bor ward with a hx of constant abdominal pain for the last few hours. She also gives a hx of having lost a cupful of fresh blood per vagina before the pain started. Abdominal exam: irritable uterus, CTG=reac- tive. Choose the single most likely dx?

  1. Abruption of placenta 2nd to pre-eclampsia
  2. Antepartum hemorrhage
  3. Placenta previa
  4. Vasa previa
  5. Revealed hemorrhage

Ans. The key is B. Antepartum hemorrhage. [Though presentation indicates abruption placenta but abrutio placenta 2nd to pre-eclump- sia can’t be the option as no features are suggestive of pre-eclumpsia here. So better option is B. Antepartum hemorrhage which also includes abruption placenta. Only bleeding cannot indicate whether it is concealed, mixed or revealed].

380.     A 62yo lady presents with right sided headache and loss of vision. What is the single most inv?

  1. ESR
  2. BUE
  3. CT head
  4. XR orbit
  5. IOP

Ans. The key is A. ESR. [Elderly (age 62), rt sided headache and loss of vision are suggestive of temporal arteritis where elevated ESR is highly suggestive of this diagnosis].

381.      A 24yo man asks his GP for a sick note from work. He says that feels down, is lethargic and has stopped enjoying playing the piccolo (his main hobby). He was admitted to the psychiatry ward last year following an episode of overspending, promiscuity and distractibility. What is the most probable dx?

  1. Psychosis
  2. Cyclothymia
  3. Bipolar affective disorder
  4. Seasonal affective disorder

Ans. The key is C. Bipolar affective disorder. [presently patient has depression and previous features of mania makes the diagnosis of bipolar affective disorder likely].

382.      A 42yo female who is obese comes with severe upper abdom- inal pain with a temp=37.8C. She has 5 children. What is the most probable dx?

  1. Ectopic pregnancy
  2. Ovarian torsion
  3. Hepatitis
  4. Biliary colic
  5. Cholecystitis

Ans. The key is cholecystitis. [Fat, female, fare, forty and fertile are the pnemonic for cholecystitis! Here the presentation of severe upper abdominal pain with fever along with the pnemonic features points towards the diagnosis of cholecystitis].

383.      A child has just recovered from meningitis. What inv will you do before discharge?

  1. CT scan
  2. EEG
  3. Blood culture
  4. Repeat LP
  5. Hearing test

Ans. The key is E. Hearing test. [deafness is a common complication of meningitis, so hearing test is suggested before discharge].

384.     A primiparous woman with no prv infection with herpes zoster is 18wk pregnant. She had recent contact with a young 21yo pt having widespread chicken pox. What is the most suitable management for the pregnant lady?

  1. Acyclovir PO
  2. Acyclovir IV +IVIG
  3. Acyclovir IV
  4. Reassure
  5. IVIG

Ans. The key is E. IVIg. [If the pregnant woman is not immune to

 

VZV and she has had a significant exposure, she should be offered varicella-zoster immunoglobulin (VZIG) as soon as possible. VZIG is effective when given up to 10 days after contact (in the case of contin- uous exposures, this is defined as 10 days from the appearance of the rash in the index case). RCOG guideline].

385.     A 40yo woman presents to the GP with low mood. Of note, she has an increased appetite and has gone up 2 dress sizes. She also complains that she can’t get out of bed until the afternoon. What is the most likely dx?

  1. Pseudo depression
  2. Moderate depression
  3. Severe depression
  4. Dysthymia
  5. Atypical depression

Ans. The key is E. Atypical depression. [Atypical depression is a sub- type of major depression or dysthymic disorder that involves several specific symptoms, including increased appetite or weight gain, sleepiness or excessive sleep, marked fatigue or weakness, moods that are strongly reactive to environmental circumstances, and feeling extremely sensitive to rejection].

386.     An 8yo boy is clinically obese. As a baby he was floppy and diffi- cult to feed. He now has learning difficulties and is constantly eating despite measures by his parents to hide food out of reach. What is the most probable dx?

  1. Cushing’s syndrome
  2. Congenital hypothyroidism
  3. Prader Willi syndrome
  4. Lawrence moon biedel syndrome
  5. Down’s syndrome

Ans. The key is C. Prader Willi syndrome. [Prader Willi syndrome rare congenital disorder characterized by learning difficulties, growth

abnormalities, and obsessive eating, caused especially by the absence of certain genes normally present on the copy of chromosome 15 inherited from the father].

387.      A 20yo lady is suffering from fever and loss of appetite. She has been dx with toxoplasmosis. What is the tx?

  1. Pyrimethamine
  2. Pyrimethamine + sulfadiazine
  3. Clindamycin
  4. Spiramycin
  5. Trimethoprim + sulfamethoxazole

Ans. The key is B. Pyrimethamine + sulfadiazine. [If the eye is involved, or if immunocompromized, tx option is pyrimethamine + sulfadiazine OHCM 9th edition, page 404].

388.     A 68yo woman has a sudden onset of pain and loss of hearing in her left ear and unsteadiness when walking. There are small lesions visible on her palate and left external auditory meatus. What is the single most likely dx?

  1. Acute mastoiditis
  2. Cholesteatoma
  3. Herpes zoster infection
  4. Oropharyngeal malignancy
  5. OM with infusion

Ans. The key is C. Herpes zoster infection.

389.      A 45yo woman has been dx with GCA and is being treated with steroids. What is the other drug that can be added to this?

  1. ACEi
  2. Beta blockers
  3. Aspirin
  4. Interferons
  5. IVIG

Ans. The key is C. Aspirin [Low dose aspirin is increasingly being rec- ommended for people with a history of giant cell arteritis. It has been found to be effective in preventing complications of giant cell arteritis, such as heart attacks or stroke (nhs.uk)].

 

 

390.     A 17yo man has acute pain and earache on the right side of his face. Temp=38.4C and has extensive pre-auricular swelling on the right, tender on palpation bilaterally. What is the single most likely dx?

  1. Acute mastoiditis
  2. Acute otitis externa
  3. Acute OM
  4. Mumps
  5. OM with effusion

Ans. The key is D. Mumps. [C/F: prodromal malaise, increased tem- perature, painful parotid swelling, becoming bilateral in 70%. OHCS 9th edition, page 142].

391.      An ECG of an elderly lady who collapsed in the ED shows rapid ventricular rate of 220bpm, QRS=140ms. What is the most probable dx?

  1. Atrial fibrillation
  2. VT
  3. SVT
  4. Mobitz type1 2nd degree heart block
  5. Sinus tachycardia

Ans. The key is B. Ventricular tachycardia. [Dx: i) history (if IHD/MI likelihood of a ventricular arrhythmia is > 95%), ii) 12 lead ECG, and iii) lack of response to IV adenosine). ECG findings in favour of VT:

Positive QRS concordance in chest leads Marked left axis deviation

AV dissociation (occurs in 25%) or 2:1 or 3:1 AV block Fusion beats or capture beats

OHCM 9th edition, page 816]. [In the given case collapse, ventricular rate of 220 and broad QRS of 140ms points towards VT].

392.     A pt presents with purple papular lesions on his face and upper trunk measuring 1-2 cm across. They aren’t painful or itchy. What is the single most likely dx?

  1. Kaposi’s sarcoma
  2. Hairy leukoplakia
  3. Cryptosporidium
  4. CMV infection
  5. Cryptococcal infection

Ans. The key is A. Kaposis sarcoma. [It is a spindle-cell tumour derived from capillary endothelial cells or from fibrous tissue, caused by hu- man herpes virus. It presents as purple papules (½ to 1 cm) or plaques on skin and mucosa (any organ). It metastasizes to nodes. OHCM 9th edition, page 716].

 

 

393.      A 6yo boy is clinically obese, his BMI >95th centile. He has no other medical prbs, examination is unremarkable. His mother says that she has tried everything to help him lose weight. What is the most probable dx?

  1. Cushing’s syndrome
  2. Congenital hypothyroidism
  3. Down’s syndrome
  4. Lawrence moon biedel syndrome
  5. Primary obesity

Ans. The key is E. Primary obesity.

 

394.     A 20yo boy is brought by his parents suspecting that he has taken some drug. He is agitated, irritated and can’t sleep. Exam: perforated nasal septum. Which of the following is the most likely to be responsible for his symptoms?

  1. Heroine
  2. Cocaine
  3. Ecstasy/MDMA/amphetamine
  4. Alcohol
  5. Opioids

Ans. The key is B. Cocaine. [drug abuse with perforated nasal septum indicates cocaine abuse].

395.      For a pt presenting with Parkinson’s disease which of the fol- lowing drugs is most useful in the management of the tremor?

  1. Apomorphine
  2. Cabergoline
  3. Selegiline
  4. Amantadine
  5. Benzhexol

Ans. The key is E. Benzhexol. [Benzhexol (an antiparkinsonian agent of anticholinergic class) is the drug of choice in parkinson’s disease induced tremor)].

396.      A 26yo woman has become aware of increasing right sided hearing deficiency since her recent pregnancy. Her eardrums are normal. Her hearing tests show: BC-normal. Weber test lateralizes to the right ear. What is the single most likely dx?

  1. Encephalopathy
  2. Functional hearing loss
  3. Tympano-sclerosis
  4. Otosclerosis
  5. Sensorineural deafness

Ans. The key is D. Otosclerosis. [There are no features of encephalop- athy. As Weber test is lateralized it is unlikely to be functional hearing loss. In tympanosclerosis ear drum becomes chalky white. So as the ear drum is normal it is not tympanosclerosis. Weber test is lateral- ized to right and deafness is also on the right. So it not sensorineural deafness but conductive deafness which makes otosclerosis as the most likely diagnosis].

397.      A 58yo T1DM on anti-HTN therapy for 13yrs developed central chest pain for 45 mins while driving a/w cold sweating and dyspnea. What is the single most appropriate dx?

  1. MI
  2. Pericarditis
  3. Pulmonary embolism
  4. Costochondritis
  5. Pneumothorax

Ans. The key is A. MI. [In pericarditis pain is aggravated by inspira- tion or lying flat and relieved by leaning forward. Pericardial rub may present and there may be fever. In pneumothorax pain is not central but pleuritic. Pulmonary embolism=dyspnoea and pleuritic chest pain. In costrochondritis localized pain/tenderness at the costochon- dral junction enhanced by motion, coughing, or sneezing. The given picture of central chest pain for 45 minutes (more than 30 minutes), sweating and dyspnoea with major risk factor of DM and Htn suggest the diagnosis of MI].

398.      A man was brought to the ED from a shopping mall after col- lapsing there. He is conscious and answering questions now. His ECG shows irregular rhythm. Your choice of inv:

  1. CT
  2. MRI
  3. 24h ECG
  4. Echo

Ans. The key is D. Echo. [Echo may show clot in atrial appendage responsible for this attack of TIA secondary to atrial fibrillation].

399.       A 10yo boy is clinically obese and the shortest in his class. He had a renal transplant last year and his mother is worried that he is being bullied. What is the most probable dx?

  1. Cushing’s syndrome
  2. Congenital hypothyroidism
  3. Pseudocushing’s syndrome
  4. Lawrence moon biedel syndrome
  5. Down’s syndrome

Ans. The key is A. Cushing’s syndrome. [Renal transplant–> immune suppression is needed-> exogenous steroid–> cushing syndrome. short stature–> if steroids are used in early age then they cause premature fusion of growth plate/calcification].

400.     A 45yo man had cancer of head of pancreas which has been removed. He has a hx of longstanding heartburn. He now comes with rigid abdomen which is tender, temp 37.5C, BP=90/70mmHg, pulse=120bpm. What is the next step of the inv?

  1. CT abdomen
  2. XR abdomen
  3. MRI abdomen
  4. US abdomen
  5. Endoscopy

Ans. The key is B. X-ray abdomen. [X-ray abdomen will help diagnos- ing perforation by showing gas under diaphragm. This is a case of perforated peptic ulcer with the features of shock, abdominal rigidity and raised temperature. Stress from serious disease and operation causes the body to produce higher amounts of acid, which can irritate preexisting ulcers leading to easy perforation].

401.      A 50yo man presents to the ED with acute back pain radiating down to his legs. Pain which is usually relieved by lying down and exacerbated by long walks and prolong sitting. What inv would be the best option?

  1. MRI
  2. CT spine
  3. XR spine
  4. Dual energy XR abruptiometry
  5. Serum paraprotein electrophoresis

Ans. The key is A. MRI. [Back pain radiating to leg, pain releaved by lying down and exacerbated by long walk and prolonged sitting are characteristic of lumber (intervertebral) disc disease].

402.      What is the most appropriate antibiotic to treat uncomplicated chlamydial infection in a 21yo female who isn’t pregnant?

  1. Erythromycin
  2. Ciprofloxacin
  3. Metronidazole
  4. Cefixime
  5. Doxycycline

Ans. The key is E. Doxycicline. [Doxycycline 100 mg twice-daily for seven days or a single dose of 1 g of azithromycin or Erythromyin 500 mg twice daily for 14 days or four times daily for seven days or Oflox- acin 200 mg twice-daily or 400 mg once-daily for 7 days. In pregnant Azithromycine 1g single dose is recommended then erythromycin 500 mg twice daily for fourteen days or four times daily for seven days. Then amoxicillin 500 mg three times daily for 7 days].

 

403.     A 45yo manual worker presented with a 2h hx of chest pain radiating to his left arm. His ECG is normal. What is the single most appropriate inv?

  1. Cardiac enzymes
  2. CXR
  3. CT
  4. ECG
  5. V/Q scan

Ans. A. Cardiac enzymes.

404.   A 26yo woman had bipolar disorder for 10yrs and is on Lithium for it. She is symptom free for the past 4 years. She is now planning her pregnancy and wants to know whether she should continue tak- ing lithium. What is the single most appropriate advice?

 

  1. Continue lithium at the same dose and stop when pregnancy is confirmed
  2. Continue lithium during pregnancy and stop when breast feeding
  3. Reduce lithium dosage but continue throughout pregnancy
  4. Reduce lithium gradually and stop when pregnancy is confirmed
  5. Switch to sodium valproate

Ans. The key is D. Reduce lithium gradually and stop when pregnancy is confirmed.

405.    A pt presents with dysphagia and pain on swallowing. He has sore mouth and soreness in the corners of the mouth. What is the single most likely dx?

  1. Kaposi’s sarcoma
  2. Molluscum contagiosum
  3. CMV infection
  4. Candida infection
  5. Toxoplasma abscess

Ans. The key is D. Candida infection. [Candida is more common than CMV].

406.    A 30yo lady has epistaxis for 30mins. Her Hgb is normal, MCV normal, WBC normal, PT/APTT/Bleeding time are normal. Where is the defect?

  1. Plts
  2. Coagulation factor
  3. Sepsis
  4. Anatomical
  5. RBC

Ans. The key is D. Anatomical. [bleeding time, coagulation profile, Hb%, cell count and parameters are normal. So the cause of bleeding here is anatomical defect].

407.     Midpoint between the suprasternal notch and pubic symphysis. What is the single most appropriate landmark?

Fundus of the gallbladder

  1. Mcburney’s point
  2. Stellate ganglion
  3. Deep inguinal ring
  4. Transpyloric plane

Ans. The key is E. Transpyloric plane.

408.    Tip of the 9th costal cartilage. What is the single most appropri- ate landmark?

  1. Fundus of the gallbladder
  2. Deep inguinal ring
  3. Termination of the spinal cord
  4. Transpyloric plane
  5. Vena cava opening in the diaphragm

Ans. The key is A. Fundus of the gallbladder.

409.    A child complains of RIF pain and diarrhea. On colonoscopy, granular transmural ulcers are seen near the ileo-cecal junction. What should be the management?

  1. Sulfasalazine
  2. Paracetamol
  3. Ibuprofen
  4. Metronidazole

Ans. The key is A. Sulfasalazine. [Pain in RIF, diarrhea, granular trans- mural ulcers near the ileo-cecal junction points towards the diagnosis of Crohn’s diseas (predominantly ileo-cecal type)].

 

 

410.    A 60yo woman presents with acute onset of bone and back pain following a rough journey in a car. Exam: tenderness at mid-thoracic vertebra with spasm, she feels better once she bends forward. What is the single most probable dx?

  1. Osteoporotic fx verterbra
  2. Myofacial pain
  3. Whiplash injury
  4. MI
  5. Pancreatitis

Ans. The key is B. Myofacial pain. [Myofascial pain syndrome is a chronic pain disorder. In myofascial pain syndrome, pressure on sen- sitive points in your muscles (trigger points) causes pain in seemingly unrelated parts of your body. This is called referred pain. Myofascial pain syndrome typically occurs after a muscle has been contracted repetitively].

411.    A 70yo woman presents with recurrent episodes of parotid swelling. She complains of difficulty in talking and speaking and her eyes feel gritty on waking in the morning. What is the single most likely dx?

  1. C1 esterase deficiency
  2. Crohns disease
  3. Mumps
  4. Sarcoidosis
  5. Sjogrens syndrome

Ans. The key is E. Sjogrens syndrome. [parotid swelling, difficulty talking and speaking (due to dryness or less salive), eyes feeling gritty on waking in the morning due to dryness of eye are suggestive of Sjogrens syndrome].

412.    A 39yo woman has not had her period for 10months. She feels well but is anxious as her mother had an early menopause. Choose the single most appropriate initial inv?

  1. Serum estradiol
  2. Serum FSH/LH
  3. Serum progesterone
  4. None
  5. Transvaginal US

Ans. The key is B. Serum FSH/LH [here serum oestrogen is also im- portant as i) low oestrogen + low FSH + low LH suggest hypothalamic amenorrhoea and i) low oestrogen + high FSH + high LH suggest premature ovarian failure! So the main determinant is serum FSH/LH. Likely cause here is premature ovarian failure].

413.      A 50yo man with DM suddenly develops persistent crushing central chest pain radiating to the neck. What is the single most appropriate dx?

  1. Angina
  2. Costochondritis (tietz’s disease)
  3. Dissecting aneurysm
  4. MI
  5. Pulmonary embolism

Ans. The key is C. Dissecting aortic aneurism. Probably wrong key. Correct key should be D. MI. [The features described is insufficient and can be seen in both aortic dissection and MI. However dissection pain is described as tearing and crushing pain is often used for mi pain.

Both dissection and mi can have pain radiation to neck. History of di- abetes goes with mi as it is a recognized risk factor for mi. Some may argue in DM mi will be painless! But it is not always the case. MI is only painless when autonomic neuropathy becomes well established].

 

521 A 22yo man has rushed into the ED asking for help. He describes recurrent episodes of fearfulness, palpitations, faintness, hyperven- tilation, dryness of the mouth with peri-oral tingling and cramping of the hands. His symptoms last 5-10 mins and have worsened since their onset 3months ago. He is worried he may be having a heart attack. An ECG shows sinus tachycardia. What is the single most appropriate immediate intervention?

  1. High flow oxygen
  2. IV sedation
  3. Rebreathe into a paper bag
  4. Refer for anxiety management course
  5. Refer for urgent cardiology opinion

Ans. The key is C. Rebreathing into paper bag. [Patient has anxiety disorder (panic) which causes hyperventilation and CO2 washout leading to respiratory alkalosis. Symptoms will improve by rebreathing into paper bag as it will cause gradual increase of CO2 in paper bag and decrease the severity of respiratory alkalosis].

 

 

 

 

522. An 8yo boy has longstanding asthma. He has admitted with a severe episode and is tired and drowsy. He has not improved on oxy- gen, inhaled B2 agonist and IV hydrocortisone. CXR shows bilateral hyperinflation. He is too breathless to use a peakflow meter and is O2 sat <90%. What is the single most appropriate inv?

  1. CBG
  2. CXR
  3. CT chest
  4. Pulse oximetry
  5. Spirometry

Ans. The key is A. CBG. [It will point towards acidosis and indicate whether assisted ventilation is needed or not].

523.  A man was operated for colorectal ca. His pain is relieved with morphine 60mg bd PO but now he can’t swallow medications. What will be the next regimen of analgesic administration?

  1. Oxycodone
  2. Fentanyl patch
  3. Morphine 60mg IV/d
  4. Morphine 240mg IV/d

Ans. The key is B. Fentanyl patch. [Here S/C morphine 1/2 the dose of oral can be given (not present in option) or I/V morphine 1/3rd the oral dose can be given. Here I/V doses are not appropriate so we should go for B. Fentanyl patch as required morphine dose is known].

524.  Just above the mid-inguinal point. What is the single most appropriate landmark?

  1. Femoral artery pulse felt
  2. Mcburney’s point
  3. Stellate ganglion
  4. Deep inguinal ring
  5. Transpyloric plane

Ans. The key is D. Deep inguinal ring.

525.  5th ICS in the ant axillary line. What is the single most appropri- ate landmark?

  1. Apex beat
  2. Chest drain insertion
  3. Stellate ganglion
  4. Transpyloric plane
  5. Vena cava opening into the diaphragm

Ans. B. Chest drain insertion.

526.  A 34yo man with MS has taken an OD of 100 tablets of parac- etamol with intent to end his life. He has been brought to the ED for tx but is refusing all intervention.

  1. Assessment
  2. Evaluate pt’s capacity to refuse tx
  3. Establish if pt has a prv mental illness

Ans. The key is B. Evaluate patients capacity to refuse treatment.

527. A 23yo woman with painless vaginal bleeding at 36wks pregnan- cy otherwise seems to be normal. What should be done next step?

  1. Vaginal US
  2. Abdominal US
  3. Vaginal exam
  4. Reassurance

Ans. The key is B. Abdominal US. This is a wrong key. The correct key is A. Vaginal US.[Painless vaginal bleeding at 36 weeks indicates the diagnosis of placenta previa, which can be better evaluated by vaginal US].

 

528.  A 29yo lady admitted with hx of repeated UTI now developed hematuria with loin pain. What is the most probable dx?

  1. Acute pyelonephritis
  2. Chronic pyelonephritis
  3. UTI
  4. Bladder stone

Ans. The key is A. Acute pyelonephritis. [In a patient having hematuria and loin pain with history of repeated UTI suggest acute pyelonephri- tis].

529. A 45yo chronic smoker attends the OPD with complaints of persistent cough and copious amount of purulent sputum. He had hx of measles in the past. Exam: finger clubbing and inspiratory crepita- tions on auscultation. What is the single most likely dx/

  1. Interstitial lung disease
  2. Bronchiectasis
  3. Asthma
  4. COPD
  5. Sarcoidosis

Ans. The key is B. Bronchiectasis. [Persistent cough with copious purulent sputum and finger clubbing points towards the diagnosis of bronchiectasis. Severe lung infections such as tuberculosis (TB), whooping cough, pneumonia or measles can damage the airways at the time of infection. Bronchiectasis may then develop].

530.  A 68yo man has had malaise for 5 days and fever for 2 days. He has cough and there is dullness to percussion at the left lung base. What is the single most appropriate inv?

  1. Bronchoscopy
  2. CXR
  3. CT
  4. MRI
  5. V/Q scan

Ans. The key is B. CXR. [Given presentation is suggestive of pneumo- nia for which investigation of choice is CXR].

531. A 5yo child was admitted with hx of feeling tired and lethargic all the time, bleeding gums and sore throat since the last 3months. Exam: hepatosplenomegaly. What is the most probable dx?

  1. ALL
  2. AML
  3. CML
  4. CLL
  5. Lymphoma

Ans. The key is A. ALL. [Commonest leukemia in children is ALL. Bleeding gums (low platelet), feeling tired and lethargic, sorethroat,

 

hepatosplenomegally all are well known features of ALL].

 

 

532.  A 65yo man presents with back pain. Exam: splenomegaly and anemia. Blood: WBC=22, Hgb=10.9, Plt=100, ESR=25. He has been found to have Philadelphia chromosome. What is the single most likely dx?

  1. ALL
  2. AML
  3. CML
  4. CLL
  5. Lymphoma

Ans. The key is C. CML. [anaemia, raised WBC count, low platelet (platelet may be variable) are known features of CML, splenomegaly (particularly if massive) is very suggestive of CML and Philadelphia chromosome is characteristic of CML].

533.  A 24yo woman has 8wk amenorrhea, right sided pelvic pain and vaginal bleeding. She is apyrexial. Peritonism is elicited in the RIF. Vaginal exam reveals right sided cervical excitation. What is the most probable dx?

  1. Ectopic pregnancy
  2. Salpingitis
  3. Endometriosis
  4. Ovarian torsion
  5. Ovarian tumor

Ans. The key is A. Ectopic pregnancy. [Salpingitis, Endometriosis, ove- rian torsions do not associated with amenorrhoea. In ovarian tumour three main features are i) increased abdominal size and persistent bloating (not bloating that comes and goes) ii) persistent pelvic and abdominal pain iii) difficulty eating and feeling full quickly, or feeling nauseous. Patient with pelvic pain and vaginal bleeding, peritonism and cervical exitation obviously points towards Ectopic pregnancy].

534.  A 64 yo woman has been treated for breast cancer with tamoxi- fen. What other drug should be added to her tx regime?

  1. Bisphosphonates
  2. Calcium
  3. Vit D
  4. Calcitonin
  5. Phosphate binders

Ans. The key is A. Bisphosphonates. [ bisohosphonates reduce the risk of bone metastasis in cancers and is normally taken as adjuvant therapy in many types of tumours including breast cancer. Plus it prevents bone resorption].

535.  A 26yo woman with regular menses and her 28yo partner comes to the GP surgery complaining of primary infertility for 2yrs. What would be the single best investigation to see whether she is ovulat- ing or not?

  1. Basal body temp estimation
  2. Cervical smear
  3. Day2 LH and FSH
  4. Day21 progesterone
  5. Endometrial biopsy

Ans. The key is D. Day 21 progesterone. [Mid-luteal progesterone level to assess ovulation: If low, it may need repeating, as ovulation does not occur every month. The blood test is taken seven days before

the anticipated period, that is on day 21 of a 28-day cycle. However, this day will need to be adjusted for different lengths of cycle. Ref: patient.co.uk].

536.  A 10yo boy who takes regular high dose inhaled steroids for his longstanding asthma has been advised to use bronchodilators to control his acute attacks. His parents are unsure when should he use his bronchodilator. What is the single most appropriate inv?

  1. CXR
  2. None
  3. Peak flow rate diary
  4. Pulse oximetry

Ans. The key is C. Peak flow rate diary. [Peak flow rate diary shows diurnal variation. This diary shows when the bronchoconstriction remains worse and guides to use bronchodilators prior to that times].

537. A woman presented with blurred vision and intermittent clumsi- ness for 3m. Reflexes are brisk in her arm and optic disc is pale. What is the single most appropriate test to confirm dx?

  1. CSF analysis
  2. CT
  3. MRI
  4. EEG
  5. EMG

Ans. The key is C. MRI. [Features are suggestive of multiple sclerosis. Investigation of choice is gadolinium enhanced MRI].

538.  A 63yo man presents after having a seizure. Exam: alert, orien- tated, inattention on the left side and hyperreflexia of the arm. What is the most probable dx?

  1. Cerebral tumor
  2. Pituitary adenoma
  3. Cerebellar abscess
  4. Huntingtons chorea
  5. Parkinsonism

Ans. The key is A. Cerebral tumour.

539. A 40yo man with a 25y hx of smoking presents with progressive hoarseness of voice, difficulty swallowing and episodes of hemopty- sis. He mentioned that he used to be a regular cannabis user. What is the single most likely dx?

  1. Nasopharyngeal cancer
  2. Pharyngeal carcinoma
  3. Sinus squamous cell carcinoma
  4. Squamous cell laryngeal cancer
  5. Hypopharyngeal tumor

Ans. The key is D. Squamous cell laryngeal cancer. Chronic hoarseness is the most common early symptom.

Other symptoms of laryngeal cancer include pain, dysphagia, a lump in the neck, sore throat, earache or a persistent cough.

Patients may also describe breathlessness, aspiration, haemoptysis, fatigue and weakness, or weight loss.

(Patient.co.uk)

540.   A 30yo lady complains of intermittent diarrhea, chronic abdom- inal and pelvic pain and tenesmus. Sometimes she notices blood in her stool. Select the most likely cause leading to her symptoms?

  1. Inflammatory bowel disease
  2. Diverticulosis
  3. Irritable bowel disease
  4. Adenomyosis
  5. UTI

Ans. The key is A. Inflammatory bowel disease. [Tenesmus excludes diverticulitis, occasional blood in stool excludes irritable bowel disease. Features are not consistent with adenomyosis or UTI but suggestive of inflammatory bowel disease].

541. A 50yo lady with weak limbs when examined was found to have burn marks on finger tips, wasted and weak hands with diminished felexes. She also has weak spastic legs and dissociated sensory loss. What is the dx?

  1. MS
  2. Syringomyelia
  3. MND
  4. Guillian-barre
  5. Freidriech’s ataxia

Ans. The key is B. Syringomyelia. [weak limbs, burn mark on fingertip (as pain and temperature sensation are lost due to spinothalamic tract damage), wasted and weak hands with diminished reflexes, weak spastic legs with dissociated sensory loss are features suggestive of

 

Syringomyelia].

Common features are given below:

Sensory features: 1) loss of pain and temperature sensation 2) sensory loss is experienced over the arms, shoulders and upper body 3) light touch, vibration and position senses in the feet are affected as the syrinx enlarges into the dorsal column.

Motor features: (when lower motor neurons of the anterior horn cells are affected) 1) muscle wasting and weakness begins in the hands and then affects the forearms and shoulders. 2) tendon reflexes are lost. Autonomic involvement like bladder and bowel can occur. [patient. co.uk]

542.  A 23yo woman is being followed up 6wks after a surgical pro- cedure to evacuate the uterus following a miscarriage. The histology has shown changes consistent with a hydatidiform mole. What is the single most appropriate inv in this case?

  1. Abdominal US
  2. Maternal karyotype
  3. Paternal blood group
  4. Serum B-HCG
  5. Transvaginal US

Ans. The key is D. Serum -HCG. [When you are first diagnosed with a hydatidiform mole, your hCG level will be raised. When the hydatidi- form mole is treated (removed), the hCG level will usually return to a normal, non-pregnant amount and should remain so. If you develop GTN, the hCG level can remain elevated or continue to rise further. So, this blood test is a good way to check for the possible development of GTN (Gestational trophoblastic neoplasia).

543.  A 67yo man with hx of weight loss complains of hoarseness of voice. CT reveals opacity in the right upper mediastinum. He denied any hx of difficulty breathing. What is the single most appropriate inv?

  1. Laryngoscopy
  2. Bronchoscopy
  3. LN biopsy
  4. Bronchoalevolar lavage
  5. Barium swallow

Ans. The key is C. Lymph node biopsy. [There is weight loss and there is an opacity in right upper mediastinum. May indicate enlarged lymph node or lymphoma causing pressure on right recurrent laryngeal nerve resulting in horseness. As CT didn’t reveal any bronchial lesion and no breathing difficulty it is unlikely to be a bronchial pathology. So CT guided lymph node biopsy can reveal the diagnosis].

544.  A 52yo man whose voice became hoarse following thyroid sur- gery 1 wk ago shows no improvement. Which anatomical site is most likely affected?

  1. Bilateral recurrent laryngeal nerve
  2. Unilateral recurrent laryngeal nerve
  3. Unilateral external laryngeal nerve
  4. Bilateral external laryngeal nerve
  5. Vocal cords

Ans. The key is B. Unilateral recurrent laryngeal nerve.

 

 

545.  A 73yo male presents with a 12m hx of falls. His relatives have also noticed rather strange behavior of late and more recently he has had episodes of enuresis. Exam: disorientation to time and place, broad-based, clumsy gait. What is the most probable dx?

  1. Dementia
  2. Pituitary adenoma
  3. CVD
  4. Syringomyelia
  5. Normal pressure hydrocephalus

Ans. The key is E. Normal pressure hydrocephalus. [hx of falls and broad based clumsy gait (balance and gait disturbance), strange behavior and disorientation to time and place (due to dementia), episodes of enuresis (urinary incontinence) points towards normal pressure hydrocephalus. Classic triad of normal pressure hydrocepha- lus: i) gait abnormality ii) urinary incontinence and iii) dementia].

546.  A 75yo nursing home resident complains of headache, confu- sion and impaired vision for 4days. She has multiple bruises on her head. What is the most likely cause of confusion in this pt/

  1. Alcohol intoxication
  2. Infection
  3. Subdural hematoma
  4. Hypoglycemia
  5. Hyponatremia

Ans. The key is C. Subdural hematoma. [elderly patient with multiple bruises on her head suggest head injury which can lead to subdural hematoma. Headache, confusion and impaired vision for 4 days indicate subacute subdural hematoma (three pahges i) acute ii) sub- acute 3 – 7 days and iii) chronic 2 – 3 weeks]. Symptoms of subdural hematoma: a headache that keeps getting worse feeling and being sickconfusion

personality changes, such as being unusually aggressive or having rapid mood swings

feeling drowsy and finding it difficult to keep your eyes open

speech problems, such as slurred speech problems with your vision, such as double vision

paralysis (loss of movement) on one side of the body problems walking and frequent falls

seizures (fits)

loss of consciousness [Reference: nhs.uk]

547. A 50yo woman returned by air to the UK from Australia. 3days later she presented with sharp chest pain and breathlessness. Her CXR and ECG are normal. What is the single most appropriate inv?

  1. Bronchoscopy
  2. Cardiac enzymes
  3. CT
  4. MRI
  5. Pulse oximetry
  6. V/Q scan
  7. CTPA

Ans. The key is G. CTPA. [Prolonged plane journey is a recognized risk factor for thromboembolism and hence pulmonary embolism also!

Sharp chest pain and breathlessness after 3 days of plane journey is highly suggestive of pulmonary embolism the investigation of choice for which is CTPA].

548.  A tall thin young man has sudden pain in the chest and becomes breathless while crying. What is the single most appropriate inv?

  1. Cardiac enzymes
  2. CXR
  3. CT
  4. ECG
  5. V/Q scan

Ans. The key is B. CXR. [tall thin young men are particularly prone to develop pneumothorax. Sudden pain and breathlessness in this young man are highly suggestive of pneumothorax. So investigation of choice is CXR].

 

 

549.  A 21yo woman has had several sudden onset episodes of palpi- tations, sweating, nausea and overwhelming fear. On one occasion she was woken from sleep and feared she was going insane. There is no prv psychiatric disorder. What is the most probable dx?

  1. Pheochromocytoma
  2. Panic disorder
  3. GAD
  4. Phobia
  5. Acute stress disorder

 

Ans. The key is B. Panic disorder. [here closest d/d to panic disorder is pheochromocytoma. But in pheochromocytoma the most important feature is resistant hypertension and other important features are headache and abdominal pain which all are absent here. Moreover overwhelming fear is more suggestive of panic disorder. Hence here the diagnosis is B. Panic disorder].

550.  A 55yo woman with a persistent cough and hx of smoking de- velops left sided chest pain exacerbated by deep breathing with fever and localized crackles. What is the single most appropriate dx?

  1. Dissecting aneurysm
  2. Pericarditis
  3. Pneumonia
  4. Pneumothorax
  5. Pulmonary embolism

Ans. The key is C. Pneumonia. [chest pain exacerbated with deep breathing, fever and localized crackles are highly suggestive of pneu- monia].

551. A 40yo woman complains of dysphagia for both solids and liquids. She sometimes suffers from severe retrosternal chest pain. Barium swallow reveals a dilated esophagus which tapers to a fine distal end. What is the best management strategy?

  1. Reassurance
  2. Antispasmodics
  3. Dilatation of the LES
  4. Endoscopic diverticulectomy
  5. Barium swallow

Ans. The key is C. Dilatation of LES. [Dysphagia for both solids and liquids suggest neuromuscular dysphagia while dysphagia only for solid suggests mechanical obstruction. Here features are consistent with achalasia for which lower oesophageal sphincter dilation (balloon dilatation) is a treatment modality].

552.  A 38yo female G4 at 32wks of pregnancy presented with thick white marks on the inside of her mouth for 3wks. Her mouth includ- ing her tongue appeared inflamed on examination. She smokes 20 cigarettes/day despite advice to quit. She attends her ANC regularly. What is the most probable dx?

  1. Lichen planus
  2. Aphthous ulcer
  3. Smoking
  4. Candidiasis
  5. Leukoplakia

Ans. The key is D. Candidiasis. [lichen planus may have lace like appearance and not thick white mark. Aphthous ulcer has yellowish floor and surrounded by erythematous halo. Smoking may cause tongue coating but not like thick white mark on the inside of mouth.

Leukoplakia is with raised edges/Bright white patches and sharply de- fined and cannot be rubbed out like candida patch; here also inflamed tongue points towards infection. So candidiasis is the most probable option].

553.  A 69yo woman has had a stroke. Her left upper and lower limbs are paralyzed and she is having difficulty in speaking. Which anatom- ical site is most likely affected?

  1. Hippocampus
  2. Cerebellum
  3. Internal capsule
  4. Thalamus
  5. Brain stem

Ans. The key is C. internal capsule. [Hippocampal lesion causes mainly memory impairment. Cerebellum has its diagnostic features that is not present here. Thalamic lesion can lead to impairment of arousal, orientation, learning and memory, facial paresis, language deficit, hemispatial neglect, hemisensory loss, hemiparesis, hemiataxia and visual field defect. Brainstem stroke causes impaired consciousness, disorder of blood pressure, and breathing abnormality. Given picture is typical of lesion in internal capsule].

554.  A 72yo man brought to the ED with onset of paraplegia follow- ing a trivial fall. He was treated for prostatic malignancy in the past. What is the single most probable dx?

  1. Paget’s disease
  2. Osteoporotic fx of vertebre
  3. Secondary
  4. Multiple myeloma
  5. Spondylosis

Ans. The key is C. Secondary. [In male osteoporotic fracture is less common. As patient had prostatic malignancy pathological fracture from secondary metastasis to bone (here vertebra leading to paraple- gia) is more common].

 

 

555.  A 14yo girl has developed an itchy, scaly patch on her scalp. She had a similar patch that cleared spontaneously 2yrs ago. Her aunt has a similar undiagnosed rash on the extensor aspects of her elbows and knees. What is the single most likely dx?

  1. Eczema
  2. Fungal infection
  3. Impetigo
  4. Lichen planus
  5. Psoriasis

Ans. The key is E. Psoriasis. [itchy, scaly patch on scalp are classic pre- sentation of scalp psoriasis. Her aunts presentations (similar rash on

extensor aspects of her elbow and knees) are suggestive of psoriasis. It is thought to be an immunological disease].

556.  A pt after transurethral prostatic biopsy. What electrolyte imbalance can he develop?

  1. Hyperkalemia
  2. Hyponatremia
  3. Hypocalcemia
  4. Hypernatremia
  5. Hypercalcemia

Ans. The key is B. Hyponatremia. [Use of fluid for bladder irrigation may lead to dilutional hyponatremia].

557. A 28yo woman has been admitted at 38wks gestation. Her BP=190/120mmHg and proteinuria +++. Immediately following admission she has a grand-mal seizure. What is the single most appropriate initial management?

  1. Diazepam IV
  2. Fetal CTG
  3. Hydralazine IV
  4. Immediate delivery
  5. Magnesium sulphate IV

Ans. The key is E. Magnesium sulphate IV. [patient has established eclampsia. So she should be treated with Magnesium sulphate as with 4g magnesium sulfate in 100mL 0.9% saline IVI over 5min + mainte- nance IVI of 1g/h for 24h. Beware depressed respiration. If recurrent seizure give 2g IVI magnesium sulfate over 5 min. Check tendon reflexes and respiratory rate every 15min. Stop magnesium sulfate

IVI if respiratory rate <14/min or tendon reflex loss, or urine output

<20mL/h]. [OHCS, 9th edition, page 49].

558.  A 27yo woman had pre-eclampsia and was delivered by C-sec- tion. She is now complaining of RUQ pain different from wound pain. What inv will you do immediately?

  1. Coagulation profile
  2. LFT
  3. Liver US
  4. MRCP
  5. None

Ans. The key is B. LFT. [In a post ceasarian patient having pre-ec- lampsia RUQ pain different from wound pain points towards the diag- nosis of HELLP syndrome. So immediate investigation to be done is LFT]. [OHCS, 9th edition, page 26].

 

559. A 10yo girl has been referred for assessment of hearing as she is finding difficulty in hearing her teacher in the class. Her hearing

tests show: BC normal, symmetrical AC threshold reduced bilaterally, weber test shows no lateralization. What is the single most likely dx?

  1. Chronic perforation of tympanic membrane
  2. Chronic secretory OM with effusion
  3. Congenital sensorineural deficit
  4. Otosclerosis
  5. Presbycusis

Ans. The key is B. Chronic secretory OM with effusion.

 

 

560.  A thin 18yo girl has bilateral parotid swelling with thickened cal- luses on the dorsum of her hand. What is the single most likely dx?

  1. Bulimia nervosa
  2. C1 esterase deficiency
  3. Crohn’s disease
  4. Mumps
  5. Sarcoidosis

Ans. The key is A. Bulimia nervosa. [18 year thin girl, bilateral parotid swelling and thickened calluses on the knuckles from self induced vomiting are suggestive of Bulimia nervosa. Bulimia often is associat- ed with bilaterak parotid swelling (parotid hypertrophy)].

561. A 48yo presents with severe chest pain since the last 40mins. In the ED he is given oxygen, GTN, morphine. ECG=ST elevation. Bloods=increased troponin levels. What is the next step of manage- ment?

  1. Beta blockers
  2. Percutaneous angiography
  3. Anticoagulant & heparin
  4. Clopidogrel
  5. Aspirin

Ans. The key is B. Percutaneous angiography. This is a wrong key! Correct key is E. Aspirin. [In any case of IHD, most important initial management is aspirin].

562.  A 34yo female presents with a foul smelling discharge. What set of organisms are we looking for to be treated here?

  1. Chlamydia, gonorrhea
  2. Chlamydia, gardenella
  3. Chlamydia, gonorrhea, gardenella
  4. Gonorrhea, gardenella
  5. Gardenella only

Ans. The key is E. Gardenella only. [Here foul smelling discharge is

caused by gardenella. So most acceptable answer is E. Gardenella only].

563.  A 6wk formula fed baby boy is found at the child health surveil- lance to be deeply jaundiced. His weight gain is poor and his stools are pale. What is the most likely dx?

  1. Galactosemia
  2. Biliary atresia
  3. G6PD deficiency
  4. Rh incompatibility
  5. Congenital viral infection

Ans. The key is B. Biliary atresia. [Deep jaundice at 6th week with pale stools suggests obstructive jaundice. So most likely diagnosis here is biliary atresia].

 

 

564.  A 45yo man with colon cancer now develops increased thirst, increased frequency in urination and weight loss. His fasting blood glucose=9mmol/L. what is the most appropriate management?

  1. Oral hypoglycemic
  2. Insulin long acting
  3. Short acting insulin before meal
  4. IV insulin
  5. Subcutaneous insulin

Ans. The key is A. Oral hypoglycemic. [Increased thirst and increased frequency in urination along with weight loss is suggestive of DM sup- ported by fasting blood glucose of 9 mmol/L. At the age of 45 most likely type of diabetes is NIDDM or type 2 DM which is treated by oral hypoglycemic agents].

565.  A 34yo man from Zimbabwe is admitted with abdominal pain to the ED. An AXR reveals bladder calcification. What is the most likely cause?

  1. Schistosoma mansoni
  2. Sarcoidosis
  3. Leishmaniasis
  4. TB
  5. Schistosoma hematobium

Ans. The key is E. Schistosoma hematobium. [Bladder involvement is caused by Schistosoma hematobium while Schistosoma mansoni causes intestinal disease].

566.  A 6yo came with full thickness burn. He is crying continuously. What is the next step of management?

  1. Refer to burn unit
  2. IV fluid stat
  3. Antibiotic
  4. Analgesia
  5. Dressing

Ans. The key is analgesia. This is a wrong key. Correct key should be

  1. IV fluid [Here already mentioned full thickness burn which is painless. Child often cry from anxiety for hypoxaemia and hypovolae- mia rather than pain. The patient then responds better to oxygen or increased fluid administration rather than to narcotic analgesics. Ref: patient.info].

 

567. A 78yo nursing home resident is revived due to the development of an intensely itchy rash. Exam: white linear lesions are seen on the wrists and elbows and red papules are present on the penis. What is the most appropriate management?

  1. Topical permethrin
  2. Referral to GUM clinic
  3. Topical betnovate
  4. Topical ketoconazole
  5. Topical selenium sulphidehyosine

Ans. The key is A. Topical permathrine. [The intensely itchy rash, scratch marks and burrows on wrist and elbow red papules on penis are suggestive of scabies. Topical permethrine are used to treat it].

 

 

568.  A 4yo has earache and fever. Has taken paracetamol several times. Now it’s noticed that he increases the TV volume. His pre- school hearing test shows symmetric loss of 40db. What is the most likely dx?

  1. OM with effusion
  2. Otitis externa
  3. Cholesteatoma
  4. CSOM
  5. Tonsillitis

Ans. The key is A. OM with effusion.

 

569. A pt presents with gradual onset of headache, neck stiffness, photophobia and fluctuating LOC. CSF shows lymphocytosis but no organism on gram stain. CT brain is normal. What is the single most likely dx?

  1. Hairy leukoplakia
  2. TB
  3. CMV infection
  4. Candida infection
  5. Cryptococcal infection

Ans. The key is B. TB. [Fungal meningitis can also present like this but it is much more rare. Moreover negative gram stain excludes fungal cause here. Hence TB meningitis is more acceptable answer].

570.  An 18m boy has been brought to the ED because he has been refusing to move his left arm and crying more than usual for the past 24h. He has recently been looked after by his mother’s new bf while she attended college. Assessment shows multiple bruises and a fx

of the left humerus which is put in plaster. What is the single most appropriate next step?

  1. Admit under care of pediatrician
  2. Discharge with painkillers
  3. Follow up in fx clinic
  4. Follow up in pediatric OPD
  5. Follow up with GP

Ans. The key is A. Admit under care of pediatrician. [This is NAI. So the child cannot be handover to the risk again and should be admitted to protect him from further injury done by mothers boyfriend while serial x-rays and relevant investigations done and asked for child protection unit’s help].

571.A 74yo female presents with headache and neck stiffness to the ED. Following a LP the pt was started on IV ceftriaxone. CSF culture = listeria monocytogenes. What is the appropriate tx?

  1. Add IV amoxicillin
  2. Change to IV amoxicillin + gentamicin
  3. Add IV ciprofloxacin
  4. Add IV co-amoxiclav
  5. Continue IV ceftriaxone as mono-therapy

Ans. The key is B. Change to IV amoxicillin + gentamycin. [From the given option B is the most acceptable. However Ampicillin + gentamy- cin is the drug combination of choice].

572. A pt presents with fever, dry cough and breathlessness. He is tachypneic but chest is clear. Oxygen saturation is normal at rest but drops on exercise. What is the single most likely dx?

  1. CMV infection
  2. Candida infection
  3. Pneumocystis carinii infection
  4. Cryptococcal infection
  5. Toxoplasma abscess

Ans. The key is C. Pneumocystis carinii infection. [Fever, dry cough, breathlessness, tachypnoea with clear chest is seen in pneumocystis carinii pneumonia. Normal oxygen saturation which drops on exercise is characteristic of pneumocystis carinii pneumonia].

573. A 14yo boy fell and hit his head in the playground school. He didn’t lose consciousness. He has swelling and tenderness of the right cheek with a subconjuctival hemorrhage on his right eye. What is the most appropriate initial inv?

  1. CT brain
  2. EEG
  3. MRI
  4. Skull XR
  5. Facial XR

Ans. The key is E. Facial X-ray. [There is no sign of basal fracture (Panda sign is suggestive of basal fracture, not subconjunctival hem- orrhage- unless posterior border of hemorrhage is not visible) and no indication of CT scan (there are some criteria for CT scan which are not fulfilled here)].

 

574.  A 15m child is due for his MMR vaccine. There is a fam hx of egg allergy. He is febrile with acute OM. What is the single most appro- priate action?

  1. Defer immunization for 2wks
  2. Don’t give vaccine
  3. Give half dose of vaccine
  4. Give paracetamol with future doses of the same vaccine
  5. Proceed with standard immunization schedule

Ans. The key is A. Defer immunization for 2 wks.

575. A 33yo lady with Hodgkin’s lymphoma presents with temp=40C, left sided abdominal pain and lymphadenitis. Blood was taken for test. What will you do next?

  1. Wait for blood test
  2. Start broad spectrum IV antibiotics
  3. Oral antibiotics
  4. CBC
  5. Monitor pyrexia

Ans. The key is B. Start broad spectrum IV antibiotics. [The patient is immunocompromized with signs of infection (temp=40 C, left sided abdominal pain and lymphadenitis) broad spectrum IV antibiotic should be started empirically while waiting for blood reports].

576.   A 40yo man with marked weight loss over the preceding 6m has bilateral white, vertically corrugated lesion on the lateral surfac- es of the tongue. What is the single most likely dx?

  1. C1 esterase deficiency
  2. Crohns disease
  3. HIV disease
  4. Sarcoidosis
  5. Sjogren’s syndrome

Ans. The key is C. HIV disease. [The lesion described is leukoplakia which is likely association of HIV disease].

577. A 3m baby was miserable and cried for 2h following his 1st rou- tine immunization with DTP, HiB and meningitis. What is the single most appropriate action?

  1. Defer immunization for 2wks
  2. Don’t give vaccine
  3. Give half dose of vaccine
  4. Give paracetamol with future doses of the same vaccine
  5. Proceed with standard immunization schedule

Ans. The key is E. Proceed with standard immunization schedule.

 

578.  A 65yo man with HTN develops gingival hyperplasia. What is the single most likely dx?

  1. ACEi
  2. Beta blockers
  3. Crohns disease
  4. Nifedipine
  5. Sarcoidosis

Ans. The key is D. Nifedipine. [Gingival hyperplasia is a recognized side effect of calcium channel blockers].

 

 

579. A 65yo woman is undergoing coronary angiography. What mea- sure will protect her kidneys from contrast?

  1. Furosemide
  2. Dextrose
  3. 45% saline
  4. 9% saline

Ans. The key is D. 0.9% saline.

580.   An 83yo woman who is a resident in a nursing home is admit- ted to hospital with a 4d hx of diarrhea. She has had no weight loss or change in appetite. She has been on analgesics for 3wks for her back pain. She is in obvious discomfort. On rectal exam: fecal impac- tion. What is the single most appropriate immediate management?

  1. Codeine phosphate for pain relief
  2. High fiber diet
  3. Oral laxative
  4. Phosphate enema
  5. Urinary catheterization

Ans. The key is D. Phosphate enema. [In feacal impaction oral laxative is not the choice but phosphate enema is the best option here].

581. A 26yo woman being treated for a carcinoma of the bronchus with steroids presents with vomiting, abdominal pain and sudden falls in the morning. What is the most specific cause for her symp- toms?

  1. Steroid side effects
  2. Postural hypotension
  3. Adrenal insufficiency
  4. Conn’s disease
  5. Cushing’s disease

Ans. The key is C. Adrenal insufficiency. [Patients on steroid develop suppression of hypothalamic pituitary adrenal axis and frequently may lead to adrenal insufficiency (here vomiting, abdominal pain and sud- den falls in the morning secondary to postural hypotension on getting

up from bed points towards the diagnosis of adrenal insufficiency)].

582.  A 78yo woman presents with unilateral headache and pain on chewing. ESR=70mm/hr. She is on oral steroids. What is the appro- priate additional tx?

  1. Bisphosphonates
  2. HRT
  3. ACEi
  4. IFN
  5. IV steroids

Ans. The key is A. Bisphosphonates. [The elderly lady with giant cell arteritis is getting high dose steroid which can lead to demineraliza- tion and osteopenia or osteoporosis. So to prevent this bisphospho- nates are given].

 

 

 

 

 

583.  A 30yo man is suffering from fever, rash and photophobia. Doctors are suspecting he is suffering from meningitis. Which is the best medication for this condition?

  1. Ampicillin
  2. Cefotaxime
  3. Tetracycline
  4. Acyclovir
  5. Dexamethasone

Ans. The key is B. Cefotaxime. [The patient is getting probable me- ningococcal meningitis. Before confirming the diagnosis suggested treatment is, where the organism is unknown:

  • <55yrs: cefotaxime 2g/6h slow
  • >55yrs: cefotaxime as above + ampicillin 2g IV/4h (for Listeria). So in given case Cefotaxime is the Ref: OHCM, 9th edition, page 832].

584.  A 15yo girl was admitted with anemia, chest infection and thrombocytopenia. She was treated and her symptoms had re- gressed. She was brought again with fever and the same symptoms a few days later. She also seems to have features of meningitis. What is the most likely dx?

  1. AML
  2. ALL
  3. Aplastic anemia
  4. CML
  5. CLL

Ans. The key is B. ALL. [The age supports the diagnosis of ALL along with the given picture. Same picture can happen in aplastic anaemia but there is not a single factor mentioned in favour of it. So ALL can be taken as best option in the given scenario].

585.  A pt was admitted to the ED after a head injury. When examined on arrival his GCS=15 and then at night his GCS deteriorated to 12. What investigation should be done?

  1. CT head
  2. XR skull
  3. IV mannitol
  4. Drill a burr hole
  5. Shift to OR

Ans. The key is A. CT head. [Initial GCS 15 followed by later GCS 12 are suggestive of intracranial haemorrhage. So the best investigation to be done is CT head].

586.  A 4yo boy who prv had normal hearing, has a mild earache re- lieved by paracetamol. He has been noticed to turn up the vol on the TV. He has bilateral dull tympanic membranes. His preschool hearing test shows symmetrical loss of 40dB. What is the single most likely dx?

  1. Acute otitis externa
  2. Acute OM
  3. Ear wax
  4. Foreign body
  5. OM with effusion

Ans. The key is OM with effusion.

 

587. An 18yo man presents to his GP with thirst and polyuria. Some 6m ago he had a significant head injury as the result of a RTA. He is referred to the local endocrine clinic. Which of the following results would be the most useful in confirming the dx of diabetes insipidus after a water deprivation test (without additional desmopressin)?

  1. Plasma sodium of 126mmol/l
  2. Plasma sodium of 150mmol/l
  3. Plasma osmolality of 335mosmol/kg and urine osmolality of 700mosmol/kg
  4. Plasma osmolality of 280mosmol/kg and urine osmolality of 700mosmol/kg
  5. Plasma osmolality of 335mosmol/kg and urine osmolality of 200mosmol/kg

Ans. The key is E. Plasma osmolality of 335 mosmol/kg and urine osmolality of 200 mosmol/kg. [In DI plasma osmolality will be high due to excess fluid loss with urine and urine osmolality will be low due to polyuria. Hence E. is the best option here(normal plasma osmo- lality 275-295 mosmol/kg and normal urine osmolality is 300-900 mosmol/kg)].

588.  A 75yo man has left-sided earache and discomfort when he swallows. There is ulceration at the back of his tongue and he has a palpable non-tender cervical mass. What is the single most likely dx?

  1. Acute mastoiditis
  2. Dental abscess
  3. Herpes zoster infection
  4. Oropharyngeal malignancy
  5. Tonsillitis

Ans. The key is D. Oropharyngeal malignancy. [Pain ear and discom- fort during swallowing, ulceration at the back of the tongue and pal- pable non tender cervical lymphnode is suggestive of oropharyngeal malignancy. Acute mastoiditis and tonsillitis will not cause tongue ulcer. Similarly dental abscess will not cause tongue ulcer. In herpes zoster pain and vesicle will be in the affected nerve distribution].

589.  A 42yo man has been tired and sleepy for the last few weeks in the morning. His work has started getting affected as he feels sleepy in the meetings. His BMI=36. What is the single most likely dx?

  1. Idiopathic hypersomnia
  2. Narcolepsy
  3. Chest hyperventilation syndrome
  4. OSAS
  5. REM-related sleep disorder

Ans. The key is D. OSAS. [Day time somnolence and obesity (high BMI of 36) points the diagnosis of OSAS].

590.  A 35yo pregnant woman has been having tingling and numb- ness of her thumb, index and middle fingers for a while. She has been treated with local steroids but it hasn’t helped her much and now she has planned to undergo a surgical procedure. Which of the following structures will be incised?

  1. Flexor digitorum profundus
  2. Transverse carpal ligament
  3. Palmar aponeurosis
  4. Extensor retinaculum

Ans. The key is B. Transverse carpal ligament. [This is a case of carpal tunnel syndrome. So transverse carpal ligament is the structure which will be incised].

591. A 58yo pt presents with altered bowel habits and bleeding per rectum. Exam and sigmoidoscopy showed an ulcer. What is the single most likely dx?

  1. Colorectal carcinoma
  2. Celiac disease
  3. Crohns disease
  4. UC
  5. IBS

Ans. The key is A. Colorectal carcinoma. [Age, altered bowel habits, bleeding per rectum and isolated ulcer on sigmoidoscopy suggest colorectal cancer].

592.  A mother is concerned that her 18m son has a vocabulary of ten words but can’t form a sentence. What is the best management strategy?

  1. Arrange hearing test
  2. Assess developmental milestones
  3. Reassurance
  4. Refer to speech therapist
  5. MRI brain

Ans. The key is C. Reassurance. [Two words joining can be done in 2yrs and inability to form a sentence in 18 months is quite normal. So the option is reassurance].

593.  A 55yo man has weight loss, dyspnea and syncope. He smokes 20 cigarettes/day. Inv confirms squamous cell carcinoma in the left bronchus. What is the single most likely biochemical abnormality to be a/w the condition?

  1. Hypercalcemia
  2. Hyperkalemia
  3. Hypernatremia
  4. Hypocalcemia
  5. Hypomagnesium

Ans. The key is A. Hypercalcemia. [Hypercalcemia is common in squa- mous cell carcinoma].

594.  A 72yo man presents with intermittent difficulty in swallowing with regurgitation of stale food materials. Sometimes he wakes up at night with a feeling of suffocation. Choose the single most likely cause of dysphagia?

  1. Benign structure
  2. Esophageal carcinoma
  3. Esophageal spasm
  4. Pharyngeal pouch
  5. Systemic sclerosis

Ans. The key is D. Pharyngeal pouch. [In benign stricture, esophageal carcinoma and systemic sclerosis there is persistent dysphagia not intermittent. In oesophageal spasm there may be intermittent dys- phagea but there will be no regurgitation of stale food material and no nocturnal regurgitation in recumbency. The clinical picture described well fits with pharyngeal pouch].

595.  A 9m child is brought to the ED with an irreducible firm swelling which descended into the left groin when the child has been crying. Exam: both testicles are palpable in the scrotum. What is the most appropriate management strategy?

  1. Reassurance
  2. Emergency herniotomy
  3. Elective herniotomy
  4. Emergency herniotomy + orchidopexy
  5. Elective herniotomy + orchidopexy

Ans. The key is C. Elective herniotmy. [If there was features of strangulation we would go for emergency herniotomy. But as only irreducible we shall proceed to elective herniotomy].

596.  A 37yo woman was admitted for femur fx repair after a RTA. On the 4th post-op day she became confused and starts picking on her bed sheets and complains of seeing spiders all over. What is the most likely dx?

  1. Delirium tremens
  2. Wernickes encephalopathy
  3. Korsakoffs psychosis
  4. Psychotic depression

Ans. The key is A. Delerium tremens. [withdrawal of alcohol due to hospital admission lead to delirium tremens. Warnicke’s encephalop- athy has triad of i) mental confusion ii) abnormal eye movements &

iii) unsteady gait. Korsakoff’s syndrome cannot be diagnosed until the person has stopped drinking alcohol for several weeks, to enable the immediate symptoms of alcohol intoxication and withdrawal to subside. Features of Korsakoffs psychosis i) dementia, loss of short term memory ii) difficulty in acquiring new information or learning

 

new skills iii) personality change iv) confabulation].

597. A 36yo pt came with diarrhea, bleeding, weight loss and fistula. What is the single most likely dx?

  1. Celiac disease
  2. Crohns disease
  3. UC
  4. IBS

Ans. The key is B. Crohns disease.

598.  A 72yo woman who is taking loop diuretics is suffering from palpitations and muscle weakness. What is the electrolyte imbalance found?

  1. Na+ 130mmol/l, K+ 5mmol/l
  2. Na+ 130mmol/l, K+ 5mmol/l
  3. Na+ 140mmol/l, K+ 5mmol/l
  4. Na+ 150mmol/l, K+ 5mmol/l
  5. None

Ans. The key is A. Na+ 130mmol/l, K+ 2.5mmol/l. [Loop diuretic caus- es hypokalemia and hyponatremia].

599.  A 60yo diabetic pt on anti-diabetic medication developed diar- rhea. What is the most likely cause for his diarrhea?

  1. Autonomic neuropathy
  2. Infective
  3. Celiac disease
  4. Crohns disease

Ans. The key is A. Autonomic neuropathy.

600.   Which artery runs in the anterior inter-ventricular groove?

  1. Acute marginal branch
  2. Left ant descending artery
  3. Septal branches
  4. Circumflex artery
  5. Right coronary artery

Ans. B. Left anterior descending artery.

601. A mother presents with her 12m daughter. The child has no meaningful words, is unable to sit unaided and can’t play with her toys. She doesn’t laugh and has poor interaction with her siblings. What is the best management strategy?

  1. Arrange hearing test
  2. Assess developmental milestones
  3. Reassure
  4. Refer to speech therapist
  5. MRI brain

Ans. The key is B. Assess developmental milestones. [At 12 month one word should be said clearly, in 8 months child can sit inde- pendently, smiles at 2 months and plays with toys since early infancy. So she needs to assess developmental milestones].

602.  A pt presents with progressive visual deterioration. Exam: large, multiple cotton wool spots in both eyes. What is the single most likely dx?

  1. Kaposi’s sarcoma
  2. Cryptosporidium
  3. CMV infection
  4. Pneumocystis carinii infection
  5. Cryptococcal infection

Ans. The key is C. CMV infection. [Large multiple cotton wool spots are seen in early stage of CMV retinitis].

603.  A 53yo had a dental extraction after which he recently had a mi- tral valve prolapse, high temp of 39C, cardiac failure and new cardiac murmur. What is the single most likely dx?

  1. Atheroma
  2. Congenital
  3. Regeneration
  4. Infection
  5. Neoplastic

Ans. The key is D. Infection. [Infective endocarditis].

604.   A 12yo boy with a hx of fall on an outstretched hand was brought to the ED with swelling and pain around the elbow. His radial nerve was affected. What is the type of fx?

  1. Angulated fx
  2. Epiphyseal fx
  3. Compound fx
  4. Spiral fx

Ans. The key is D. Spiral fracture. It is wrong key. The correct option should be A. Angulated (supracondylar fracture). [Around 50% inter- osseous nerve lesions occur in supracondylar fracture whereas 25% shows radial nerve damage. If the fracture is spiral fracture of lower

third of humerus it causes nerve damage in 18% almost all of which are radial nerve lesion. However as the fracture is around the elbow (not above) it is supracondylar fracture].

605.  A 32yo lady complains that she hears everyone saying that she is an evil person. What type of hallucinations is she suffering from?

  1. 2nd person auditory hallucinations
  2. 3rd person auditory hallucinations
  3. Echo de la pense
  4. Gedankenlautwerden

Ans. The key is B. 3rd person auditory hallucinations. [Third person hallucinations are auditory hallucinations in which patients hear voices talking about themselves, referring to them in the third person, for example “he is an evil person”].

606.   A 65yo woman had an excision of colonic tumor 3yrs ago. Now she is losing weight and feels lethargic. Exam: pale but no abdominal findings. What is the most appropriate inv?

  1. CA 125
  2. CA 153
  3. CA 199
  4. CEA
  5. AFP

Ans. The key is D. CEA. [CA 125 = ovarian cancer; CA 153 = cancer breast; CA 199 = pancreatic cancer; CEA = colorectal carcinoma; AFP = hepatocellular carcinoma].

607.  A 46yo African-Caribbean man is found to have BP=160/90mmHg on 3 separate occasions. What is the best initial tx?

  1. ACEi
  2. Beta-blockers
  3. ARBs
  4. None
  5. CCB

Ans. The key is E. CCB. [If age less than 55 years but Afro-Caribbean origin then CCB].

 

 

608.   A 39yo woman will undergo tubal sterilization and she wants to know the failure rate of this type of sterilization.

  1. 1:50
  2. 1:200
  3. 1:500
  4. 1:1000
  5. 1:5000

 

Ans. The key is B. 1:200.

609.  Which of the following reflexes and innervating spinal nerves are correctly paired?

  1. Anal reflex – S1
  2. Ankle jerk – L5
  3. Biceps jerk – C7 & C8
  4. Knee jerk – L3 & L4
  5. Triceps jerk – T1

Ans. The key is D. Knee jerk – L3 & L4. [Anal reflex – S2-4; Ankle jerk

– S1-2; Biceps jerk – C5-6; Knee jerk – L3-4; Triceps jerk – C7].

610. A 62yo man with rheumatoid arthritis struck his hand against a door. He subsequently found that although he could extend the

interphalangeal joint of his right thumb, the MCP joint of the thumb remained flex. What is the single most likely tendon to have been damaged?

  1. Extensor carpi ulnaris
  2. Extensor digitorum
  3. Extensor indicis
  4. Extensor pollicis brevis
  5. Extensor pollicis longus

Ans. The key is D. Extensor pollicis brevis. [Action of extensor pollicis brevis = extension of thumb at metacarpophalangeal joint. Extensor pollicis longus = extends the terminal phalanx of the thumb].

611.        A 68yo lady complains of falls to the ground without any warn- ing, maintains consciousness and no confusion. She says this has occurred at number of times. What is the dx?

  1. Stokes Adams attack
  2. Hypoglycemia
  3. Vasovagal syncope
  4. Drop attacks
  5. Epilepsy

Ans. The key is D. Drop attacks. [Drop attacks are sudden sponta- neous falls while standing or walking, with complete recovery in sec- onds or minutes. There is usually no recognized loss of consciousness, and the event is remembered].

612.A 50yo man complains of being pursued by the police for a crime he denies committing. He has poor concentration and impaired

short-term memory. He admits to drinking large amounts of alcohol for the last 20yrs. What is the most probable dx?

  1. Dementia
  2. Hallucination
  3. Wernicke’s encephalopathy
  4. Schizophrenia
  5. Korsakoff psychosis

Ans. The key is E. Korsakoff psychosis. [Dementia, i.e. short term memory loss is seen in korsakoff psychosis].

613.  A pt with prv hx of HTN, the membranes have ruptured and the cervix is 3cm dilated. 4h later on examination showed that the cervix was still 3cm dilated. What is the single most appropriate manage- ment for her labor?

  1. Repeat vaginal examination in 4h
  2. CTG
  3. C-section
  4. External rotation
  5. IV syntocin drip

Ans. The key is E. IV syntocinon drip. [There is no progress of labour in 4 hours. Hence syntocinon drip should be given].

614. A 6yo girl has had 2 short episodes of cough and wheeze over the last 12m. These 2 acute episodes responded quickly to broncho- dilator, she has no symptoms or abnormal physical signs. She has slight eczema and her mother has asthma. What is the single most appropriate inv?

  1. CXR
  2. Peak flow rate diary
  3. Pulse oximetry
  4. Spirometry
  5. Sweat test

Ans. The key is D. Spirometry. [spirometry is the preferred initial test (if available) to assess the presence and severity of airflow obstruction less effort dependent and more repeatable though less applicable in acute severe asthma].

615. A 45yo man had recently started taking anti-HTN therapy. 6m later his RBS=14mmol/l. Which single drug is most likely to have caused this?

  1. Amlodipine
  2. Bendroflumethiazide
  3. Doxazosin
  4. Lorsartan
  5. Ramipril

Ans. The key is B. Bendroflumethiazide. [High blood sugar is a well known side effect of bendroflumethiazide].

 

 

616. A 27yo waitress has pelvic pain, dysmenorrhea and increasingly heavy periods. She also complains of dyspareunia. There is general- ized pelvic tenderness without peritonism. Pelvic US is normal. What is the most likely dx?

  1. Endometriosis
  2. Uterine fibroid
  3. Pelvic congestion syndrome
  4. PID
  5. Tubal pregnancy

Ans. The key is C. Pelvic congestion syndrome. [In pelvic congestion syndrome there develops varicose veins in the lower abdomen from prolonged standing (as occurred here in a waitress who remains standing for long) with some pain syndromes like pelvic pain, dys- menorrea, dyspareunia and generalized pelvic tenderness without peritonism. Also there may be associated menorrhagia].

617.A 14yo girl is clinically obese. She has not started her periods yet and has severe acne. Among her inv, a high insulin level is found. What is the most probable dx?

  1. Cushing’s syndrome
  2. Grave’s disease
  3. Acquired hypothyroidism
  4. PCOS
  5. Addison’s disease

Ans. The key is D. PCOS. [It is not cushing’s as insulin levels in cushing’s are not usually raised! Here obesity, primary amenorrhea , acne and particularly high level of insulin makes the likely diagnosis to PCOS].

618. An 18yo girl with primary amenorrhea complains of severe abdominal pain every 4-8weeks which is now getting worse. Exam: lower abdominal mass is felt. What is the most probable dx?

  1. Ectopic pregnancy
  2. Ovarian carcinoma
  3. Hematometrium
  4. Biliary colic
  5. Renal carcinoma

Ans. The key is C. Hematometrium. [Primary amenorrhea and periodic pain indicate hematometrium either secondary to imperforated hymen or vaginal septum].

 

619.A 14yo boy with asthma suddenly developed chest pain and increasing breathlessness during a game of football. When seen in the ED he was not cyanosed. He has reduced breath sounds on the right side. His oxygen saturation is 94% on air. What is the single most appropriate inv?

  1. Capillary blood gases
  2. CXR
  3. CT chest
  4. Exercise challenge
  5. MRI chest

Ans. The key is B. CXR. [Asthma is a predisposing factor for sponta- neous pneumothorax. The presentation indicates pneumothorax for which most appropriate investigation is CXR].

 

 

620.  A 36yo woman was recently admitted to a psychiatric ward. She believes that the staff and other pts know exactly what she is thinking all the time. What is the most likely symptom this pt is suffering from?

  1. Thought insertion
  2. Thought withdrawal
  3. Thought block
  4. Though broadcasting
  5. Hallucination

Ans. The key is D. Thought broadcasting.

621. A 60yo woman is admitted to the hospital after a fall. She is noted to have poor eye contact. When asked how she is feeling, she admits to feeling low in mood and losing enjoyment in all her usual hobbies. She has also found it difficult to concentrate, feels that she is not good at anything, feels guilty over minor issues and feels very negative about the future. What is the most likely dx?

  1. Mild depression
  2. Moderate depression
  3. Severe depression
  4. Psychosis
  5. Seasonal depression

Ans. The key is A. Mild depression. [Mild depression: i)Low mood ii) Anhedonia iii) Guilt iv) Hopelessness v) Worthlessness vi) Inability to concentrate].

622.  A 70yo woman lives in a nursing home following a stroke has developed reddish scaly rash on her trunk. She has many scratch marks on her limbs and trunk with scaling lesions on her hands and feet. What is the single most appropriate initial tx?

  1. Aqueous cream
  2. Chlorphenaramine
  3. Coal tar
  4. 1% hydrocortisone ointment
  5. Permethrin

Ans. The key is E. Permethrine. [This is a case of scabies and scaly rash denotes the infection of most severe type the crusted or Norwe- gian scabies. Should be treated with permethrine].

623.  A 16yo boy following a RTA was brought to the ED with a swell- ing and deformity in his right thigh. Exam: airway is patent and is found to have a pulseless leg. Which structure is involved in this fx?

  1. Femoral artery
  2. Posterior tibial artery
  3. Common peroneal nerve
  4. Dorsalis pedis

Ans. The key is A. Femoral artery.

 

 

 

 

 

624.  A man sat cross-legged for about 30mins. After this he was unable to dorsiflex his left foot and had loss of sensation in the web space between the big toe and the 2nd toe. He also has sensory loss on the same side of the foot after 2h. Which of the following was affected?

  1. Femoral nerve
  2. Sural nerve
  3. Peroneal nerve
  4. Sciatic nerve

Ans. The key is C. Peroneal nerve. [Common peroneal nerve winds round the fibular neck at knee joint and when a man sits cross legged for a considerable time pressure exerted on the nerve may cause nerve palsi].

625.  A 25yo woman is presenting with diarrhea and abdominal bloat- ing over the last 4m. Exam: she has blistering rash over her elbows. Biochemistry: low serum albumin, calcium and folate conc. On jejunal biopsy, there is shortening of the villi and lymphocytosis. What is the most likely dx?

  1. Celiac disease
  2. Whipple’s disease
  3. Crohn’s disease
  4. Tropical sprue
  5. Giardiasis
  6. Cystic fibrosis

Ans. The key is A. Celiac disease. [diarrhea, abdominal bloating, blistering skin rash over elbow (Dermatitis herpetiformis), low serum albumin, calcium and folate conc. Supported by shortening of villi and lymphocytosis on jejuna biopsy is classic presentation of celiac disease].

626.  A 19yo man presents for the 1st time with a firm and unshakable belief that he is being followed by terrorists who are plotting against him. What is the single best term for this man’s condition?

  1. Delusion of persecution
  2. Delusion of grandeur
  3. Delusion of control
  4. Delusion of reference
  5. Delusion of nihilism

Ans. The key is A. Delusion of persecution. [Delusions of persecution refer to false beliefs or perceptions in which a person believes that they are being treated with malicious intent, hostility, or harassment – despite significant evidence to suggest otherwise].

627. A 19yo female is brought in by her parents. They are concerned about her BMI which is 12. She is satisfied with it. What is the next step?

  1. Psychiatric referral for admission
  2. Family counselling
  3. Social service
  4. Start antidepo
  5. Medical admission

Ans. The key is E. Medical admission. [The diagnosis is anorexia nervosa. At this critical low BMI medical admission is indicated to improve her deficiency states and proper nutrition. ((BMI <15kg/m2, rapid weight loss + evidence of system failure) requires urgent referral to eating disorder unit (EDU), medical unit (MU) or paediatric medical wards].

 

628.  A lady who works at a nursing home presents with itching. Exam: linear tracks on the wrist. She says that 2d ago she had come in contact with a nursing home inmate with similar symptoms. What is the mechanism of itching?

  1. Infection
  2. Destruction of keratinocytes
  3. Allergic reaction
  4. Immunosuppression
  5. None

Ans. The key is C. Allergic reaction. [The probable diagnosis is scabies in which there is itching due to allergic reaction to mites (Sarcoptes scabii) waste products].

629.  A teacher had a respiratory infection for which she was prescribed antibiotics. After the antibiotic course when she rejoined school, she lost her voice completely. What is the single most appro- priate dx?

  1. Recurrent laryngeal nerve palsy
  2. Angioedema
  3. Laryngeal obstruction by medication
  4. Laryngitis
  5. Functional dysphonia/vocal cords

Ans. The key is E. Functional dysphonia/vocal cords. [Functional dysphonia is poor voice quality without any obvious anatomical, neurological or other organic difficulties affecting the larynx or voice box. It is often secondary to viral infection].

 

 

 

 

630.  A 43yo lady is admitted with pyrexia, arthropathy, breathlessness and syncope. She was recently dx with pulmonary emboli. There is an early diastolic sound and a mid-diastolic rumble. Her JVP is elevated with prominent a-waves. What is the most likely cause?

  1. Mitral regurgitation
  2. Ventricular ectopics
  3. Pulmonary regurgitation
  4. Atrial myxoma
  5. Complete heart block

Ans. The key is D. Atrial myxoma. [Pyrexia, arthropathy, breathlessness, syncope and early diastolic sound and a mid diastolic rumble are known features of atrial myxoma].

631. A 28yo man presents with a maculopapular rash over his trunk and palms. He also has numerous mouth ulcers. He had a penile ulcer which healed 2wks ago. What will you do to confirm the dx?

  1. PCR for treponemal and non-treponemal antibiodies
  2. Dark ground microscopy from mouth ulcer
  3. Blood culture for treponema
  4. Dengue fever

Ans. The key is A. PCR for treponomal and non-treponemal antibod- ies. [Non treponemal antibody test if positive indicate that there may be syphilis and it is not confirmatory alone. That is why treponemal antibody test should also be done to confirm it. On the other hand re- solved disease may show negative treponemal test which is confirmed by positive non-treponemal test].

632.  A 34yo man complains of arthralgia, abdominal pain and vom- iting, a facial rash that is worse in the summer and hematuria. Urea and creatinine are slightly elevated with urinalysis demonstrating red cell casts. PMH is remarkable for childhood eczema. Which inv is most likely to lead to a dx?

  1. US KUB
  2. Joint aspiration
  3. Auto antibodies
  4. IVU
  5. Renal biopsy

Ans. The key is C. Auto antibodies. [Likely diagnosis is SLE for which auto antibody (anti ds DNA antibody) should be done].

633.  A 56yo woman has had severe abdominal pain for 24h radiating to her back and is accompanied by nausea and vomiting. She appears to be tachycardic and in shock. She was found to have gallstones, 2yrs ago. What is the most likely inv to confirm dx?

  1. US abdomen
  2. LFT
  3. Serum lipase
  4. Angiography
  5. CT abdomen

Ans. The key is C. Serum lipase. [The likely diagnosis is pancreatitis hence serum lipase].

634.  A 32yo female with axillary freckles and café au lait spots wants to know the chances of her child also having similar condition.

  1. 1:2
  2. 1:4
  3. No genetic link
  4. 1:16
  5. Depends on the genetic make up of the partner

Ans. The key is E. Depends on genetic make up of the partner. [De-

pends on the make up of the partner, ideally it’s 1:2 since it’s autoso- mal dominant with complete penetrance but if the patients spouse also has neurofibromatosis, it raises it to at least 75% with a possible 100%].

635.  A 40yo man has pain, redness and swelling over the nasal end of his right lower eyelid. The eye is watery with some purulent

discharge. The redness extends on to the nasal peri-orbital area and mucoid discharge can be expressed from the lacrimal punctum. What is the single most appropriate clinical dx?

  1. Acute conjunctivitis
  2. Acute dacrocystitis
  3. Acut iritis
  4. Retrobulbar neuritis
  5. Scleritis

Ans. The key is B. Acute dacrocystitis.

 

 

636.  A 60yo lady has severe chest pain. ECG shows changes of infe- rior wall MI. ECG also shows progressive prolongation of PR interval until a QRS complex is dropped. What is the most probable dx?

  1. Atrial fibrillation
  2. VT
  3. SVT
  4. Mobitz type I 2nd degree heart block
  5. Mobitz type II 2nd degree heart block

Ans. The key is D. Mobitz type I 2nd degree heart block. [Inferior MI is frequently associated with conduction defect].

637. A 52yo woman speaks rapidly without any pause and ignores interruptions. She doesn’t even pause to take enough breaths. What term best describes this kind of speech?

  1. Flight of ideas
  2. Broca’s aphasia
  3. Wernicke’s aphasia
  4. Pressure of speech
  5. Verbal dysphasia

Ans. The key is D. Pressure of speech.

 

638.  A 30yo woman has been feeling low and having difficulty in concentrating since her mother passed away 2m ago. She feels lethargic and tends to have breathlessness and tremors from time to time. What is the most likely dx?

  1. Adjustment disorder
  2. PTSD
  3. Panic disorder
  4. GAD
  5. Bereavement

Ans. The key is A. Adjustment disorder. [When 2 months passed it is no more normal bereavement but major depression or adjustment disorder].

639.  A 32yo man on psychiatric medications complains of inability to ejaculate. Which drug is most likely to cause these symptoms?

  1. Lithium
  2. Haloperidol
  3. Chlorpromazine
  4. Fluoxetine
  5. Clozapine

Ans. The key is D. Fluoxetine. [SSRIs are frequently associated with delayed ejaculation].

640.   A 4yo boy is brought by his parents with complains of wetting his bed at night and whenever he gets excited. What would be the most appropriate management for this child?

  1. Desmopressin
  2. Oxybutanin
  3. Behavioural therapy
  4. Tamsulosin
  5. Restrict fluid intake

Ans. The key is C. Behavioural therapy. Probably this is wrong option! There is no correct option for this question. [The question is inappro- priate. Child is of 4 yrs of age and before 5 yrs only reassurance, no treatment is indicated].

641. A 34yo DM pt is undergoing contrast radiography. What mea- sure should be taken to prevent renal damage with contrast dye?

  1. Reduce contrast dye
  2. Plenty of fluids
  3. NSAIDS
  4. ACEi
  5. IV dextrose

Ans. The key is B. Plenty of fluids.

642.  A 75yo woman presents to the breast clinic having noticed that she has had a blood stained discharge from the left nipple, together with dry skin over the left areola. Exam: blood stained discharge with dry flaky skin noted on the left areola. The nipple was noted to be ulcerated. Wht is the most appropriate inv?

  1. FNAC
  2. MRI
  3. Punch biopsy
  4. Open biopsy
  5. Stereotactic biopsy

Ans. The key is C. Punch biopsy. [As the lesion is on the surface punch biopsy can be well obtained].

643.  A 50yo man presents with low mood, poor concentration, anhedonia and insomnia. He has had 2 episodes of increased activity, promiscuity and aggressive behavior in the past. He was arrest 8m ago for trying to rob a bank claiming it as his own. Which drug is most likely to benefit him?

  1. Haloperidol
  2. Citalopram
  3. Desipramine
  4. Carbamazepine
  5. Ethosuximide

Ans. The key is D. Carbamazepine. [It is a case of bipolar disorder. Mainstay of treatment is mood stabilizers such as i) Lithium carbon- ate ii) Anticonvulsant medicines iii) Antipsychotic medicines. So from the given options Carbamazepine is the most appropriate drug].

644.  A 25yo woman complains of dizziness, nausea, vomiting, visual disturbances and anxiety which keep coming from time to time.

Most of the attacks are a/w sudden change in posture. What is the most likely dx?

  1. Panic disorder
  2. Carotid sinus syncope
  3. BPPV
  4. Vertebrobasilar insufficiency
  5. Postural hypotension

Ans. The key is C. Benign paroxysmal positional vertigo (BPPV). [Dizziness, nausea, vomiting and nystagmus which keep coming from time to time are common features of BPPV].

645.  A 56yo man was recently put on anti-HTN meds and recent biochemistry on 2 occasions showed: Na+=132, K+=7.6, Urea=11.3, Creat=112. Which of the following drugs is responsible for this result?

  1. Amlodipine
  2. Bendroflumethiazide
  3. Doxazosin
  4. Atenolol
  5. Ramipril

Ans. The key is E. Ramipril. [ACEI and ARB are known to raise the serum potassium level].

646.  A 46yo woman has offensive yellow discharge from one nipple. She had a hx of breast abscess 3yrs ago. What is the possible dx?

  1. Duct papilloma
  2. Duct ectasia
  3. Duct fistula
  4. Breast cancer

Ans. The key is C. Duct fistula.

647. A 35yo woman undergoing tx for TB presents with malar rash, photosensitivity and hematuria. What is the single most likely posi- tive antibody?

  1. Anti Ds DNA
  2. Anti Sm
  3. Anti Histone
  4. Anti La
  5. Anti centromere

Ans. The key is C. Antihistone antibody. [Anti histone antibody is present in 95% cases of drug induced lupus].

648.  A 6wk child with profuse projectile vomiting. What is the first thing you will do?

  1. US
  2. Check serum K+ level
  3. ABG
  4. NG tube
  5. IV fluids

Ans. The key is B. Check serum K+ level. This is a wrong key! Correct key is E. IV fluid.

 

649.  A 55yo woman who attends the clinic has recently been dx with a depressive episode. She complains of unintentionally waking early in the morning, a recent disinterest in sex and a loss of appetite, losing 5kg weight in the last month. She feels that her mood is worse at the beginning of the day. What is the most likely dx for this pt?

  1. Mild depression
  2. Moderate depression
  3. Severe depression
  4. Low mood
  5. Pseudo depression

Ans. The key is B. Moderate depression. [Sleep disturbance, disin- terest in sex and loss of appetite points towards the diagnosis of moderate depression].

650.  An employer sent his worker to the ED after having hit his head on a machine. Exam: normal. What is the single most likely inv you would do?

  1. Skull XR
  2. CT head
  3. MRI head
  4. Reassure

Ans. The key is A. Skull XR. This is a wrong key! Correct key should be

  1. Reassurance. [Indications for CT are not fulfilled by given criteria. NICE has mentioned X-ray skull is no more preferred over CT. Hence answer for this question is D. Reassurance].

651. A lady with fam hx of ovarian carcinoma has a pelvis US that fails to reveal any abnormality. What is the single most appropriate inv?

  1. Pelvic CT
  2. CA 125
  3. CA 153
  4. Laparoscopy
  5. MRI

Ans. The key is B. CA 125. [CA 125 is tumour marker for ovarian tumour].

652.  A 10yo boy is taken to his GP by his parents with behavioural prbs. He attends a special school due to inappropriate behavior and during the interview with his parents the boy barks at infrequent episodes and shouts expletives. What is the most likely dx?

  1. Asperger syndrome
  2. Cotard syndrome
  3. Rett syndrome
  4. Ekbom syndrome
  5. Tourette’s syndrome

Ans. The key is E. Tourette’s syndrome. [Tourete’s syndrome may have

motor tics like blinking, facial grimacing, shoulder shrugging. Other complex motor tics may be sniffing, touching objects, hopping, jump- ing, bending or twisting. It has vocal tics like throat clearing, sniffing, grunting or barking and more complex like coprolalia (uttering socially inappropriate words) or echolalia (repeating the word or phrase of others)].

653.  A 52yo male presents with sudden complete loss of vision from right eye. He also had been complaining of right sided headaches which would come up more on chewing. On fundoscopy, the retina was pale and a cherry red spot could be seen in the macular region. What caused this vision loss?

  1. CRAO
  2. CRVO
  3. Branch RAO
  4. Branch RVO
  5. Circumciliary vein occlusion

Ans. The key is A. CRAO. [Pale retina with cherry red spot in macular region is seen in CRAO].

 

 

 

 

654.  A 48yo woman presents with left-sided severe headache. She also has a red, watering eye and complains of seeing colored haloes in her vision. What is the most appropriate next step?

  1. Measure IOP
  2. Relieve pain with aspirin
  3. 100% oxygen
  4. CT
  5. Relieve pain with sumatriptan

Ans. The key is A. Measure IOP. [Probable case of angle closure glau- coma requiring measurement of IOP to establish the diagnosis].

655.  A 31yo woman presents with 7-10days following childbirth, with loss of feeling for the child, loss of appetite, sleep disturbance and intrusive and unpleasant thoughts of harming the baby. What is the best tx for this pt?

  1. Fluoxetine
  2. Haloperidol
  3. CBT
  4. Reassurance
  5. ECT

Ans. The key is A. Fluoxetine. This is wrong key! Correct key should be

  1. Haloperidol. [Dx is postpartum psychosis. Patient.info, Wikipedia says antipsychotic and if needed additional mood stabilizer may be used as 1st line. NHS says antidepressant, antipsychotic and mood

stabilizer (any one or in combination) as 1st line treatment. As antipsy- chotic has given priority likely correct answer is C. IV haloperidol. 2nd line is CBT, and last resort is ECT].

656.  A 56yo male pt presents with intermittent vertigo, tinnitus and hearing loss. What is the best drug tx for this pt?

  1. Buccal prochlorperazine
  2. Oral flupenphenazine
  3. TCA
  4. Gentamicin patch on the round window
  5. No med tx available

Ans. The key is A. Buccal prochlorperazine. [Probable case of Menieres disease. Treated with prochlorperazine].

657. An 82yo woman has developed painful rash on one side of her forehead and ant scalp. Lesions have also affected her cornea. What is the single most appropriate option?

  1. Accessory nerve
  2. Facial nerve
  3. Olfactory nerve
  4. Optic nerve
  5. Trigeminal nerve

Ans. The key is E. Trigeminal nerve. [Probable herpes zoster opthal- micus].

 

 

 

 

 

658.  A 24yo woman presents with episodes of peri-oral tingling and carpo pedal spasms every time she has to give a public talk. This also happens to her before interviews, exams and after arguments. What is the best management strategy for this pt?

  1. Diazepam
  2. Rebreathe in a paper bag
  3. Desensitization
  4. Buspirone
  5. Propranolol

Ans. The key is B. Rebreathe in a paper bag. This is wrong key. More correct option is C. Desensitization.[Desensitization is the treatment of choice in long run. For prevention proranalol before expected exposure and if patient presents with an attack then rebreathing in a paperbag to subside her acute problems].

 

659.  A 32yo woman P3 of 39wks gestation reports having sponta- neous ROM 4days ago. She didn’t attend the delivery suite as she knew that would happen and had already decided on a home birth. Today she feels very hot and sweaty. She thought that she was start- ing to have labour pains but she describes the pain as more constant. Exam: uterus is tender throughout. Blood tests show raised CRP and WBC. Select the most likely dx?

  1. Round ligament stretching
  2. Chorioamnionitis
  3. Uterine rupture
  4. Labor
  5. DIC

Ans. The key is B. Chorioamnionitis. [Prolonged rupture of membrane can lead to chorioamnionitis].

660.  A 63yo man continues to experience chest pain and has a temp of 37.8C 2 days after an acute MI. His ECG shows widespread ST elevation with upward concavity. What is the single most likely explanation for the abnormal inv?

  1. Acute pericarditis
  2. Cardiac tamponade
  3. Atrial thrombus
  4. Left ventricular aneurysm
  5. Dressler syndrome

Ans. The key is A. Acute pericarditis. [Chest pain, raised temperature and ECG findings of widespread ST elevation with upwards concavity is diagnostic of acute pericarditis particularly after MI].

661. A 55yo man presents with an ulcer of the scrotum. Which of the following LN is involved?

  1. External iliac LN
  2. Pre-aortic LN
  3. Aortic LN
  4. Inguinal LN
  5. Iliac LN
  6. Submental LN
  7. Submandibular LN
  8. Deep cervical LN

Ans. The key is D. Inguinal LN.

662.  A 35yo woman has butterfly rash on her face and she suffers symmetrical joint pains on knee and elbow, ESR is raised. What is the most discriminative inv for dx?

  1. Anti DNA antibodies
  2. Anti Jo1 antibodies
  3. Anti nuclear antibodies
  4. Anti centromere antibodies
  5. Anti la antibodies

Ans. The key is A. Anti DNA antibodies. [Anti DNA antibodies to diagnose SLE].

663.  Pt had a fight following which he developed bleeding, ringing and hearing loss from one ear. What is the inv of choice?

  1. CT
  2. XR skull
  3. Otoscopy
  4. MRI vestibule

e.    Coagulation studytAns. The key is A. CT. This is a wrong key! Correct key is Otoscopy.

  1. A 35yo IVDA (Intra Venous Drug Abuser) on penicillin and flucloxacillin for cellulitis now presents with jaundice, pale stools and dark What is the single most likely dx?
  2. Hep A
  3. Cholestatic jaundice
  4. Chronic active hepatitis
  5. Primary biliary cirrhosis
  6. Hep B

Ans. The key is B. Cholestatic jaundice. [Flucloxacillin can cause cholestatic jaundice].

  1. A 79yo woman has been dx with Her BMI=22. RBS are 8 and 10mmol/l. Her BP=130/80mmHg. Her fasting cholester-

ol=5.7mmol/l. She is currently symptom-free but has microalbumin- uria. What is the single most appropriate drug management?

  1. ACEi and glibenclamide
  2. ACEi and metformin
  3. Statin and ACEi
  4. Statin and glibenclamide
  5. Statin and metformin

Ans. The key is C. Statin and ACEi. [Diabetic patients are advocated statin irrespective of cholesterol levels and diabetic microalbuminuria is best treated by ACEI. As initially we shall give lifestyle advice and no medicine for diabetes even then we shall start with statin and ACEI].

666.  A 68yo woman is unable to extend the IP joint of her right thumb 7wks following a fx of the right radius. Other finger and thumb movements are normal. What is the single most likely tendon to be damaged?

  1. Abductor pollicis longus
  2. Extensor pollicis brevis
  3. Extensor pollicis longus
  4. Flexor digitorum profundus
  5. Flexor pollicis longus

Ans. The key is C. Extensor policis longus. [Full extension of right thumb is achieved by extensor pollicis longus].

667. A mother presents her 6m son who is vocalizing. She has no- ticed that he doesn’t respond to loud noises. His motor milestones are normal. What is the best management strategy?

  1. Arrange hearing test
  2. Assess development milestones
  3. Reassure
  4. Refer to speech therapist
  5. MRI brain

Ans. The key is A. Arrange hearing test. [Normal motor milestones indicate normal development but unable to respond to loud noise at 6 months may indicate deafness].

668.  A 39yo man presents to the ED with persistent cough, sputum and dyspnea. He gave a hx of smoking 20 cigarettes/d for the last 10 years. Pt was given oxygen in ambulance but he is not improving. What is the next step?

  1. Prednisolone
  2. Salbutamol
  3. Check ABG
  4. CXR
  5. ECG

Ans. The key is C. Check ABG. [The patient has COPD and as no improvement with oxygen, next step is to check ABG to give guidance for next treatment plan].

 

669.  A 66yo woman has been brought to the hospital on a number of occasions with a hx of loss of memory. Her PMH is significant for an MI 6yrs ago. It is noted that she has a step wise decline of her cognitive functions. What is the most likely dx?

  1. Alzhemiers
  2. Vascular dementia
  3. Pick’s dementia
  4. Huntington’s disease
  5. Lewy body dementia

Ans. The key is B. Vascular dementia. [Age 65 yrs (vascular dementia is rare at age before 65), history of MI and stepwise decline of cogni- tive function indicates vasculopathy and hence vascular dementia].

 

 

670.  A 55yo man returns for routine follow up 6wks after an MI. He gets breathless when walking uphill. His ECG shows ST elevation in leads V1, V2, V3 and V4. What is the single most likely explanation for the abnormal investigation?

  1. Heart block
  2. Right ventricular strain
  3. Atrial thrombus
  4. Left ventricular aneurysm
  5. Dressler’s syndrome

Ans. The key is D. Left ventricular aneurism. [Features of heart failure and persistent ST elevation suggests the dx of left ventricular aneu- rysm].

671.  A 4m girl has severe FTT (Failure To Thrive) and increasing jaundice which was 1st noticed at 1wk of age. She has an enlarged liver and scratches on her skin. Her parents have been unable to seek medical care. What is the most likely dx?

  1. Biliary atresia
  2. G6PD deficiency
  3. Hep B
  4. Spherocytosis

Ans. The key is A. Biliary atresia. [Increasing jaundice at this age with failure to thrive, enlarged liver and scratches (itching) indicate choles- tatic jaundice likely from biliary atresia].

672. A 76yo man suddenly collapsed and died. At post mortem exam, a retroperitoneal hematoma due to ruptured aortic aneurysm was noted. What is the most likely underlying cause of the aortic aneu- rysm?

  1. Atheroma
  2. Cystic medial necrosis
  3. Dissecting aneurysm
  4. Polyarteritis nodosa
  5. Syphilis

Ans. The key is A. Atheroma.

673. A 33yo male came to the hospital with complaint of occasional left sided chest pain that lasts <30mins, following exercise, which relieves upon taking rest. What is the most probable dx?

  1. Unstable angina
  2. Decubitus angina
  3. Stable angina
  4. Coronary spasm
  5. MI

Ans. The key is C. Stable angina.

 

 

 

674. A 42yo overweight smoker comes with heavy periods. A scan reveals a normal uterus. She would like a long term tx with minimal side effects that would offer tx for the menorrhagia and provide contraception. She is unsure whether she would like more children. She is adamant that she doesn’t want surgery as she is terrified of the prospect. Select the best management for her menorrhagia?

  1. COCP
  2. GrH analogues
  3. IU/systemic progesterone
  4. NSAIDs
  5. Copper containing IUCD

Ans. The key is C. IU/systemic progesterone. [As patient is smoker, COCP should be avoided. In the given case option C. i.e. mirena is most suitable].

675. A 10yo male child was brought by his mother complaining that her child watches TV at very high volumes, doesn’t like to play

outside and instead has become more sincere with reading. She also says that her son doesn’t respond to her. What do you expect to see on otoscopy?

  1. Flamingo pink tympanic membrane
  2. Attic perforation
  3. Bluish grey tympanic membrane with air fluid levels
  4. Inflamed tympanic membrane with cart wheel appearance of vessels
  5. Red and inflamed tympanic membrane

Ans. The key is C. Bluish grey tympanic membrane with air fluid levels. [In glue air there occur conductive deafness so the child watch tv with high volume and does not respond to others for this deafness].

676. A lady underwent debulking surgery for ovarian carcinoma. Soon after the surgery she presents with signs of intestinal obstruction. What is the single most appropriate inv?

  1. Pelvic CT
  2. CA 125
  3. Laparotomy
  4. Laparoscopy
  5. Abdominal US

Ans. The key is C. Laparotomy. [In cancer patient obstruction does not respond to medical treatment and hence diagnostic and therapeutic laparotomy is the option of choice].

677. A 45yo woman undergoing tx for RA for the last 5yrs presents with dizziness, easy fatigabiliy and lack of energy. A blood film shows MCV 106. What could be the most probable reason for her anemia?

  1. Steroids
  2. Chronic disease
  3. NSAIDs
  4. Methotrexate
  5. Sulfasalazine

Ans. The key is D. Methotrexate. [Methotrexate leads to folate defi- ciency anaemia].

 

678.  A 62yo man who has recently had flu-like illness has woken to find his speech altered. Movement of his eyelids and lips are weak on the right side. Exam: normal. Which anatomical site is most likely to be affected?

  1. Facial nerve
  2. Hypoglossal nerve
  3. Oculomotor nerve
  4. Trigeminal nerve
  5. Glossopharyngeal nerve

Ans. The key is A. Facial nerve. [Viral infection is a common cause of facial nerve palsy].

679. A 5yo girl has had an URTI for 3 days and has been treated with paracetamol by her mother. For the last 12h she has been hot and irritable with severe pain in her right ear. What is the most likely dx?

  1. Herpes zoster infection
  2. Impacted ear wax
  3. Mumps
  4. OM
  5. Perforation of eardrum

Ans. The key is D. OM.

680.   A 35yo man has a temp=39C, cough with purulent sputum and right sided chest pain on inspiration. He has herpes labialis. What is the single most likely causative organism?

  1. Coagulase +ve cocci in sputum
  2. Gram -ve diplococci in sputum
  3. Gram +ve diplococci in sputum
  4. Pneumocystis carinii in sputum
  5. Serology for legionella

Ans. The key is C. Gram +ve diplococcic in sputum. [High temperature, cough with purulent sputum, pleuritic chest pain and herpes labialis are recognized feature of pneumococcal pneumonia (pneumococcus=- Gram +ve diplococci].

681. A 27yo female was brought to the ED by her friend from a movie theatre. She complains of sudden severe pain in the eye followed by vomiting and also was seeing colored halos. She gives a past hx of recurrent headaches which used to resolve spontaneously. Exam: fixed, dilated ovoid pupil seen. What is the first inv?

  1. CT head
  2. MRI orbits
  3. Blood culture and sensitivity
  4. Toxicology screen
  5. Applanation tonometry

Ans. The key is E. Applanation tonometry. [Darkness can cause

dilatation of pupil (which occurred in the darkness of theatre room) which (halfway dilatation) most often precipitate acute attack of angle closure glaucoma and the test to diagnose this is applanation tonometry].

 

 

 

 

682.  An 82yo male suddenly becomes unconscious and fell down. He recovered completely within minutes. What is the best inv you do to dx the case?

  1. ECG
  2. EEG
  3. Blood glucose level
  4. CT
  5. CXR

Ans. The key is A. ECG. [Here sudden unconciuosness without any provocation, which makes arrhythmia the most likely cause; hence we should do ECG].

683.  A child admitted with progressive muscle weakness and fre- quent falls. What is the most probable dx?

  1. Duchenne’s MD
  2. Becker’s MD
  3. Polymyositis
  4. Dermatomyositis
  5. Polymyalgia rheumatic

Ans. The key is A. Duchenne’s MD.

684.  A 56yo man presents to the ED with chest pain. The following ECG was taken. What is the most likely dx?

 
  

 

  1. Anterior MI
  2. Inferior MI
  3. Lateral MI
  4. Posterior MI
  5. NSTEMI

Ans. The key is A. Anterior MI. [Here ST elevation in L1, aVL, v2,v3,v4 and v5. So the diagnosis is anterior MI (as L1 and aVL and v2-v5,

anterolateral would be more appropriate description)].

685.  A schizophrenic says: life is unfair. I like fairs. Fairs have food. It must be good. What term describes this pt’s speech?

  1. Neologism
  2. Flight of ideas
  3. Broc1a’s aphasia
  4. Wernicke’s aphasia
  5. Clang association

Ans. The key is E. Clang association. [The rhymic use of words as described is known as clanging or clang association often seen in schizophrenia].

 

 

686.  A man comes to the ED with hx of pulsatile swelling in the abdomen, he has hx of HTN and exam: pulse=120bpm,

BP=70/40mmHg. He is restless and in shock. What emergency management should be done on priority basis?

  1. Urgent abdominal CT
  2. Urgent abdominal US
  3. IV fluids 9% NS crystalloids to bring SBP to 90mmHg
  4. IV fluids 9% NS crystalloids to bring SBP to 120mmHg
  5. Dopamine inj

Ans. The key is C. IV fluids 0.9% NS crystalloids to bring SBP to 90mmHg [Probable ruptured aortic aneurism. Immediate IV normal saline to raise the BP to 90 mmHg to keep the vital organs perfused till definitive measures are taken].

687.  A 5yo boy has cough and swelling at the knee after falling on the ground with rashes on the buttocks which are non-blanching. PT=13, APTT=71, Hgb=11, WBC=8, Plt=200. Choose the most likely dx?

  1. NAI
  2. Hemophilia
  3. HSP
  4. Osler weber reindu syndrome
  5. Von-Willebrand disease

Ans. The key is B. Hemophilia. This is a controversial key! Correct key should be E. Von-Willebrand disease. [Likely D/D may be B or C! in von willebrands disease usually there is no hemarthrosis (except in type 3) and in hemophilia no nonblanching rash (there may be bruis- ing). isolated rise in APTT is highly suggestive of hemophilia (given case). This is an ill defined question and only likely diagnosis may be type 3 von willebrands disease (in HPS- APTT is not raised and there may be thrombocytosis, so this is not a case of HSP].

 

688.  A 45yo woman presents with discharge from the left nipple. The discharge is brownish-green and foul smelling. What is the most likely dx?

  1. Duct papilloma
  2. Intra-ductal papilloma
  3. Duct ectasia
  4. Mammary duct fistula
  5. Breast abscess

Ans. The key is C. Duct ectasia. [Women near menopause are more affected by duct ectasia characterized by brown, green or cheesy discharge. This condition is harmless and tends to clear up without treatment].

689.  A 10yo boy presents with generalized swelling. This has been present for 4days and included swollen ankles and puffiness of the face. It started a few days after he had a mild cold with runny nose. His only PMH was eczema. Urine analysis: hematuria, proteinuria 10g/24h, creat 60umol/l and albumin=15g/l. What is the single most likely dx?

  1. IgA nephropathy
  2. HSP
  3. Minimal change nephropathy
  4. Wilson’s disease
  5. Cardiac failure

Ans. The key is A. IgA nephropathy. [10 yr old boy, history of URTI and hematuria points towards the diagnosis of IgA nephropathy. It may

be present with proteinuria and generalized swelling. The important differentiating point from rapidly progressive GN is duration. IgA ne- phropathy <1o days (usually 4/5 days history of infection but in rapidly progressive GN history of infection for >10 days].

690.  A 28yo man complains of vertigo, nausea and vomiting for more than 30 mins and tinnitus, hearing loss in the left ear. What is the tx for this pt?

  1. Buccal prochlorperazine (2nd line)
  2. Metachlorpromide
  3. Cyclazine (1st line)
  4. Cotrimazole
  5. Ondansetron

Ans. The key is C. Cyclizine. More appropriate is A. Buccal prochlorper- azine (patient.info). [Both prochlorperazine and cyclizine can be used in Meniere’s disease].

691. A 16yo girl has been unwell for 5days with malaise, headache and dry cough. She has a few crackles in her chest. Her CXR shows patchy consolidation in the lower lobes. What is the single most likely causative organism?

  1. Cold agglutinins
  2. Gram –ve diplococci in sputum
  3. Gram +ve diplococcic in sputum
  4. Serology for legionella
  5. Sputum staining for mycobacterium TB

Ans. The key is A. Cold agglutinins. [Mycoplasma pneumonia, a form of atypical bacterial pneumonia related to cold agglutinin disease].

692.  A child with increasing jaundice and pale stools. Choose the appropriate test:

  1. Endomyseal antibodies
  2. Sweat test
  3. TFT
  4. LFT
  5. US

Ans. The key is E. US. [This is obstructive jaundice where US may show dilatation of common bile duct or stones].

693.  A 73yo woman with RA is unable to extend the fingers of her right hand at the MCP joint and IP joints following a fall. What is the single most likely tendon to have been damaged?

  1. Extensor carpi radialis
  2. Extensor carpi ulnaris
  3. Extensor digitorum
  4. Extensor indicis
  5. Flexor digitorum profundum

Ans. The key is C. Extensor digitorum. [The extensor digitorum extends the phalanges, then the wrist, and finally the elbow. It acts principally on the proximal phalanges].

694.  You are called to see a 20yo woman 2h post-LSCS. She has not passed urine since her operation. She denied any urinary symptoms preoperatively. Exam: appears unwell, temp=37.5C, BP=94/73mmHg, pulse=116bpm, sat=97%. Her abdomen is distended with tenderness in the left flank and suprapubic region. Bowel sounds are not audible. Choose the most appropriate post Csection complication for this lady?

  1. UTI
  2. Urinary tract injury
  3. Pleurisy
  4. Acute pyelonephritis
  5. Paralytic ileus

Ans. The key is B. Urinary tract injury.

695.  A 58yo man has a headache and confusion of 3 days after slip- ping and hitting his head in the garden. What is the most appropri- ate initial inv?

  1. XR skull
  2. XR face
  3. CT brain
  4. MRI brain
  5. EEG

Ans. The key is C. CT brain. [The likely diagnosis is subdural haemato- ma for which appropriate initial investigation is CT scan of brain].

696.  A 4yo boy has a cough and arthritis followed by rash on legs which are non-blanching on glass test. No hx of fever. PT=13, APTT=31, Hgb=12, WBC=6.5, Plt=300. What’s the most likely dx?

  1. Meningitis septicemia
  2. Hemophilia
  3. HSP
  4. ITP
  5. TTP

Ans. The key is C. HSP. [Usually occurs below 10 yrs of age. The char- acteristic rash and lab findings matches with HSP].

 

697. A 72yo man presents to the ED with chest pain. The following ECG was taken. What is the most likely dx?

 

 

  1. Anterior MI
  2. Inferior MI
  3. Lateral MI
  4. Posterior MI
  5. NSTEMI

Ans. The key is B. Inferior MI. [There is pathological Q wave and mild ST elevation in leads II, III and aVF].

698.  A young man has diarrhea, loss of weight and flatulence for 3 days. What is the most imp tx?

  1. Metronidazole
  2. Fluconazole
  3. Vancomycin
  4. Amoxicillin

Ans. The key is A. Metronidazole.

699.  A 6yo child presented with drooling of saliva and severe stridor. He is febrile and sick looking. XR Neck in extension shows a thumb sign. Choose the single most likely dx.

  1. Croup
  2. URTI
  3. Diphtheria
  4. Acute epiglottitis

Ans. The key is D. Acute epiglottitis. [Drooling of saliva and stridor along with thumb sign in neck X-ray are features of acute epiglottitis].

700.   A mother presents with her 3yo son who has indistinct nasal speech. He snores at night and has restless sleep. He is tired by day. What is the best management strategy?

  1. Arrange hearing test
  2. Assess development milestones
  3. Refer to ENT surgeon
  4. Refer to speech therapist
  5. MRI brain

Ans. The key is C. Refer to ENT surgeon. [Probable enlarged adenoid].

701. A 17yo boy while playing football got a kick and now he is com- plaining of severe pain and swelling of the left side of his scrotum. What inv is the most appropriate to dx?

  1. Needle aspiration of scrotum
  2. US scrotum
  3. MSU
  4. Surgical exploration of scrotum
  5. Urine test for hematuria

Ans. The key is D. Surgcal exploration of scrotum. [This is a case of testicular torsion which needs urgent diagnostic and therapeutic surgical exploration of the scrotum].

702.  A 50yo man has had hoarseness of voice and drooping eyelid for 2m. a mass is palpable in the right supraclavicular fossa. He smokes 20 cigarettes/day for the last 30yrs. What is the most likely dx?

  1. Carcinoma larynx
  2. Carcinoma thyroid
  3. Carcinoma right bronchus
  4. Mesothelioma
  5. Pancoast tumor

Ans. The key is E. Pancoast tumour. [Hoarseness of voice is due to compression of the recurrent laryngeal nerve, ptosis due to com- pression of the sympathetic ganglion, palpable mass in right su- praclavicular fossa due to involvement of the supraclavicular lymph node. History of smoking and given picture indicates the diagnosis of Pancoast tumour].

703.  An 84yo man got surgical pain which is well controlled by oral morphine 60mg BD. However, now this pt is unable to swallow. What is the most appropriate next step?

  1. Morphine 60mg state
  2. Morphine 60mg TDS
  3. Oxycodone 10mg OD
  4. Morphine 60mg IV
  5. Fentanyl patches

Ans. The key is E. Fentanyl patches. [When dose of oral morphine is known and rout should be changed Phentanyl patch is adviced as the fixed dose is known and patch can release the required dose for

a given period (when we use patch we can not change the dose). But if the pain control is not optimal we should follow the next step (i.e. parenteral morphine) as per pain ladder. But as no correct dose of parenteral morphine is in the options (iv morphine dose is one-third of oral morphinre) we have to go for fentanyl patches].

704.  A 19yo man has exercise induced asthma. This has prv been controlled using a salbutamol inhaler as req, but he now gets attacks with exercise. What is the single most appropriate tx?

  1. Regular salbutamol
  2. Regular salbutamol and budesonide
  3. Sodium cromoglycate
  4. Oral steroid
  5. Inhaled steroid

Ans. The key is C. Sodium chromoglycate. This is wrong key! Correct key should be B. Regular salbutamol and budesonide. [Chrommogly- cate should be used in exercise induced asthma if salbutamol and ste- roids fail]. [This question is not appropriate as single most appropriate treatment to add or to chose from given options is not clear].

705.  A 3yo boy has a sudden onset of fever, vomiting and bilateral face swelling. Few days earlier the GP saw him for bilateral parotid pain and gave analgesics. What is the most appropriate next step?

  1. Analgesic
  2. Antibiotic
  3. Biopsy
  4. Immediate surgery
  5. Reassurance

Ans. The key is E. Reassurance. [A case of mumps. Self limiting condition].

 

 

 

 

 

 

 

706.  A 75yo man with adenocarcinoma of the prostate which has spread outside the capsule of the gland has ARF. What is the most appropriate next inv?

  1. MRI spine
  2. Radionuclide bone scan
  3. Trans rectal US
  4. US pelvis
  5. US KUB

Ans. The key is E. US KUB. [extension beyond capsule may cause obstruction of ureters, causing loin pain, anuria, symptoms of acute kidney injury or chronic kidney disease (here ARF) ref: patient.info].

 

707.  A 57yo male presents with sudden onset severe abdominal pain and rigidity against a 4d background of LIF pain and pyrexia. He has no PM/SHx of note and isn’t on any meds. What is the most likely dx?

  1. Intussusception
  2. Ischemic colon
  3. Sigmoid volvulus
  4. Perforated diverticulum
  5. Perforated Meckel’s diverticulum

Ans. The key is D. Perforated diverticulum. [Sudden onset, severe abdominal pain, rigidity, left iliac fossa pain and fever are in favour of perforated diverticulum].

708.   A 46yo woman has weight gain, sensitivity to cold, pulse=50b- pm, heart is enlarged with murmur. What is the single most likely dx?

  1. Hypothyroidism
  2. Hyperthyroidism
  3. Cushing’s syndrome
  4. Addison’s disease11
  5. Pheochromocytoma

Ans. The key is A. Hypothyroidism. [The given symptoms are classic presentation of hypothyroidism (cardiac murmur though is not that common)].

709.  An alcoholic who has completely given up drinking hears voices. What is the most appropriate tx?

  1. Olanzapine
  2. Diazepam
  3. Acamprosate
  4. Disulfiram
  5. Haloperidol

Ans. The key is A. Olanzapine. This is wrong key. Correct key is B. Diazepam. [A case of delirium tremens. Treated with benzodiazepines like chlordiazepoxide or diazepam].

 

 

710. A 6yo boy has completed an induction course of chemo for ALL. He has an enlarged left scrotum. What is the most appropriate next step?

  1. Herniotomy
  2. CT abdomen
  3. Biopsy
  4. Immediate surgery
  5. Reassurance

Ans. The key is B. CT abdomen. This is wrong key. Correct key is C. Biopsy. [Relapse may directly involve testis and excisional biopsy is done to confirm recurrence of leukemia].

Ref: Link: http://ascopubs.org/doi/full/10.1200/jco.2009.23.8014

711.        A 32yo miner is rescued after being trapped under a fallen rock for 4h. After applying a bladder catether, 15-20ml of reddish brown urine was obtained. HR=120bpm, SBP=100mmHg. What would be the next appropriate step?

  1. Dopamine IV
  2. Fluid challenge
  3. Furosemide IV
  4. 20% Mannitol IV
  5. Antibiotics

Ans. The key is B. Fluid challenge. [The diagnosis is rhabdomyolysis. So IV fluid is the next appropriate step].

712.A 60yo man has had spontaneous painful swelling of his right knee for 3days. 5days prv he had an inguinal hernia repaired as a day case. He takes bendroflumethiazide 2.5mg daily. He is apyrexial. What is the single most appropriate diagnostic inv?

  1. Blood culture
  2. CRP
  3. D-dimer
  4. XR knee
  5. Serum uric acid

Ans. The key is E. Serum uric acid. [Thiazide diuretics causes hyperui- cemia which can precipitate acute attack of gout].

713. A 27yo woman with anxiety and weight loss has tachycardia, tremor and mild proptosis. What single mechanism accounts for her weight loss?

  1. Deficiency in thyroid hormone
  2. Increased level of calcitonin
  3. Increased metabolic rate
  4. Insulin resistance
  5. Reduced caloric intake

Ans. The key is C. Increased metabolic rate. [The given features are of thyrotoxicosis in which increased metabolism causes loss of patients weight].

714. A man with carcinoma and multiple metastasis presents with in- tractable nausea and vomiting. He has become drowsy and confused. What is the most appropriate management?

  1. Dexamethasone IM
  2. Dexamethasone PO
  3. Ondansetron IM
  4. Ondansetron PO
  5. Morphine oral

Ans. The key is C. Ondensatron IM. [For cancer or chemotherapy induced vomiting ondensatron is the drug of choice. As here vomiting is intractable IM ondensatron should be given not oral].

715.A 19yo man presents with weight loss, increasing thirst and increasing frequency of going to the washroom. His father, grandfa- ther and 2 sisters have been dx with DM. What is the most likely type of DM this pt suffers from?

  1. IDDM
  2. NIDDM
  3. LADA
  4. MODY
  5. DKA

Ans. The key is D. MODY. [Key features of MODY are: Being diag- nosed with diabetes under the age of 25 and having a parent with diabetes, with diabetes in two or more generations].

716. A 42yo woman with a PMH of severe headache treated in the ED presents with signs and symptoms of renal failure. She has been seen by her GP for HTN and abdominal pain with OP inv pending. Which inv is most likely to lead to a dx?

  1. US KUB
  2. CT brain
  3. IVU
  4. Renal artery Doppler
  5. Renal biopsy

Ans. The key is A. US KUB. [Hypertension, abdominal pain and fea- tures of renal failure indicates the diagnosis of ADPKD for which the diagnosis is best made by US KUB].

717.In perforation of a post gastric ulcer, where will the fluid accumulate in the peritoneal cavity?

  1. Left paracolic gutter
  2. Pelvic cavity
  3. Lesser sac
  4. Under the diaphragm
  5. Right paracolic gutter

Ans. The key is C. Lesser sac.

 

718. A 62yo male is brought to the ED by his daughter because of his persistent lying. He is a known alcoholic and has been admitted recently with delirium tremens. On questioning, he denies any prob- lem with memory. He knows his name and address and states that he was at the betting shop this morning, but his daughter interjects calling him a liar explaining that he was at her home. What is the most likely dx?

  1. Ganser syndrome
  2. Cotard syndrome
  3. Wernicke’s encephalopathy
  4. Korsakoff psychosis
  5. Alcohol withdrawal

Ans. The key is D. Korsakoff psychosis. [In Korsakoff psychosis there is confabulation (a memory disturbance, defined as the production of fabricated, distorted or misinterpreted memories about oneself or the world, without the conscious intention to deceive) which may present like this].

719. A 70yo man presented with muscle weakness and inability to climb the stairs. Inv: CPK raised, ESR 15. What is the most likely dx?

  1. Polymyositis
  2. Polymyalgia rheumatic
  3. Reactive arthritis
  4. RS
  5. Duchenne’s MD

Ans. The key is A. Polymyositis. [Proximal muscle weakness (inability to climb the stairs) and raised CPK points towards polymyositis].

720.  A 65yo known alcoholic is brought into the hospital with confusion, aggressiveness and ophthalmoplegia. He is treated with diazepoxide. What other drug would you like to prescribe?

  1. Antibiotics
  2. Glucose
  3. IV fluids
  4. Disulfiram
  5. Vit B complex

Ans. The key is E. Vit B complex. [This is a case of Wernicke’s encephalopathy which is treated with Vit B complex].

721.A pt suffering from schizophrenia laughs while talking about his father’s death. Which term best describes his condition?

  1. Depression
  2. Flat affect
  3. Emotional liability
  4. Incongruent affect
  5. Clang association

Ans. The key is D. Incongruent affect. [Incongruent affect means inappropriate emotional response like loughing hearing a sad news or crying hearing a good news].

 

 

722. A 72yo man has been on warfarin for 2yrs because of past TIAs and strokes. What is the most imp complication the pt should be careful with?

  1. Headache
  2. Osteoporosis
  3. Ear infection
  4. Limb ischemia
  5. Diarrhea

Ans. The key is B. Osteoporosis. It is a wrong key. The correct option is A. Headache. [Headache from intracranial hemorrhage is more important complication about which patient should be careful with (it is more important than osteoporosis)].

723. A 24yo woman is afraid to leave her house as whenever she goes out, she tends to have SOB and sweating. She has stopped going out except with her husband. What is the most likely dx?

  1. Social phobia
  2. Claustrophobia
  3. Depression
  4. Panic disorder
  5. Agoraphobia

Ans. The key is E. Agoraphobia. [Agoraphobia: Extreme or irrational fear of open or public places].

724.  A pt on HTN drugs develops hyperkalemia. Which anti-HTN is likely to cause it?

  1. Ramipril
  2. Lorsartan
  3. Thiazide
  4. Nifedipine
  5. Furosemide

Ans. The key is A. Ramipril. [Both ramipril and losartan can cause hyperkalemia].

725. A young man develops itching worse at night and following bathing. Exam: greysish white linear rash can be seen on the wrist and periumbilical area. What is the dx?

  1. Scabies
  2. Polycythemia
  3. Urticarial
  4. Atopic eczema
  5. Lichen planus

Ans. The key is A. Scabies.

726. A 40yo lady who has been a smoker since she was a teenager has the following blood result: Hgb=19. What hormone should you check?

  1. Aldosterone
  2. Cortisol
  3. Erythropoietin
  4. T4
  5. TSH

Ans. The key is C. Erythropoetin. [Smoking causes raised carboxyhemoglobin level causing hypoxemia and raised erythropoeitin which leads to raised hemoglobin level (secondary polycythemia)].

727. A 25yo man presents with hoarseness of voice. He has swollen vocal cords. His BMI=32 and he smokes 20-25 cigarettes/day. What would you advise him?

  1. Stop smoking
  2. Lose weight

Ans. The key is A. Stop smoking.

 

728.  A 64yo male was admitted to the medical ward with complaint of diarrhea, abdominal pain and weight loss for few months. Exam: clubbing, perianal skin tags and abdominal tenderness. Colonscopy reveals transmural granulomatous inflammation involving ileocecal junction. He was dx with what?

  1. CD
  2. UC
  3. Bowel cancer
  4. Gastric cancer
  5. IBD

Ans. The key is A. CD. [The given picture is typical of Crohn’s disease].

 

729. A pt presents with hemoptysis 7d after tonsillectomy. What is the next step?

  1. Packing
  2. Oral antibiotics and discharge
  3. Admit and IV antibiotics
  4. Return to OT and explore
  5. Ice cream and cold fluids

Ans. The key is C. Admit and IV antibiotics. [Secondary hemorrhage can occur from sloughing of tissue from surgical wound following infection].

730.  A 55yo man presents with HTN. He complains of headache and visual disturbances. He also reports itching after a hot bath and

burning sensation in finger and toes. His face is flushed red. PE: mild splenomegaly. Inv: Hgb=20g/dl, WBC=20, plt=500, EPO normal.

What is the likely dx?

  1. Myelofibrosis
  2. Polycythemia rubra vera
  3. Essential thrombocythemia
  4. CML
  5. CLL

Ans. The key is B. Polycythemia rubra vera. [Raised hemglobin, raised cell counts and normal erythropoeitine along with symptoms of hyperviscosity like headache and associated hypertension are diagnostic of polycythemia rubra vera].

 

 

731. An old man having T2DM with increased skin tanning, heart failure and having high ferritin (hemochromatosis) level is refusing tx. Where is the first site of cancer?

  1. Testes
  2. Adrenal
  3. Liver
  4. Pancreas

Ans. The key is C. Liver.

 

732. A 60yo DM lady presents with severe peri-anal pain and swelling. What’s the cause?

  1. Anal carcinoma
  2. Anal fissure
  3. Hemorrhoid
  4. Anal abscess

Ans. The key is D. Anal abscess. [Diabetes predisposes infection to occur easily].

733. A woman is sad, fatigues and she is eating more and also has sleeping disturbance and hears the voice of her husband who died 3yrs ago. What is the dx?

  1. OCD
  2. Psychotic depression
  3. Grieving
  4. Severe depression

Ans. The key is B. Psychotic depression. [Here features of atypical depression along with hallucination makes the likely diagnosis to be psychotic depression].

734.  A 40yo teetotaler woman is recovering from a hysterectomy 2days ago. At night she becomes agitated and complains of seeing animals and children walking around the ward. What is the most likely dx?

  1. Delirium tremens
  2. Toxic confusional state
  3. Hysteria
  4. Mania
  5. Drug induced personality disorder

Ans. The key is B. Toxic confusional state. [This is not delirium tremens as the womean is teetotaler].

 

 

735.  A woman with a hx of drug abuse and increased alcohol intake, now comes for help and she is concerned about her problem. What is the most appropriate management option?

  1. Voluntary admission
  2. Psychiatry team
  3. Mental health team
  4. Psychiatry voluntary admission

Ans. The key is B. Psychiatry team.

736.  A 28yo woman who is 32 wks pregnant in her 3rd pregnancy is diagnosed as a case of placental abruption. After all the effective measures, she is still bleeding. What is the underlying pathology?

  1. Clotting factor problem
  2. Clauser’s syndrome
  3. Platelet problem
  4. Succiturate lobe
  5. Villamentous insertion of placenta

Ans. The key is A. Clotting factor problem.

737. An old woman having decreased vision can’t see properly at night. She has changed her glasses quite a few times but with no effect. She has normal pupils and cornea. What is the most likely dx?

  1. Cataract
  2. Glaucoma
  3. Retinal detachment
  4. Iritis
  5. GCA

Ans. The key is A. Cataract. [glaucoma may have dilated oval pupil].

738.  A 53yo man was admitted to the hospital for inv of hemoptysis. 3 days after admission he developed alternating state of consciousness, ataxic gait and some visual problems. What is the most appropriate management of this pt?

  1. Acamprosate
  2. Chlordiazepoxide
  3. Diazepam
  4. High potent vitamins
  5. Disulfiram

Ans. The key is D. High potent vitamins. [This is a case of Wernicke’s encephalopathy (as hospital admission prevented him from consuming alcohol) and the treatment for this is high potent vitamins].

739.  A pt underwent hip surgery. Later he presents with SOB and chest pain. What is the dx?

  1. Pulmonary embolism
  2. MI
  3. Tension pneumothorax
  4. Fat embolism
  5. None

Ans. The key is A. Pulmonary embolism.

740.  A 25yo man presents with hx of breathlessness. A transthoracic echo reveals a patent foramen ovale. What diagnostic inv would you do for a patent foramen ovale?

  1. Transesophageal echo
  2. Bubble echo
  3. Transthoracic echo
  4. ECG

Ans. The key is B. Bubble echo. [Bubble echo is actually extension of transoesophageal echo in that here additional bubbles are added during transoesophageal echo to get better visualization of foramen ovale].

 

741.A 25yo woman with a hx of several episodes of depression is brought to the ED after she was found with several empty bottles of her meds. She complains of coarse tremor, nausea and vomiting.

Which of the following drugs is likely to have caused her symptoms?

  1. Fluoxetine
  2. Amitryptilline
  3. Lithium
  4. Phenelzine
  5. Olanzapine

Ans. The key is C. Lithium. [Side effect of lithium is fine tremor (in therapeutic dose) but lithium toxicity or lithium poisoning causes coarse tremor. Coarse tremor, nausea and vomiting are well known feature of lithium overdose].

742. A 23yo man feels anxious and agitated when faced with stress. He has an interview in 3days and would like some help in relieving his symptoms. What is the most appropriate management?

  1. SSRI
  2. CBT
  3. Propranolol
  4. Diazepam

Ans. The key is C. Propranolol. [Inappropriate anxiety during interview is performance phobia which is better helped by propranolol when help needed for short term like here as 3 days].

743.  An 8yo boy dx with asthma is on salbutamol and beclomethasone. However, he wakes up at night due to his symptoms. What is the next appropriate management?

  1. LABA
  2. High dose steroid
  3. Aminophylline
  4. Oral prednisolone
  5. Sodium cromoglycate

Ans. The key is A. LABA. [Patient is in step2 with poor control. So next step is to add LABA and if still not controlled give high dose inhaled corticosteroids].

744.  A woman presents with a hx of poisoning 10x with different substances. There are no obvious signs of depression or suicidal behavior. What is the best preventive step?

  1. Open access to ED
  2. 24h help line
  3. CBT
  4. Anti-depressants
  5. Insight into problem

Ans. The key is E. Insight into problem. [Patient is not depressed and there is no suicidal behaviour. Despite repeated poisoning may indicate she is facing some stress and so insight into her problem should be sought for].

745.  A boy was rushed to the ED unconscious after he had taken methadone belonging to the sister. He was given naloxone and he regained consciousness. After a while he started getting drowsy again. What is responsible for his present drop in level of conscious- ness?

  1. Naloxone is absorbed faster than methadone
  2. Methadone is absorbed faster than naloxone
  3. He has also taken another substance apart from methadone
  4. The methadone had already caused some brain damage
  5. Naloxone is eliminated faster than methadone

Ans. The key is E. Naloxone is eliminated faster than methadone.

746.  A 24yo male on remand in prison for murder is referred by the prison doctor. He is noted to be behaving oddly whilst in prison and complains of seeing things. He has a prv hx of IV drug abuse. On questioning he provides inappropriate but approximate answers

to all questions stating that Bill Clinton is the prime minister of England. What is the prisoner suffering from?

  1. Capgras syndrome
  2. Cotard syndrome
  3. Ganser syndrome
  4. Ekbom syndrome
  5. Tourette’s syndrome

Ans. The key is C. Ganser syndrome. [Ganser syndrome is characterized by nonsensical or wrong answers to questions or doing things incorrectly often with visual pseudohallucinations and a decreased state of consciousness. It is also sometimes called prison psychosis, because the syndrome occurs most frequently in prison inmates, where it may represent an attempt to gain leniency from prison or court officials].

747. A 32yo lady has recently become more active, sleeps less and bought a house and 2 new cars. What is the most likely dx?

  1. Bipolar disorder
  2. Mania
  3. Hypomania
  4. Schizophrenia

Ans. The key is C. Hypomania. [Individuals in a hypomanic state have a decreased need for sleep, are extremely outgoing and competitive, have a great deal of energy and are otherwise often fully functioning (unlike full mania where unlike fully functioning there is a. obvious difficulties at work or in social relationships and activities b. requires admission to hospital to protect the person or others, or c. the person is suffering psychosis)].

748.  The body of a 65yo man who was treated for TB and bronchitis was seen at autopsy. His legs were swollen and his liver showed signs of a transudate fluid. What was the cause of the transudate?

  1. Liver cirrhosis
  2. Alcoholic liver disease
  3. Cardiac failure
  4. Budd-chiari syndrome
  5. TB

Ans. The key is C. Cardiac failure.

749.  A 60yo woman has tiredness. She has noticed that her skin looks permenantly tanned and she describes dizziness on standing up. What is the single most likely electrolyte pattern to be found?

  1. Na+=120mmol/L, K+=5.9mmol/L
  2. Na+=125mmol/L, K+=2.9mmol/L
  3. Na+=140mmol/L, K+=4.5mmol/L
  4. Na+=150mmol/L, K+=3.5mmol/L
  5. Na+=150mmol/L, K+=5.9mmol/L

Ans. The key is A. Na+=120mmol/L, K+=5.9mmol/L. [Tanned skin (hyperpigmentation) and postural hypotension in a tired women points towards Addison’s disease where hyponatremia and hyperkalemia is seen].

750.  A 20yo girl with amenorrhea and BMI of 14 still thinks she has to lose weight. What is the most likely dx?

  1. Anorexia nervosa
  2. Bulimia nervosa
  3. OCD
  4. Depression
  5. Body dysmorphic disorder

Ans. The key is A. Anorexia nervosa.

 

751.A guy who has several convictions and has been imprisoned several times, breaks up with his family and doesn’t contact his children. What type of personality disorder is this?

  1. Borderline
  2. Antisocial
  3. Schizotypal
  4. Schizoid
  5. Criminal

Ans. The key is B. Antisocial. [Antisocial personality disorder is characterized by a pervasive pattern of disregard for, or violation of, the rights of others. There may be an impoverished moral sense or conscience and a history of crime, legal problems, and impulsive and aggressive behavior].

 

 

752. A 17yo lady presents with a worm in her ear. She is very agitated and anxious. What is the next step?

  1. Remove under GA
  2. Suction
  3. Alcohol drops
  4. Forceps

Ans. The key is C. Alcohol drops. [It is a living object. So it should be killed first and so alcohol drop is the correct option].

753. A 20yo male smoker is noted to have intense rubor of the feet and absent foot pulse. Exam: amputated right 2nd toe. What is the most probable dx?

  1. Intermittent claudication
  2. Cardiovascular syphilis
  3. Buerger’s disease
  4. Embolism
  5. Acute limb ischemia

Ans. The key is C. Buerger’s disease. [ The traditional diagnosis of Buerger’s disease is based on 5 criteria (smoking history, onset before the age of 50 years, infrapopliteal arterial occlusive disease, either upper limb involvement or phlebitis migrans, and absence of

atherosclerotic risk factors other than smoking). As there is no specific diagnostic test and an absence of positive serologic markers, confi- dent clinical diagnosis should be made only when all these 5 criteria have been fulfilled].

754.  A young lady after a heavy bout of drinking last night comes to the ED with dizziness, abdominal pain, vomiting blood with cool peripheries. After initial resuscitation, oxygen and fluids, she still continues to bleed with pulse=130bpm and BP=85/58mmHg. What would be your next best management?

  1. Clotting screen
  2. US
  3. CT
  4. Endoscopy
  5. Omeprazole

Ans. The key is D. Endoscopy. [In young it is unlikely to get oesophageal varices rather than the bleeding is from probable gastric erosion and endoscopy should be done (diagnostic and therapeutic- like sclerotherapy].

755. A 12yo boy complains of acute development of purpura on the dependent areas of his body 2wks after a URTI. The purpura doesn’t blanch on pressure, tests reveal: Hgb=12, plts=50, WBC=5. Bleeding time=10mins, APTT=40s, PT=1.02. What is the most likely dx?

  1. ITP
  2. TTP
  3. Von Willebrand’s disease
  4. Hemophilia A
  5. Hemophilia B

Ans. The key is A. ITP. [Isolated thrombocytopenia and H/O prior URTI with development of purpura on the dependent areas of the body favours the diagnosis of ITP. (In ITP BT is prolonged which is present here.)].

756.  A woman presents with hx of falls, becomes pale and clumsy. She is hypertensive and takes atenolol, bendroflumethiazide and amlodipine. What inv is needed?

  1. 24h ECG
  2. 24h BP monitoring
  3. ECG
  4. Echo
  5. CT head

Ans. The key is B. 24h BP monitoring. [All these drugs (particularly prolonged use of thiazide and amlodipine) are well established cause of postural hypotension. So 24 hour BP monitoring should be done].

757. A 43yo woman has been feeling lethargic and tired. Her BP=160/90mmHg. Bloods: Na+=140mmol/L, K+=3.1mmol/L. What is the most likely dx?

  1. Cushing’s syndrome
  2. Conn’s syndrome
  3. Hyperparathyroidism
  4. Renal disease
  5. Pheochromocytoma

Ans. The key is B. Conn’s syndrome. [Here potassium is < 3.5, sodium is at its high normal and hypertension is present so no confusion that it is conn’s syndrome. Cushing’s syndrome is a D/D but it has typical clinical feature which is usually described if the question is for cush- ing’s syndrome. below is picture reference of conn’s syndrome from patient.info. It can also be cushing’s syndrome! So if we like to be sure we have to know aldosterone and rennin level. i) Renin low, aldoste- rone high- Conn’s syndrome (primary hyperaldosteronism) ii) Renin low, aldosterone low- Cushing’s syndrome iii) Renin high, aldosterone high- Renovascular disease (secondary hyperaldosteronism)].

 
  

 

Reference: patient.info

758.  A 2yo child aspirated a foreign object which was removed at the hosp. the parents are now asking how to remove it if that ever happens at home. What do you advise?

  1. Hemlich maneuver
  2. Bring to the hospital
  3. Turn the child on his back and give thumps
  4. CPR
  5. Remove manually by fingers

Ans. The key is C. Turn the child on his back and give thumps.

 

759. A 28yo pregnant lady presents with severe lower abdominal pain with excessive per vaginal bleeding at 34wks gestation. What should be the initial inv of choice?

  1. Coagulation profile
  2. US abdomen
  3. CT pelvis
  4. D-dimer
  5. Kleiuber test

Ans. The key is B. US abdomen. [Likely case of abruption placenta (as excessive bleeding probably revealed type) for which initial investiga- tion of choice is US abdomen. Other features that will favour abruptio is hard uterus and evidence of fetal distress. It will be difficult to feel the fetal parts].

760.  A 3yo child with severe diarrhea and vomiting, looks lethargic, has sunken eyes and a feeble cry. What is the choice of fluids?

  1. 9%NS
  2. 9%NS + 5%Dextrose
  3. 45%NS + 5%Dextrose
  4. 0.45%NS

Ans. The key is A. 0.9% NS. [Patient has features of severe dehy- dration and should be resuscitated first and for resuscitation fluid of choice is 0.9% NS].

761.A lady with depression has a bag full of meds. She now presents with coarse tremors. Which drug caused her symptoms?

  1. Lithium
  2. Thyroxine
  3. Amitriptyline
  4. Sodium valproate
  5. Tetrabenazine

Ans. The key is A. Lithium. It is a wrong key! Right key is D. Sodium valproate. [Side effect of lithium in therapeutic range is fine tremor. Sodium valproate (a well known mood stabilizer) can cause coarse tremor in therapeutic range].

762. A 38yo man has had a liver biopsy as part of inv for abnormal LFTs. The pathologist report states: special stains demonstrate the presence of a very large amount of iron pigment within the hepato- cytes. What single condition is identified by the pathologists report?

  1. Alpha 1 antitrypsin deficiency
  2. Hemangioma
  3. Hemochromatosis
  4. Hemosiderosis
  5. Wilson’s disease

Ans. The key is D. Hemosiderosis. It is a wrong key! The correct key

should be C. Hemochromatosis. [In hemochromatosis distribution pattern of iron in liver biopsy by pearl’s stain is more prominent iron granules in periportal hepatocyte and relative sparing of kupffer cells and decreased intensity near the central vein. By contrast iron overload in hemosiderosis causes to accumulation of iron granules predominantly in kupffer cells and more in central area rather than peripheral hepatocyte. In given case there are large amount of iron pigment in hepatocyte which is diagnostic of hemochromatosis].

 

 

763. A 27yo man presents to the ED after an RTA where his foot was stuck under a truck for several hours. He now has swelling on the foot. Exam: foot tenderness, loss of sense in the space between the 3rd metatarsal and big toe and his dorsalis pedis is not felt. What is the most likely dx?

  1. Compartment syndrome
  2. Arterial rupture
  3. Arterial embolus
  4. DVT
  5. Fibular fx

Ans. The key is A. Compartment syndrome. [Acute compartment syn- drome occurs after a traumatic injury such as a car crash. The trauma causes a severe high pressure in the compartment which results in insufficient blood supply to muscles and nerves. Acute compartment syndrome is a medical emergency that requires surgery to correct. If untreated, the lack of blood supply leads to permanent muscle and nerve damage and can result in the loss of function of the limb].

764.  You are a FY doctor in the ED when a mother brings her 2yo son to you with a 1h hx of noisy breathing. She state that although he had mild coryza over the last week, he was improving and so they had gone to a children’s picnic with nursery friends. Another parent had found him coughing and spluttering, and ever since his breath-

ing has remained noisy. Though he appears well in the ED, his current observations demonstrate a raised RR and sat=91% on air. What is the most likely dx?

  1. Anaplyaxis
  2. Croup
  3. Foreign body aspiration
  4. Epiglottitis

Ans. The key is C. Foreign body aspiration.

765. A pt taking doxycycline complains of nausea, indigestion, ab- dominal pain and vomiting. What will you advise?

  1. Take it after meals
  2. Take it before meals
  3. Stop the drug
  4. Take antiacids
  5. Take antiemetic

Ans. The key is A. Take it after meal.

766. A 52yo man with hx of ant MI 3 wks ago developed sudden onset of dyspnea. Exam: BP=100/60mmHg, pulse=100bpm, SaO2=88%, chest=bilateral crackles. Which inv would you do to find the underlying cause?

  1. CXR
  2. Echo
  3. D-dimer
  4. Radionucleotide lung scan
  5. Troponin

Ans. The key is A. CXR. It is wrong key. Correct key should be B. Echo. [The question does not want to know the features of pulmonary oedema rather it is clearly mentioned “which investigation would you do to find out the UNDERLYING CAUSE”. As in this patient sudden papillary muscle rupture or VSD may be the likely cause (ventricular aneurism may take a more slower course) so echo seems to be more logical approach].

767. A 49yo man lost his job and now is homeless. He was found wandering in the park. He is muttering that some people are after him. Alcohol was tested and it was negative. What will your next step be?

  1. Thiamine
  2. Neuropsycho analysis
  3. Mini mental state
  4. CT head
  5. MRI head

Ans. The key is B. Neuropsychoanalysis. [As the patient has persecu- tory delusion he is likely suffering from psychosis for which neuropsy- choanalysis is the logical approach].